Sunteți pe pagina 1din 88

1

Laws on Natural Resources Cases Full Text

1. Sunbeam Convenience Food vs CA 181 SCRA 443

G.R. No. L-50464 January 29, 1990

SUNBEAM CONVENIENCE FOODS INC., CORAL BEACH DEVELOPMENT CORP., and the
REGISTER OF DEEDS OF BATAAN, petitioners,
vs.
HON. COURT OF APPEALS and THE REPUBLIC OF THE PHILIPPINES, respondents.

Filoteo T. Banzon for petitioners.

SARMIENTO, J.:

In this petition for review on certiorari, Convenience Foods Corporation (hereafter simply
SUNBEAM) and Coral Beach Development Corporation (hereafter simply CORAL BEACH) bring to
our attention the decision rendered by the Court of Appeals in "Republic of the Philippines v. Hon.
Pedro T. Santiago, et al.," disposing as follows:

WHEREFORE, the writ prayed for is granted. The order of the respondent judge dated October 7,
1977, dismissing Civil Case No. 4062 is set aside, and respondent judge is ordered to require private
respondents to file their answer to the complaint in said Civil Case No. 4062 and thereafter to
proceed with the trial of the case on the merits and to render judgment thereon.'

The following facts stated by the respondent Court in its decision and restated by the petitioners in
their petition are accurate:

(a) On April 29, 1963, the Director of Lands caused the issuance of a Sales Patent in favor of
defendant Sunbeam Convenience Foods, Inc., over the parcels of land both situated in Mariveles,
Bataan and more particularly described and bounded as follows:

Lot 1-Sgs-2409 (area 3,113,695 sq. m )

Lot 2-Sgs-2409 area 1,401,855 sq. m

(b) On May 3, 1963, the aforesaid Sales Patent was registered with the defendant Register of Deeds
of Bataan who in turn issued Original Certificate of Title No. Sp-24 in favor of defendant Sunbeam
Convenience Foods, Inc., for the two parcels of land above-described;

(c) Subsequently, Original Certificate of Title No. Sp-24 was cancelled and in lieu thereof, Transfer
Certificate of Title No. T-12421 was issued over Lot 1, Sgs-2409, while Transfer Certificate of Title
Full Text Cases in Natural Resources
2
No. 12422 was issued over Lot 2, Sgs-2409, both in favor of defendant Coral Beach Development
Corporation I

(d) On May 11, 1976, the Solicitor General in the name of the Republic of the Philippines instituted
before the Court of First Instance of Bataan, an action for reversion docketed as Civil Case No.
4062. 2

SUNBEAM and CORAL BEACH filed a Motion to Dismiss on the following grounds:

1. The Republic of the Philippines should have exhausted all administrative remedies before filing
the case in court;

2. The title issued to SUNBEAM and CORAL BEACH had become indefeasible and imprescriptible;

3. The action for reversion was defective, having been initiated by the Solicitor General and not by
the Director of Lands. 3

The then Court of First Instance of Bataan dismissed the complaint in the Order of October 7,
1977,4 adopting mainly the theory that since the titles sought to be cancelled emanated from the
administrative act of the Bureau of Lands Director, the latter, not the courts, had jurisdiction over
the disposition of the land.

The Solicitor General received the copy of the Order on October 11, 1977 and filed a Notice of
Appeal dated October 25, 1977 .5 The Solicitor General then moved for an extension of thirty days
within which to file the Record on Appeal and to pay the docket fee in order to perfect the appeal.
This was to be followed by another motion for extension filed by the Solicitor General, resulting in
the Court of Appeals granting the petitioner another extension of fifteen days from December 10,
1977. Finally before this period of extension lapsed, instead of an appeal, a petition for certiorari
with the respondent Court of Appeals was filed.

According to the Solicitor General, the Court of First Instance committed grave abuse of discretion
in dismissing the complaint and in

a. Not finding that since the lower court acted in a Motion to Dismiss, the correctness of its decision
must be decided in the assumed truth and accuracy of the allegations of the complaint. The
complaint alleges that the lands in question are forest lands; hence, inalienable.

b. Finding that Lots I and 2 are alienable and disposable lands of the public domain under the
jurisdiction of the Director of Lands despite clear and positive evidence to the contrary.

c. Concluding that the complaint for reversion is defective as it was not initiated by the Director of
Lands.

d. Finding that the complaint for reversion states no cause of action for alleged failure of petitioner
to exhaust administrative remedies. 6

Full Text Cases in Natural Resources


3
The Court of Appeals gave due course to the petition for certiorari, set aside the Order of Dismissal
rendered by the Court of First Instance in Civil Case No. 4062, and ordered the presiding judge
Hon. Pedro T. Santiago to receive the answers of the private respondents SUNBEAM and CORAL
BEACH in the action for reversion.

Hence Sunbeam and Coral Beach filed this petition for review.

A review is not a matter of right but of sound judicial discretion, and is granted only when there are
special and important reasons therefore. The following, while neither controlling nor fully measuring
the Court's discretion, enumerates the premises for granting a review:

(a) When the Court of Appeals has decided a question of substance, not theretofore determined by
the Supreme Court or has decided it in a way probably not in accord with law or the applicable
decisions of the Supreme Court; and

(b) When the Court of Appeals has so far departed from the accepted and usual course of judicial
proceedings or so far sanctioned such departure by a lower court as to call for supervision . 7

We agree with the Court of Appeals' granting of the petition filed by the Republic of the Philippines
charging the then Court of First Instance with grave abuse of discretion. The filing of the Motion to
Dismiss the complaint for reversion by SUNBEAM and CORAL BEACH on the ground of lack of
cause of action, necessarily carried with it the admission, for purposes of the motion, of the truth of
all material facts pleaded in the complaint instituted by the Republic.

An important factual issue raised in the complaint was the classification of the lands as forest lands.
This material allegation stated in the Republic's complaint' was never denied specifically 9 by the
defendants (petitioners herein) SUNBEAM and CORAL BEACH.

If it is true that the lands are forest lands, then all these proceedings become moot and academic.
Land remains unclassified land until it is released therefrom and rendered open to disposition. 10

Our adherence to the Regalian doctrine subjects all agricultural, timber, and mineral lands to the
dominion of the State.11 Thus, before any land may be declassified from the forest group and
converted into alienable or disposable land for agricultural or other purposes, there must be a
positive act from the government. Even rules on the confirmation of imperfect titles do not apply
unless and until the land classified as forest land is released in an official proclamation to that effect
so that it may form part of the disposable agricultural lands of the public domain. 12

The mere fact that a title was issued by the Director of Lands does not confer any validity on such
title if the property covered by the title or patent is part of the public forest. 13

The only way to resolve this question of fact as to the classification of the land is by remanding the
case to the lower court for a full- dress trial on the issues involved.

Generally, the rules of procedure must be observed so that the efficient administration of justice is
ensured. However, the rules of procedure should be viewed as mere tools designed to facilitate the

Full Text Cases in Natural Resources


4
attainment of justice.14They
must lead to the proper and just determination of litigation, without tying
the hands of the law or making it indifferent to realities.1âwphi1

Certiorari is one such remedy. Considered extraordinary, it is made available only when there is no
appeal, nor any plain, speedy or adequate remedy in the ordinary course of the law. 15 The long line
of decisions denying the petition for certiorari, either before appeal was availed of or specially in
instances where the appeal period has lapsed, far outnumbers the instances when certiorari was
given due course. The few significant exceptions were: when public welfare and the advancement
of public policy dictate; or when the broader interests of justice so require, or when the writs issued
are null, 16 or when the questioned order amounts to an oppressive exercise of judicial authority. 17

We find nothing disagreeable with the action of the Court of Appeals to give due course to the
petition considering that the issue affected a matter of public concern which is the disposition of the
lands of our matrimony No less than the Constitution protects its policy.

We therefore find no compelling reason to disturb the findings of the appellate court, in the absence
of a clear showing that the Court of Appeals has decided a question of substance in a manner
inconsistent with jurisprudence, or that the respondent Court has departed from the accepted and
usual course of judicial proceedings. In sum, no reversible error has been committed by the
respondent court. 18

WHEREFORE, the petition is DENIED and the decision of the Court of Appeals is affirmed. Costs
against the petitioners.

SO ORDERED.

2. Republic vs Sayo 191 SCRA 71

G.R. No. L-60413 October 31, 1990

REPUBLIC OF THE PHILIPPINES, petitioner,


vs.
HON. SOFRONIO G. SAYO, Judge, Br. I, C I, Nueva Vizcaya, HEIRS OF CASIANO SANDOVAL,
HEIRS OF LIBERATO BAYAUA, JOSE C. REYES, and PHILIPPINE CACAO AND FARM
PRODUCTS, INC., respondents.

Celso D. Gangan respondent Heirs of Liberato Bayaua.

Acosta & Associates fox Phil. Cacao and Farm Products, Inc.

Jose Reyes & Associates for Heirs of Casiano Sandoval, et al.

NARVASA, J.:

Full Text Cases in Natural Resources


5
Sought to be annulled and set aside in this special civil action of certiorari is the decision of
respondent Judge Sofronio G. Sayo rendered on March 5, 1981 in Land Registration Case No. N-
109, LRC Record No. 20850, confirming, by virtue of a compromise agreement, the title of the
private respondents over a tract of land.

The spouses, Casiano Sandoval and Luz Marquez, filed an original application for registration of a
tract of land identified as Lot No. 7454 of the Cadastral Survey of Santiago, BL Cad. 211 (July 17,
1961) and having an area of 33,950 hectares. The land was formerly part of the Municipality of
Santiago, Province of Isabela, but had been transferred to Nueva Vizcaya in virtue of Republic Act
No. 236.

Oppositions were filed by the Government, through the Director of Lands and the Director of
Forestry, and some others, including the Heirs of Liberato Bayaua.1 In due course, an order of
general default was thereafter entered on December 11, 1961 against the whole world except the
oppositors.

The case dragged on for about twenty (20) years until March 3, 1981 when a compromise
agreement was entered into by and among all the parties, assisted by their respective counsel,
namely: the Heirs of Casiano Sandoval (who had since died), the Bureau of Lands, the Bureau of
Forest Development, the Heirs of Liberato Bayaua, and the Philippine Cacao and Farm Products,
Inc. Under the compromise agreement, the Heirs of Casiano Sandoval (as applicants) renounced
their claims and ceded —

1) in favor of the Bureau of Lands, an area of 4,109 hectares;

2) in favor of the Bureau of Forest Development, 12,341 hectares;

3) in favor of the Heirs of Liberato Bayaua, 4,000 hectares; and

4) in favor of Philippine Cacao & Farm Products, Inc., 8,000 hectares.

The remaining area of 5,500 hectares was, under the compromise agreement, adjudicated to and
acknowledged as owned by the Heirs of Casiano Sandoval, but out of this area, 1,500 hectares
were assigned by the Casiano Heirs to their counsel, Jose C. Reyes, in payment of his attorney's
fees. In consideration of the areas respectively allocated to them, all the parties also mutually waived
and renounced all their prior claims to and over Lot No. 7454 of the Santiago Cadastre.

In a decision rendered on March 5, 1981, the respondent Judge approved the compromise
agreement and confirmed the title and ownership of the parties in accordance with its terms.

The Solicitor General, in behalf of the Republic of the Philippines, has taken the present recourse
in a bid to have that decision of March 5, 1981 annulled as being patently void and rendered in
excess of jurisdiction or with grave abuse of discretion. The Solicitor General contends that —

1) no evidence whatever was adduced by the parties in support of their petitions for registration;

Full Text Cases in Natural Resources


6
2) neither the Director of Lands nor the Director of Forest Development had legal authority to enter
into the compromise agreement;

3) as counsel of the Republic, he should have been but was not given notice of the compromise
agreement or otherwise accorded an opportunity to take part therein;

4) that he was not even served with notice of the decision approving the compromise; it was the
Sangguniang Panlalawigan of Quirino Province that drew his attention to the "patently erroneous
decision" and requested him to take immediate remedial measures to bring about its annulment.

The respondents maintain, on the other hand, that the Solicitor General's arguments are premised
on the proposition that Lot 7454 is public land, but it is not. According to them, as pointed out in the
application for registration, the private character of the land is demonstrated by the following
circumstances, to wit:

1) the possessory information title of the applicants and their predecessors-in-interest;

2) the fact that Lot 7454 was never claimed to be public land by the Director of Lands in the proper
cadastral proceedings;

3) the pre-war certification of the National Library dated August 16, 1932 to the effect that the
(Estadistica de Propiedades) of Isabela issued in 1896 and appearing in the Bureau of Archives,
the property in question was registered under the 'Spanish system of land registration as private
property owned by Don Liberato Bayaua, applicants' predecessors-in-interest;

4) the proceeding for registration, brought under Act 496 (the Torrens Act) presupposes that there
is already a title to be confirmed by the court, distinguishing it from proceedings under the Public
Land Act where the presumption is always that the land involved belongs to the State.

Under the Regalian Doctrine 2 all lands not otherwise appearing to be clearly within private
ownership are presumed to belong to the State. Hence it is that all applicants in land registration
proceedings have the burden of overcoming the presumption that the land thus sought to be
registered forms part of the public domain. 3 Unless the applicant succeeds in showing by clear and
convincing evidence that the property involved was acquired by him or his ancestors either by
composition title from the Spanish Government or by possessory information title, or any other
means for the proper acquisition of public lands, the property must be held to be part of the public
domain . 4 The applicant must present competent and persuasive proof to substantiate his claim;
he may not rely on general statements, or mere conclusions of law other than factual evidence of
possession and title. 5

In the proceeding at bar, it appears that the principal document relied upon and presented by the
applicants for registration, to prove the private character of the large tract of land subject of their
application, was a photocopy of a certification of the National Library dated August 16, 1932 (already
above mentioned) to the effect that according to the Government's (Estadistica de Propiedades) of
Isabela issued in 1896, the property in question was registered under the Spanish system of land
registration as private property of Don Liberato Bayaua. But, as this Court has already had occasion
to rule, that Spanish document, the (Estadistica de Propiedades,) cannot be considered a title to

Full Text Cases in Natural Resources


7
property, it not being one of the grants made during the Spanish regime, and obviously not
constituting primary evidence of ownership. 6 It is an inefficacious document on which to base any
finding of the private character of the land in question.

And, of course, to argue that the initiation of an application for registration of land under the Torrens
Act is proof that the land is of private ownership, not pertaining to the public domain, is to beg the
question. It is precisely the character of the land as private which the applicant has the obligation of
establishing. For there can be no doubt of the intendment of the Land Registration Act, Act 496, that
every applicant show a proper title for registration; indeed, even in the absence of any adverse
claim, the applicant is not assured of a favorable decree by the Land Registration Court, if he fails
to establish a proper title for official recognition.

It thus appears that the decision of the Registration Court a quo is based solely on the compromise
agreement of the parties. But that compromise agreement included private persons who had not
adduced any competent evidence of their ownership over the land subject of the registration
proceeding. Portions of the land in controversy were assigned to persons or entities who had
presented nothing whatever to prove their ownership of any part of the land. What was done was to
consider the compromise agreement as proof of title of the parties taking part therein, a totally
unacceptable proposition. The result has been the adjudication of lands of no little extension to
persons who had not submitted any substantiation at all of their pretensions to ownership, founded
on nothing but the agreement among themselves that they had rights and interests over the land.

The assent of the Directors of Lands and Forest Development to the compromise agreement did
not and could not supply the absence of evidence of title required of the private respondents.

As to the informacion posesoria invoked by the private respondents, it should be pointed out that
under the Spanish Mortgage Law, it was considered a mode of acquiring title to public lands, subject
to two (2) conditions: first, the inscription thereof in the Registry of Property, and second, actual,
public, adverse, and uninterrupted possession of the land for twenty (20) years (later reduced to ten
[10] years); but where, as here, proof of fulfillment of these conditions is absent, the informacion
posesoria cannot be considered as anything more than prima facie evidence of possession. 7

Finally, it was error to disregard the Solicitor General in the execution of the compromise agreement
and its submission to the Court for approval. It is, after all, the Solicitor General, who is the principal
counsel of the Government; this is the reason for our holding that "Court orders and decisions sent
to the fiscal, acting as agent of the Solicitor General in land registration cases, are not binding until
they are actually received by the Solicitor General." 8

It thus appears that the compromise agreement and the judgment approving it must be, as they are
hereby, declared null and void, and set aside. Considerations of fairness however indicate the
remand of the case to the Registration Court so that the private parties may be afforded an
opportunity to establish by competent evidence their respective claims to the property.

WHEREFORE, the decision of the respondent Judge complained of is ANNULLED and SET ASIDE.
Land Registration Case No. N-109 subject of the petition is REMANDED to the court of origin which
shall conduct further appropriate proceedings therein, receiving the evidence of the parties and

Full Text Cases in Natural Resources


8
thereafter rendering judgment as such evidence and the law may warrant. No pronouncement as to
costs.

SO ORDERED.

3. Seville vs National Development Company GR No. L-129401 02/02/2001

[G.R. No. 129401. February 2, 2001]


FELIPE SEVILLE in his capacity as judicial administrator of the estate of JOAQUIN ORTEGA and/or
FELIPE SEVILLE, EMILIA ESTRADA, MARIA S. TELLDER, MA. ISABEL SEVILLE, MA. TERESITA
LICARDO, FRANCISCO SEVILLE, RAMON O. SEVILLE, JOSE MARIE SEVILLE, GEMMA ALVAREZ-
ASAYAS, ANNABELLE ALVAREZ-GONZALES, SYLVIA ALVAREZ-LIOK, ADOLFO O. ALVAREZ JR.,
DIANA ALVAREZ-DABON, MARIA SALVADOR O. POLANCOS and JOAQUIN ORTEGA II as
successors-in-interest of JOAQUIN ORTEGA and his estate, petitioners, vs. NATIONAL
DEVELOPMENT COMPANY, LEYTE SAB-A BASIN DEVELOPMENT AUTHORITY, PHILIPPINE
ASSOCIATED SMELTING AND REFINING CORPORATION, LEPANTO CONSOLIDATED MINING
CO., PHILIPPINE PHOSPHATE FERTILIZER CORPORATION, CALIXTRA YAP and REGISTER OF
DEEDS OF LEYTE, respondents.
DECISION
PANGANIBAN, J.:
Unless a public land is shown to have been reclassified as alienable or actually alienated by the State
to a private person, that piece of land remains part of the public domain. Hence, occupation thereof,
however long, cannot ripen into ownership.
The Case

Before us is a Petition for Review on Certiorari assailing the November 29, 1996 Decision of the Court
of Appeals[1] (CA), as well as the May 19, 1997 CA Resolution[2] denying the Motion for
Reconsideration. The dispositive part of the CA Decision reads as follows:
WHEREFORE, the appealed decision is REVERSED and SET ASIDE. Another judgment is hereby
rendered dismissing the complaint. The counterclaims of appellants are denied. Costs against
plaintiffs-appellees.[3]
The Facts

The appellate court narrated the undisputed facts in this manner:


1. By virtue of Presidential Decree No. 625, Leyte Sab-A Basin Development Authority (LSBDA) was
created to integrate government and private sector efforts for a planned development and balanced
growth of the Sab-a Basin in the [P]rovince of Leyte, empowered to acquire real property in the
successful prosecution of its business. Letter of Instruction No. 962 authorized LSBDA to acquire
privately-owned lands circumscribed in the Leyte Industrial Development Estate (LIDE) by way of
negotiated sales with the landowners.
2. On June 14, 1980, [Respondent] Calixtra Yap sold to LSBDA Lot No. 057 SWO 08-000047 consisting
of 464,920 square meters, located at Barangay Sto. Rosario, Isabel, Leyte, covered under Tax
Declarations Nos. 3181, 3579, 3425, 1292 and 4251 under the name of said vendor.
3. On June 1, 1982, appellant LSBDA filed a Miscellaneous Sales Application with the Bureau of Lands
covering said lot together with other lots acquired by LSBDA with an aggregate area of 442, 7508
square meters.

Full Text Cases in Natural Resources


9
4. After due notice and investigation conducted by the Bureau of Lands, Miscellaneous Sales Patent
No. 9353 was issued in the name of [Respondent] LSBDA on the basis of which Original Certificate of
Title No. P-28131 was transcribed in the Registration Book for the [P]rovince of Leyte on August 12,
1983 in the name of [Respondent] LSBDA. On December 14, 1989, LSBDA assigned all its rights over
the subject property to its [Co-respondent] National Development Company (NDC) as a result of which
a new Transfer Certificate of Title was issued on March 2, 1990 by the Registry of Deeds for the
province of Northern Leyte in the name of NDC. The subject property was leased to [Respondents]
Philippine Associated Smelting & Refining Corporation (PASAR), Philippine Phosphate Fertilizer
Corporation (PHILPHOS) and Lepanto Consolidated Mining Co., Inc. (LEPANTO).
5. On November 29, 1988, the Estate of Joaquin Ortega represented by judicial administrator Felipe
Seville filed with the Regional Trial Court (Branch 12) of Ormoc City, a complaint for recovery of real
property, rentals and damages against the above-named [respondents] which complaint was later on
amended on May 11, 1990. [Respondents] filed their respective Answers. After trial, the trial court
rendered judgment the dispositive portion of which reads as follows:
WHEREFORE, [a] decision is hereby rendered for [petitioners] and against [respondents].
1. The Deed of Sale executed by Calixtra Yap on June 14, 1980 in favor of LSBDA, (Exhibit PP and
25) conveying the subject property to said LSBDA is declared NULL and VOID ab initio;
2. The intestate estate of JOAQUIN ORTEGA is declared the owner in fee simple of the 735,333 square
meters real property subject of the present action and defendant NDC is ordered to segregate the same
area from OCT P-28131 and CONVEY the same to the Estate of Joaquin Ortega;
3. Upon the segregation of the 735,333 square meters from OCT No. P-28131 the Register of Deeds
of the Province of Leyte is ordered to issue a new title to the said portion in the name of the Intestate
Estate of Joaquin Ortega;
4. [Respondents] LSBDA, NDC, PASAR, are ordered to pay jointly and severally to [petitioners] the
sum of FOUR MILLION SEVEN HUNDRED EIGHTY FOUR THOUSAND EIGHT HUNDRED FORTY
SIX PESOS (P4,784,846.00) as rentals due from 1979 to the present, plus accrued interest pursuant
to par. 2 of the Lease Contract between NDC and PASAR. (Exhibit 54)
5. [Respondents] LSBDA, NDC, and PHILPHOS are also ordered to pay jointly and severally
[petitioners] the sum of TWO MILLION EIGHTY SIX THOUSAND THREE HUNDRED NINETY EIGHT
PESOS AND SIXTY CENTAVOS (P2,086,398.60) as accrued rentals of PHILPHOS from 1979 to
present, plus the accrued interest for non-payment pursuant to paragraph 2 of the same Lease Contract
cited above;
6. [Respondents] are ordered to pay jointly and severally [petitioners] P200,000.00 as indemnity for the
value of the ancestral home;
7. [Respondents] are also ordered to pay jointly and severally [petitioners] the sum of P250,000.00 as
reimbursement for attorneys fees and the further sum of P50,000.00 as expenses for litigation;
8. Finally, [petitioners] and [respondents] are ordered to sit down together and discuss the possibility
of a compromise agreement on how the improvements introduced on the landholding subject of the
present suit should be disposed of and for the parties to submit to this Court a joint manifestation relative
thereto. In the absence of any such compromise agreement, such improvements shall be disposed of
pursuant to Article 449 of the New Civil Code.
Costs against [respondents].
SO ORDERED.[4]
Ruling of the Court of Appeals

Citing the Regalian doctrine that lands not appearing to be privately owned are presumed to be part of
the public domain, the CA held that, first, there was no competent evidence to prove that the property
in question was private in character. Second, possession thereof, no matter how long, would not ripen
Full Text Cases in Natural Resources
10
into ownership, absent any showing that the land had been classified as alienable.Third, the property
had been untitled before the issuance of the Miscellaneous Sales Patent in favor of the LSBDA. Fourth,
petitioners were guilty of laches, because they had failed to apply for the judicial confirmation of their
title, if they had any. Fifth, there was no evidence of bad faith on the part of LSBDA in dealing with Yap
regarding the property.
Hence, this Petition.[5]

The Issues

In their Memorandum, petitioners submit the following issues for the consideration of the Court: [6]
A. Whether or not the sale by Calixtra Yap of the Estate of the Late Joaquin Ortega in favor of LSBDA
was null and void.
B. Whether or not the issuance of a Miscellaneous Sales Patent and an Original Certificate of Title in
favor of LSBDA was valid.
C. Whether or not petitioners are guilty of laches.
D. Whether or not petitioners are entitled to the remedy of reconveyance and the damages awarded
by the trial court.
In the main, the Court is called upon to determine the validity of LSBDAs title. In resolving this issue, it
will also ascertain whether, before the issuance of the title, the land was private or public.
The Courts Ruling

The Petition has no merit.


Main Issue:
Validity of LSBDAs Title

Petitioners argue that LSBDAs title to 73 hectares of the 402-hectare Leyte Industrial Development
Estate was void, having allegedly been obtained from Calixtra Yap who had no right to it.They maintain
that they acquired title to the disputed property by acquisitive prescription, because they and their
predecessors in interest had been in possession of it for more than thirty years. [7]Although it was the
subject of settlement proceedings, petitioners further claim that Yap sold the same to LSBDA without
the permission of the trial court.
Disputing these contentions, respondents and the appellate court maintain that petitioners have not
shown that the land had previously been classified as alienable and disposable. Absent such
classification, they argue that possession of it, no matter how long, could not ripen into ownership.
We agree with respondents and the appellate court. First. There was no showing that the land had
been classified as alienable before the title was issued to LSBDA; hence, petitioners could not have
become owners thereof through acquisitive prescription. Second, petitioners challenge to LSBDAs title
cannot be granted, because it is based on a wrong premise and amounts to a collateral attack, which
is not allowed by law.

Public Character of the Land

Under the Regalian doctrine, all the lands of the public domain belong to the State, which is the source
of any asserted right to ownership of land. All lands not otherwise appearing to be clearly within private
ownership are presumed to belong to the State.[8] In Menguito v. Republic,[9] the court held that [u]nless
public land is shown to have been reclassified or alienated to a private person by the State, it remains
part of the inalienable public domain. Indeed, occupation thereof in the concept of owner, no matter
how long, cannot ripen into ownership and be registered as a title. To overcome such presumption,

Full Text Cases in Natural Resources


11
incontrovertible evidence must be shown by the applicant. Absent such evidence, the land sought to
be registered remains inalienable.
A person in open, continuous, exclusive an notorious possession of a public land for more than thirty
years acquires an imperfect title thereto. That title may be the subject of judicial confirmation, pursuant
to Section 48 of the Public Land Act, which provides:
SECTION 48. The following described citizens of the Philippines, occupying lands of public domain or
claiming to own any such lands or an interest thereon, but whose titles have not been perfected or
completed, may apply to the Court of First Instance of the province where the land is located for
confirmation of their claims, and the issuance of a certificate of title therefore, under the Land
Registration Act, to wit:
xxx xxx xxx
(b) those who by themselves or through their predecessor in-interest have been in open, continuous,
exclusive and notorious possession and occupation of agricultural lands of the public domain, under a
bona fide claim of acquisition or ownership, for at least thirty years immediately preceding the filing of
the application for confirmation of title except when prevented by war or force majeure. They shall be
conclusively presumed to have performed all the conditions essential to a Government grant and shall
he entitled to a certificate of title under the provisions of this Chapter.
Under Section 4 of Presidential Decree (PD) No. 1073,[10] paragraph b of the aforecited provision
applies only to alienable and disposable lands of the public domain. The provision reads:
SEC. 4. The provisions of Section 48 (b) and Section 48 (c), Chapter VIII, of the Public Land Act, are
hereby amended in the sense that these provisions shall apply only to alienable and disposable lands
of the public domain which have been in open, continuous, exclusive and notorious possession and
occupation by the applicant himself or thru his predecessor-in-interest, under a bona fide claim of
acquisition of ownership, since June 12, 1945.
It should be stressed that petitioners had no certificate of title over the disputed property. Although they
claim that their title was based on acquisitive prescription, they fail to present incontrovertible proof that
the land had previously been classified as alienable. They simply brush aside the conclusion of the CA
on this crucial point by saying that it was without factual basis.[11]Instead, they maintain that the private
character of the land was evidenced by various tax declarations, Deeds of Sale, and Decisions of the
trial court and even the Supreme Court.[12]
Petitioners arguments are not convincing. Tax declarations are not conclusive proofs of ownership, let
alone of the private character of the land. At best, they are merely indicia of a claim of
ownership.[13] In Spouses Palomo v. CA,[14] the Court also rejected tax declarations as proof of private
ownership, absent any showing that the forest land in question had been reclassified as alienable.
Moreover, the Deeds of Sale of portions of the disputed property, which Joaquin Ortega and several
vendors executed, do not prove that the land was private in character. The question remains:What was
the character of the land when Ortega purchased it? Indeed, a vendee acquires only those rights
belonging to the vendor. But petitioners failed to show that, at the time, the vendors were already its
owners, or that the land was already classified as alienable.
Also misplaced is petitioners reliance on Ortega v. CA,[15] in which the Supreme Court allegedly
recognized the private character of the disputed property. In that case, the sole issue was whether the
respondent judge xxx acted in excess of jurisdiction when he converted Civil Case No. 1184-O, an
action for quieting of title, declaration of nullity of sale, and annulment of tax declaration of a parcel of
land, into an action for the declaration of who is the legal wife, who are the legitimate children, if any,
and who are the compulsory heirs of the deceased Joaquin Ortega. [16] The Court did not all make any
ruling that the property had been classified as alienable.
In any event, Ortega arose from a suit for quieting of title, an action quasi in rem that was binding only
between the parties.[17] The present respondents as well as the Bureau of Lands, which subsequently
Full Text Cases in Natural Resources
12
declared that the land was public, are not bound by that ruling, because they were not impleaded
therein.
While petitioners refer to the trial court proceedings supposedly recognizing the private character of the
disputed property, they make no claim that these cases directly involve the classification of the land, or
that the Bureau of Lands is a party thereto.
Clearly, the burden of proof that the land has been classified as alienable is on the claimant. [18] In the
present case, petitioners failed to discharge this burden. Hence, their possession of the disputed
property, however long, cannot ripen into ownership.

LSBDAs Title

Equally unmeritous is the argument of petitioners that the title of LSBDA is void. As earlier stated, they
claim that such title was derived from Calixtra Yap, who was allegedly not the owner of the
property. Petitioners assume that LSBDA, having acquired the rights of Yap, resorted to a confirmation
of her imperfect title under Section 48 of the Public Land Act. This argument is devoid of factual or legal
basis.
Petitioners fail to consider that the title of LSBDA was based, not on the conveyance made by Yap, but
on Miscellaneous Sales Patent No. 9353 issued by the director of the Bureau of Lands. In fact, after
LSBDA had filed an application for patent, the Bureau of Lands conducted an investigation and found
that the land was part of the public domain. After compliance with the notice and publication
requirements, LSBDA acquired the property in a public auction conducted by the Bureau of Lands. [19]
Petitioners insist, however, that LSBDA was estopped from claiming that the land was public, because
the Deed of Sale executed by Yap in its favor stipulated that the seller is the absolute owner in fee
simple of the xxx described property.[20] It is scarcely necessary to address this point. To begin with,
the power to classify a land as alienable belongs to the State, not to private entities. Hence, the
pronouncements of Yap or LSBDA cannot effect the reclassification of the property. Moreover, the
assailed misrepresentation was made by Yap as seller. Hence, objections thereto should be raised not
by petitioners but by LSBDA, the contracting party obviously aggrieved.
In any case, the actions of LSBDA after Yaps conveyance demonstrated its position that the disputed
land was part of the public domain. That this was so can be inferred from LSBDAs subsequent
application for a Miscellaneous Sales Patent and, in a public auction, its purchase of the property from
the Bureau of Lands. Indeed, Yap merely conveyed a claim, not a title which she did not have.
Collateral Attack

There is another reason for denying the present Petition. Petitioners insist that they are not seeking the
re-opening of a decree under the Torrens system. Supposedly, they are only praying for the segregation
of 735,333 square meters of land, or 73 hectares more or less from the OCT No. P-28131 issued to
LSBDA.[21] This disputation is mere quibbling over the words, plain and simple.
Semantics aside, petitioners are effectively seeking the modification of LSBDAs OCT, which allegedly
encompassed even a parcel of land allegedly belonging to them. Hence, the present suit, purportedly
filed for the recovery of real property and damages, is tantamount to a collateral attack not sanctioned
by law. Section 48 of PD 1529, the Property Registration Decree, expressly provides:
SEC. 48. Certificate not subject to collateral attack. -- A certificate of title shall not be subject to collateral
attack. It cannot be altered, modified, or cancelled except in a direct proceeding in accordance with
law.
It has been held that a certificate of title, once registered, should not thereafter be impugned, altered,
changed, modified, enlarged or diminished, except in a direct proceeding permitted by law.Otherwise,
the reliance on registered titles would be lost.[22]
Moreover, the title became indefeasible and incontrovertible after the lapse of one year from the time
of its registration and issuance.[23] Section 32 of PD 1529 provides that [u]pon the expiration of said
Full Text Cases in Natural Resources
13
period of one year, the decree of registration and the certificate of title shall become
incontrovertible. Any person aggrieved buy such decree of registration in any case may pursue his
remedy by action for damages against the applicant or other persons responsible for the
fraud. Although LSBDAs title was registered in 1983, petitioners filed the amended Complaint only in
1990.

Reconveyance

Petitioners also claim that the disputed property should be reconveyed to them. This cannot be
allowed. Considering that the land was public before the Miscellaneous Sales Patent was issued to
LSBDA, petitioners have no standing to ask for the reconveyance of the property to them. The proper
remedy is an action for reversion, which may be instituted only by the Office of the Solicitor General,
pursuant to section 101 of the Public Land Act, which reads as follows:
SEC. 101. All actions for the reversion to the Government of lands of the public domain or
improvements thereon shall be instituted by the Solicitor General or the officer acting in his stead, in
the proper courts, in the name of the [Republic] of the Philippines.
Verily, the prayer for reconveyance and, for that matter, the entire case of petitioners rest on the theory
that they have acquired the property by acquisitive prescription; and that Yap, without any right or
authority, sold the same to LSBDA.

Conclusion

In the light of our earlier disquisition, the theory has no leg to stand on. Absent any showing that the
land has been classified as alienable, their possession thereof, no matter how lengthy, cannot ripen
into ownership. In other words, they have not become owners of the disputed property. Moreover,
LSBDAs title was derived from a Miscellaneous Sales Patent, not from Yap. Finally, petitioners cannot,
by a collateral attack, challenge a certificate of title that has already become indefeasible and
incontrovertible.
If petitioners believe that they have been defrauded by Yap, they should seek redress, not in these
proceedings, but in a proper action in accordance with law.
WHEREFORE, the Petition is hereby DENIED and the assailed Decision AFFIRMED. Costs against
petitioners.
SO ORDERED.

4. Director of Lands vs Immediate Appellate Court 219 SCRA 108

G.R. No. 73246 March 2, 1993

DIRECTOR OF LANDS AND DIRECTOR OF FOREST DEVELOPMENT, petitioners,


vs.
INTERMEDIATE APPELLATE COURT AND J. ANTONIO ARANETA, respondents.

The Solicitor General for petitioners.

Jimenez, Leynes & Associates for private respondent.

NOCON, J.:
Full Text Cases in Natural Resources
14
For review before Us is the decision of the Court of Appeals in the land registration case entitled J.
Antonio Araneta v. The Director of Lands and Director of Forest Development, AC-G.R. CV. No.
00636,1 affirming the lower court's approval of the application for registration of a parcel of land in
favor of applicant therein, J. Antonio Araneta.

Evidence show that the land involved is actually an island known as Tambac Island in Lingayen
Gulf. Situated in the Municipality of Bani, Pangasinan, the area consists of 187,288 square meters,
more or less. The initial application for registration was filed for Pacific Farms, Inc. under the
provisions of the Land Registration Act, Act No. 496, as amended.

The Republic of the Philippines, thru the Director of Lands opposed the application alleging that the
applicant, Pacific Farms, Inc. does not possess a fee simple title to the land nor did its predecessors
possess the land for at least thirty (30) years immediately preceding the filing of application. The
opposition likewise specifically alleged that the applicant is a private corporation disqualified under
the (1973) new Philippine Constitution from acquiring alienable lands of the public domain citing
Section 11, Article 14.2

The Director of Forest Development also entered its opposition alleging that the land is within the
unclassified public land and, hence, inalienable. Other private parties also filed their oppositions,
but were subsequently withdrawn.

In an amended application, Pacific Farms, Inc. filed a manifestation-motion to change the applicant
from Pacific Farms, Inc. to J. Antonio Araneta. Despite the supposed amendment, there was no
republication.

Evidence presented by the applicant include the testimony of Placido Orlando, fishery guard of
Pacific Farms, Inc., who said he has known the disputed land since he attained the age of reason
for some forty (40) years now; that when he first came to know the property it was then owned by
and in the possession of Paulino Castelo, Juan Ambrosio and Julio Castelo, and later on the whole
island was bought by Atty. Vicente Castelo who in turn sold it to J. Antonio Araneta.

Deposition by oral examination of Araneta was also presented, together with documents of sale, tax
declarations and receipts, and survey of property. Applicant, however, failed to present the tracing
cloth plan and instead submitted to the court certified copies thereof.

While this case is pending here in Court, respondent filed an Omnibus Motion for Substitution of
private respondent.3 Apparently, Antonio Araneta had assigned his rights to and interest in Tambac
Island to Amancio R. Garcia4 who in turn assigned his rights and interest in the same property to
Johnny A. Khonghun whose nationality was not alleged in the pleadings.

On October 4, 1979, the trial court rendered a decision adjudicating the subject property to J.
Antonio Araneta. On appeal to the then Intermediate Appellate Court, the decision of the lower court
was affirmed on December 12, 1985.

Petitioners raised the following errors:

Full Text Cases in Natural Resources


15
I. The lower court erred in adjudicating the lands subject of registration to applicant-appellee despite
his failure to present the original tracing cloth plan the submission of which is a statutory requirement
of mandatory character.

II. The lower court erred in not denying registration in favor of J. Antonio Araneta since the
amendment of the application was simply an attempt to avoid the application of the constitutional
provision disqualifying a private corporation — the Pacific Farms, Inc. in this case — from acquiring
lands of public domain.

III. The lower court erred in not declaring the land known as the "Tambac Island" not subject of
registration it being an island formed on the seas.

IV. The lower court erred in adjudicating the land to the applicant under the provisions of Presidential
Decree No. 1529, otherwise known as the Property Registration Decree, despite absence of any
specific invocation of this law in the original and amended application.

V. The lower court erred in not granting the government's motion for reconsideration at least to
enable it to present proof of the status of the land as within the unclassified public forest, and hence
beyond the court's jurisdiction to adjudicate as private property.

VI. The lower court erred in not declaring that the applicant has failed to overthrow the presumption
that the land is a portion of the public domain belonging to the Republic of the Philippines.

From the foregoing it appears that the more important issues are: 1) whether the presentation of the
tracing cloth plan is necessary; and 2) whether the land known as "Tambac Island" can be subject
to registration.

By mere consideration of the first assignment of error, We can right away glean the merit of the
petition.

Respondent claims that the tracing cloth plan is with the files of the Land Registration Commission,
and the only evidence that can be presented to that fact is the request for the issuance of a certified
copy thereof and the certified copy issued pursuant to the request.5 Respondent further argues that
failure of the petitioners to object to the presentation of the certified copy of the tracing cloth plan
was the basis of the trial court's denial of petitioner's motion for reconsideration.

In a very recent decision of this Court, entitled The Director of Lands v. The Honorable Intermediate
Appellate Court and Lino Anit,6 We have ruled that the submission of the tracing cloth plan is a
mandatory requirement for registration. Reiterating Our ruling in Director of Lands v. Reyes,7 We
asserted that failure to submit in evidence the original tracing cloth plan is fatal it being a statutory
requirement of mandatory character.

It is of no import that petitioner failed to object to the presentation of the certified copy of the said
plan. What is required is the original tracing cloth plan of the land applied for and objection to such
requirement cannot be waived either expressly or impliedly.8 This case is no different from the case
of Director of Lands v. Reyes, supra wherein We said that if the original tracing cloth plan was

Full Text Cases in Natural Resources


16
indeed with the Land Registration Commission, there is no reason why the applicant cannot easily
retrieve the same and submit it in evidence, it being an essential requirement for registration.

As to the second assignment of error, We are inclined to agree with petitioners that the amendment
of the application from the name of Pacific Farms Inc., as applicant, to the name of J. Antonio
Araneta Inc., was a mere attempt to evade disqualification. Our Constitution, whether the 19739 or
1987, 10 prohibits private corporations or associations from holding alienable lands of the public
domain except by lease. Apparently realizing such prohibition, respondent amended its application
to conform with the mandates of the law.

However, We cannot go along with petitioners' position that the absence of republication of an
amended application for registration is a jurisdictional flaw. We should distinguish. Amendments to
the application may be due to change in parties or substantial change in the boundaries or increase
in the area of the land applied for.

In the former case, neither the Land Registration Act, as amended, nor Presidential Decree No.
1529, otherwise known as the Property Registration Decree, requires republication and registration
may be allowed by the court at any stage of the proceeding upon just and reasonable terms. 11 On
the other hand, republication is required if the amendment is due to substantial change in the
boundaries or increase in the area of the land applied for.

As to the fourth assignment of error. We do not see any relevant dispute in the lower court's
application of Presidential Decree No. 1529, instead of Act No. 496, in adjudicating the land to the
then applicant, assuming that the land involved is registrable. Both laws are existing and can stand
together. P.D. 1529 was enacted to codify the various laws relative to registration of property, in
order to facilitate effective implementation of said laws.12

The third, fifth and sixth assignment of errors are likewise meritorious and shall be discussed
forthwith together.

Respondent asserts that contrary to the allegation of petitioners, the reports of the District Land
Officer of Dagupan City, Land Inspector Perfecto Daroy and Supervising Land Examiner Teodoro
P. Nieva show that the subject property is an unclassified public land, not forest land. This claim is
rather misleading. The report of Supervising Land Examiner Nieva specifically states that the "land
is within the unclassified forest land" under the administrative jurisdiction of the then Bureau of
Forest Development.13 This was based on the reports of Land Inspector Daroy and District Land
Officer Feliciano Liggayu.

Lands of the public domain are classified under three main categories, namely: Mineral, Forest and
Disposable or Alienable Lands.14 Under the Commonwealth Constitution, only agricultural lands
were allowed to be alienated. Their disposition was provided for under Commonwealth Act No. 141
(Secs. 6-7), which states that it is only the President, upon the recommendation of the proper
department head, who has the authority to classify the lands of the public domain into alienable or
disposable, timber and mineral lands. Mineral and Timber or forest lands are not subject to private
ownership unless they are first reclassified as agricultural lands and so released for alienation. 15In
the absence of such classification, the land remains as unclassified land until released therefrom
and rendered open to disposition. Courts have no authority to do so. 16

Full Text Cases in Natural Resources


17
This is in consonance with the Regalian doctrine that all lands of the public domain belong to the
State, and that the State is the source of any asserted right to ownership in land and charged with
the conservation of such patrimony. Under the Regalian Doctrine, all lands not otherwise appearing
to be clearly within private ownership are presumed to belong to the State. Hence, a positive act of
the government is needed to declassify a forest land into alienable or disposable land for agricultural
or other purposes. 17

The burden of proof in overcoming the presumption of state ownership of the lands of the public
domain is on the person applying for registration that the land subject of the application is alienable
or disposable.18

Unless the applicant succeeds in showing by convincing evidence that the property involved was
acquired by him or his ancestors either by composition title from the Spanish Government or by
possessory information title, or any other means for the proper acquisition of public lands, the
property must be held to be part of the public domain. The applicant must present evidence and
persuasive proof to substantiate his claim. 19

In this particular case, respondent presented proof that as early as 1921, the subject property has
been declared for tax purposes with receipts attached, in the names of respondent's predecessors-
in-interest. Nevertheless, in that span of time there had been no attempt to register the same either
under Act 496 or under the Spanish Mortgage Law. It is also rather intriguing that Vicente Castelo
who acquired almost 90% of the property from Alejo Ambrosia, et al. on June 18, 1958 and from
Julio Castelo on June 19, 1958 immediately sold the same to applicant J. Antonio Araneta on 3 July
1958.

According to the report of Land Investigator Daroy, the land was declared for taxation purposes in
the name of Vicente Castelo only in 1958 and the purported old tax declarations are not on file with
the Provincial Assessor's Office.

In any case tax declarations and receipts are not conclusive evidence of ownership or of the right
to possess land when not supported by evidence. 20 The fact that the disputed property may have
been declared for taxation purposes in the names of the applicants or of their predecessors-in-
interest way back in 1921 does not necessarily prove ownership. They are merely indicia of a claim
of ownership.21

Respondent's contention that the BFD, LC Map No. 681, certified on August 8, 1927 which was the
basis of the report and recommendation of the Land Examiner, is too antiquated; that it cannot be
conclusively relied upon and was not even presented in evidence, is not well taken. As We have
said in the case of Director of Lands v. CA:22

And the fact that BF Map LC No. 673 dated March 1, 1927 showing subject property to be within
unclassified region was not presented in evidence will not operate against the State considering the
stipulation between the parties and under the well-settled rule that the State cannot be estopped by
the omission, mistake or error of its officials or agents, if omission there was, in fact.

Respondent even admitted that Tambac Island is still an unclassified public land as of 1927 and
remains to be unclassified.

Full Text Cases in Natural Resources


18
Since the subject property is still unclassified, whatever possession
the applicant may have had and however long, cannot ripen into private ownership. 23 The
conversion of subject property does not automatically render the property as alienable and
disposable.

In effect what the courts a quo have done is to release the subject property from the unclassified
category, which is beyond their competence and jurisdiction. We reiterate that the classification of
public lands is an exclusive prerogative of the Executive Department of the Government and not of
the Courts. In the absence of such classification, the land remains unclassified until released
therefrom and rendered open to disposition. 24

In fairness to respondent, the petitioners should seriously consider the matter of the reclassification
of the land in question. The attempt of people to have disposable lands they have been tilling for
generations titled in their name should not only be viewed with understanding attitude, but as a
matter of policy encouraged. 25

WHEREFORE, the petition is hereby GRANTED and the decisions of the courts a quo are
REVERSED.

SO ORDERED.

5. United Paracale vs Dela Rosa 221 SCRA 108

G.R. No. 63786-87. April 7, 1993.

UNITED PARACALE MINING COMPANY, INC., AND COCO GROVE, INC., petitioners,
vs.
HON. JOSELITO DELA ROSA, in his capacity as the former Judge of the Court of First Instance of
Camarines Norte, Branch 2, et al., respondents.

G.R. No. 70423. April 7, 1993.

ZAMBALES CHROMITE MINING COMPANY, INC., petitioner, vs. HON. ALFREDO L. BENIPAYO,
as Judge, Regional Trial Court of Manila, and PHILZEA MINING & DEVELOPMENT
CORPORATION, respondents.

G.R. No. 73931. April 7, 1993.

JOSEPH V. LOPEZ and MIGUEL C. ANDRADE, petitioners,


vs.
THE INTERMEDIATE APPELLATE COURT MARSMAN & COMPANY, INC. and UNITED
PARACALE MINING COMPANY, INC., respondents.

Sycip, Salazar, Hernandez and Gatmaitan for United Paracale and Coco Grove, Inc.

Pedro A. Venida for respondents in G.R. 63786-87 and petitioners in G.R. 73931.

Full Text Cases in Natural Resources


19
J.V. Natividad & Associates for Zambales Chromite.

SYLLABUS

1. STATUTORY CONSTRUCTION; INTERPRETATION OF A STATUTE; PROPER ONLY WHEN


THERE IS DOUBT OR AMBIGUITY IN ITS LANGUAGE; CASE AT BAR. — The view of the
petitioner that by virtue of the registration of the mining claims under the Philippine Bill of 1902 and
Act No. 624, the mining claims became private property and thereby brought outside the control
and supervision of the Director of Mines is without legal basis. The abovecited law does not
distinguish between private property and lands of the public domain. The provision of law involved
is clear and is not susceptible to interpretation. A condition sine qua non before the court may
construe or interpret a statute is that there be doubt or ambiguity in its language. Section 7 of P.D.
1281 quoted above defining the original and exclusive jurisdiction of the Director of Mines is clear.
Time and again, it has been repeatedly declared by this Court that where the law speaks in clear
and categorical language, there is no room for interpretation. There is only room for application.
[Cebu Portland Cement Company vs. Municipality of Naga, Cebu, 35 SCRA 708 (1968)] Where the
law is clear and unambiguous, it must be taken to mean exactly what it says and the court has no
choice but to see to it that its mandate is obeyed. [Chartered Bank Employees Association vs. Ople,
138 SCRA 273 (1985)].

2. REMEDIAL LAW; CIVIL PROCEDURE; JUDICIAL RELIEF; NOT A VESTED RIGHT; A MERE
STATUTORY PRIVILEGE, NOT A PROPERTY RIGHT. — There can be no vested right in a judicial
relief for this is a mere statutory privilege and not a property right. The distinction between statutory
privileges and vested rights must be borne in mind for persons have no vested rights in statutory
privileges. The state may change or take away rights which were created by the law of the state,
although it may not take away property which was vested by virtue of such rights.(16A Am. Jur. 2d,
pp. 652-653) Besides, the right to judicial relief is not a right which may constitute vested right
because to be vested, a right must have become a title, legal or equitable, to the present or future
enjoyment of property, or to the present or future enforcement of a demand or legal exemption from
a demand made by another. (National Carloading Corporation v. Phoenix-E1 Paso Express, Inc.,
cited in 16A Am, Jur. 2d, p. 651) Definitely, the judicial relief herein referred to by the petitioner does
not fall under any of these.

3. CIVIL LAW; PRESIDENTIAL DECREE 1214; A VALID EXERCISE OF THE SOVEREIGN


POWER OF THE STATE AS OWNER OF LAND OF PUBLIC DOMAIN; SUSTAINED IN CASE AT
BAR. — The heart of these twin petitions is the question of constitutionality of P.D. 1214. Unless
P.D. 1214 is successfully assailed, the petitioners will be but mere applicants for the lease of the
mining claims involved and would thus have no causes of action against private respondents. This
question has been resolved by this Court in Santa Rosa Mining Company, Inc. vs. Leido, Jr. [156
SCRA 1 (1987), which ruling was reiterated in Zambales Chromite Mining Company, Inc. vs. Leido,
Jr., 176 SCRA 602 (1989)] thus: "(W)e hold that Presidential Decree No. 1214 is not
unconstitutional.' It is a valid exercise of the sovereign power of the State, as owner, over lands of
the public domain, of which petitioner's mining claims still form a part, and over the patrimony of the
nation, of which mineral deposits are a valuable asset. It may be underscored, in this connection,
that the Decree does not cover all mining claims located under the Phil. Bill of 1902, but only those
claims over which their locators had failed to obtain a patent. And even then, such locators may still
avail of the renewable twenty-five year (25) lease prescribed by Pres. Decree No. 463, the Mineral

Full Text Cases in Natural Resources


20
Development Resources Decree of 1974. Mere location does not mean absolute ownership over
the affected land or the mining claim. It merely segregates the located land or area from the public
domain by barring other would-be locators from locating the same and appropriating for themselves
the minerals found therein. To rule otherwise would imply that location is all that is needed to acquire
and maintain rights over a located mining claim. This, we cannot approve or sanction because it is
contrary to the intention of the lawmaker that the locator should faithfully and consistently comply
with the requirements for annual work and improvements in the located mining claim. This, we
cannot approve or sanction because it is contrary to the intention of the lawmaker that the locator
should faithfully and consistently comply with the requirements for annual work and improvements
in the located mining claim. Presidential Decree No. 1214 is in accord with Sec. 8, Art. XIV of the
1973 Constitution. The same Constitutional mandate is found in Sec. 2, Art. XII of the 1987
Constitution.

DECISION

CAMPOS, JR., J p:

The cases herein were consolidated upon the representations of petitioners that they involve the
same issues or questions of law or at the very least, revolve around the same set of facts. A perusal
of the records, however, reveals the contrary. Only two petitions are properly consolidated. Thus, it
behooves Us to discuss the cases separately.

In blatant violation of Section 2, Rule 45 of the Rules of Court which in part, provides:

"Sec. 2. Contents of the petition — The petition shall contain a concise statement of the matters
involved, the assignment of errors made in the court below, and the reasons relied on for the
allowance of the petition, . . . (Emphasis Ours.).

"Only questions of law may be raised in the petition and must be distinctly set forth . . ."

this petition devotes nine (9) pages under the subtitle "Summary statement of the matters involved"
to a discussion of matters off tangent from the real issues in the case. Definitely, the question of
whether or not the Court of Appeals erred in ruling that the Regional Trial Court did not commit
grave abuse of discretion in issuing an order suspending hearing pending the resolution of their
motion to dismiss, does not involve the Philippine Bill of 1902, Executive Order No. 141, much less
P.D. 1214. The counsel for petitioners even discussed pending cases in this Court (G.R. No. 63786
- 87 and 69203) which have completely nothing to do with the instant petition except for the fact that
the parties therein are being represented by the same counsel as in this petition. In several
pleadings subsequent to their petition, petitioners insisted that the proceedings in the court below
must be restrained until this Court resolves the pending cases abovecited. For this reason this case
was consolidated thereto.

A summary of the real matters involved in this petition is found in the respondent Court's decision,
to wit:

"This is a petition for certiorari and prohibition to enjoin the Regional Trial Court, Branch XL, in
Camarines Norte from issuing a writ of preliminary injunction in Civil Case No. 5148 and to disqualify

Full Text Cases in Natural Resources


21
the respondent judge from acting in that case. The case was brought by the respondents Marsman
and Co., Inc. and United Paracale Mining, Inc., to enjoin the petitioners, Joseph V. Lopez and Miguel
Andrade, from entering and conducting mining operations within the "McDonald" and "San Antonio"
Tunnels in Paracale, Camarines Norte, in which the private respondents have mining claims
("Tulisan," "Santa Marta," "California," and "Rocky Mountain Fraction"). On December 11, 1984 the
RTC issued a restraining order against the petitioners.

On December 12 the petitioners filed their answer alleging that, in accordance with PD 1214, the
private respondents had forfeited their right to the mining claims. They likewise argued that in view
of PD 605, the RTC had no jurisdiction to entertain the case. On the same day the petitioners moved
for the disqualification of the respondent judge of the RTC, claiming (sic) that in issuing the
restraining order of December 11, 1984, he showed his "bias, prejudice and personal hatred of and
hostility to the [petitioners'] counsel [Atty. Pedro A. Venida]."

On December 24, 1984 the petitioners filed a motion for a preliminary hearing on their defense that
the RTC lacked jurisdiction under PD 605 to issue a temporary restraining order or injunction in
cases involving or growing out of the action of administrative officials on the applications for permits
for the disposition, exploitation, utilization, or exploration or development of the natural resources.
Accordingly the RTC, in its order of September 5, 1985, suspended the hearing of the case until the
resolution of the petitioners' motion to dismiss. It is at this point that the present petition was filed."
1

The respondent Court denied this petition on grounds that: (1) the questions being raised are not
proper in a petition for certiorari under Rule 65 but rather defenses which should be raised in the
action itself; (2) the question of jurisdiction which has yet to be resolved by the trial court pending
resolution of the motion to dismiss is prematurely raised; and (3) there was no basis for determining
whether or not the judge must be disqualified. 2

The review of this decision is what is on appeal before.

We refuse to be persuaded by the petitioners that the RTC must be enjoined from exercising its
jurisdiction in settling the case presented before it for the reason that the constitutionality of the law
involved in the said case is being questioned before this Court. This case should have been
disposed of independently of the other petitions herein.

The respondent Court of Appeals committed no reversible error. Neither did it commit grave abuse
of discretion as what petitioners want this Court to believe. The petitioners fail to point out any
assigned error which the respondent Court had supposedly committed but simply narrate the action
taken by it. Much less have they stated the reasons relied upon for the allowance of the instant
petition. For being insufficient in substance and in form, the instant petition lacks merit and must be
dismissed.

G.R. No. 70423

This is a petition involving the question of jurisdiction of regular courts in cases which had been
placed under the original and exclusive jurisdiction of the Bureau of Mines under P.D. 1281.

Full Text Cases in Natural Resources


22
This petition seeks to reverse the order of then Judge, now Associate Justice of the Court of
Appeals, Hon. Alfredo L. Benipayo, dismissing the complaint filed by petitioner herein on the ground
of lack of jurisdiction, citing Section 7 of P.D. 1281 and the doctrine enunciated in Twin Peaks Mining
Association, et al. vs. Navarro, 3 that an action for the enforcement of mining contracts, in this case
cancellation of a mining contract, is outside the competence of regular courts in view of the law
cited. 4

The complaint filed with the then CFI of Manila, Branch XVI, was one for the rescission of its mining
contract with herein private respondent on grounds of violations of the terms and conditions thereof,
with prayer for the issuance of a preliminary injunction and/or temporary restraining order. The trial
court, however, upon motion of the defendant therein, dismissed the case.

Petitioner wants Us to construe Section 7 of P.D. 1281 as applicable only to mineral lands forming
part of the public domain and not to mining claims located and registered under Philippine Bill of
1902 and Act No. 624 as is its case.

Section 7 of P.D. 1281 reads as follows:

Sec. 7. In addition to its regulatory and adjudicative functions over companies, partnerships or
persons engaged in mining exploration, development and exploitation, the Bureau of Mines shall
have original and exclusive jurisdiction to hear and decide cases involving:

(a) a mining property subject of different agreements entered into by the claim holder thereof with
several mining operators;

(b) complaints from claimowners that the mining property subject of an operating agreement has
not been placed into actual operations within the period stipulated therein; and

(c) cancellation and/or reinforcement of mining contracts due to the refusal of the
claimowner/operator to aside by the terms and conditions thereof.

All actions and decisions of the Director of Mines on the above cases are subject to review, motu
proprio or upon appeal by any person aggrieved thereby, by the Secretary of Natural Resources
whose decision shall be final and executory after the lapse of thirty (30) days from receipt by the
aggrieved party of said decision, unless appealed to the President in accordance with the applicable
provisions of Presidential Decree No. 309 and Letter of Instructions Nos. 119 and 135.

The view of the petitioner that by virtue of the registration of the mining claims under the Philippine
Bill of 1902 and Act No. 624, the mining claims became private property and thereby brought outside
the control and supervision of the Director of Mines is without legal basis. The abovecited law does
not distinguish between private property and lands of the public domain. The provision of law
involved is clear and is not susceptible to interpretation. A condition sine qua non before the court
may construe or interpret a statute is that there be doubt or ambiguity in its language. 5 Section 7
of P.D. 1281 quoted above defining the original and exclusive jurisdiction of the Director of Mines is
clear. Time and again, it has been repeatedly declared by this Court that where the law speaks in
clear and categorical language, there is no room for interpretation. There is only room for

Full Text Cases in Natural Resources


23
application. 6 Where the law is clear and unambiguous, it must be taken to mean exactly what it
says and the court has no choice but to see to it that its mandate is obeyed. 7

This Court in Benguet Corporation vs. Leviste, 8 made these pronouncements:

"We grant the petition. Presidential Decree No. 1281 which took effect on January 16, 1978 vests
the Bureau of Mines with jurisdictional supervision and control over all holders of mining claims or
applicants for and/or grantees of mining licenses, permits, leases and/or operators thereof, including
mining service contracts and service contractors insofar as their mining activities are concerned. To
effectively discharge its task as the Government's arm in the administration and disposition of
mineral resources, Section 7 of P.D. 1281 confers upon the Bureau quasi-judicial powers as follows:

xxx xxx xxx

Analyzing the objective of P.D. 1281, particularly said Section 7 thereof, the Court in Twin Peaks
Mining Association, the case relied upon by petitioner, noted that the trend is to make the
adjudication of mining cases a purely administrative matter. This observation was reiterated in the
more recent case of Atlas Consolidated Mining & Development Corporation vs. Court of Appeals."

The petitioner further argues that to hold that P.D. 1281 retroactively applies to its mining claims
which according to it is private property would constitute impairment of vested rights since by shifting
the forum of the petitioner's case from the courts to the Bureau of Mines, as urged by private
respondent, the substantive rights to full protection of its property rights shall be greatly impaired
and prejudiced. The judicial relief available for the redress of private property rights violated, now
being enjoyed by petitioner shall be lost altogether.

This argument does not merit Our approval. There can be no vested right in a judicial relief for this
is a mere statutory privilege and not a property right. The distinction between statutory privileges
and vested rights must be borne in mind for persons have no vested rights in statutory privileges.
The state may change or take away rights which were created by the law of the state, although it
may not take away property which was vested by virtue of such rights. 9 Besides, the right to judicial
relief is not a right which may constitute vested right because to be vested, a right must have become
a title, legal or equitable, to the present or future enjoyment of property, or to the present or future
enforcement of a demand or legal exemption from a demand made by another. 10 Definitely, the
judicial relief herein referred to by the petitioner does not fall under any of these.

The case at bar falls within the original and exclusive jurisdiction of the Bureau of Mines, hence, the
trial court did not err in dismissing the petitioner's complaint on the ground of lack of jurisdiction.

G.R. Nos. 63786-87

In these petitions filed by petitioners United Paracale Mining Company, Inc. and Coco Grove, Inc.,
petitioners seek to set aside the Order of dismissal of the case they filed with the trial court for the
ejectment of their respective defendants from the mining claims which were allegedly privately
owned by them having been located and perfected under the provisions of the Philippine Bill of 1902
and Act No. 624.

Full Text Cases in Natural Resources


24
The heart of these twin petitions is the question of constitutionality of P.D. 1214. Unless P.D. 1214
is successfully assailed, the petitioners will be but mere applicants for the lease of the mining claims
involved and would thus have no causes of action against private respondents.

This question has been resolved by this Court in Santa Rosa Mining Company, Inc. vs. Leido, Jr.
11 thus:

"(W)e hold that Presidential Decree No. 1214 is not unconstitutional. ** It is a valid exercise of the
sovereign power of the State, as owner, over lands of the public domain, of which petitioner's mining
claims still form a part, and over the patrimony of the nation, of which mineral deposits are a valuable
asset. It may be underscored, in this connection, that the Decree does not cover all mining claims
located under the Phil. Bill of 1902, but only those claims over which their locators had failed to
obtain a patent. And even then, such locators may still avail of the renewable twenty-five year (25)
lease prescribed by Pres. Decree No. 463, the Mineral Development Resources Decree of 1974.

Mere location does not mean absolute ownership over the affected land or the mining claim. It
merely segregates the located land or area from the public domain by barring other would-be
locators from locating the same and appropriating for themselves the minerals found therein. To
rule otherwise would imply that location is all that is needed to acquire and maintain rights over a
located mining claim. This, we cannot approve or sanction because it is contrary to the intention of
the lawmaker that the locator should faithfully and consistently comply with the requirements for
annual work and improvements in the located mining claim.

Presidential Decree No. 1214 is in accord with Sec. 8, Art. XIV of the 1973 Constitution which states:

'All lands of the public domain, waters, minerals, coal, petroleum, and other mineral oils, all forces
of potential energy, fisheries, wildlife, and other natural resources of the Philippines belong to the
State. With the exception of agricultural, industrial or commercial, residential and resettlement lands
of the public domain, natural resources shall not be alienated, and no license, concession, or lease
for the exploration, development, and exploitation, or utilization of any of the natural resources shall
be granted for a period exceeding twenty-five years, renewable for not more than twenty-five years,
except as to water rights for irrigation, water supply, fisheries, or industrial uses other than
development of water power, in which cases, beneficial use may be the measure and the limit of
the grant.'

The same Constitutional mandate is found in Sec. 2, Art. XII of the 1987 Constitution, which
declares:

'All lands of the public domain, waters, minerals, coal, petroleum, and other mineral oils, all forces
of potential energy, fisheries, forests or timber, wildlife, flora and fauna, and other natural resources
are owned by the State. With the exception of agricultural lands, all other natural resources shall
not be alienated. The exploration, development, and utilization of natural resources shall be under
the full control and supervision of the State . . .'"

Notwithstanding Our ruling , in favor of the constitutionality of P.D. 1214, petitioners contend that
having filed mining lease applications on the mining claims they have previously located and
registered under then existing laws, pursuant to the requirements of this Presidential Decree, and

Full Text Cases in Natural Resources


25
despite the waiver of their rights to the issuance of mining patents therefor (emphasis theirs), they
cannot be placed in equal footing with those who forfeit all rights by virtue of non-filing of an
application within the prescribed period such that they (petitioners) have no causes of action against
private respondents.

We are not persuaded by this contention.

Although We may agree that those who filed their mining lease applications have better rights than
those who forfeited all their right by not filing at all, this, however, does not amount to any vested
right which could be the basis for their cause of action against herein private respondents. What is
precisely waived is their right to the issuance of a mining patent upon application. This in effect
grants the government the power, in the exercise of its sound discretion, to award the patent to the
applicant most qualified to explore, develop and exploit the mineral resources of the country in line
with the objectives of P.D. 463, and not necessarily to the original locator of the mining claim. To
sustain their contention that they can question the award of mining patents to applicants other than
them would put to naught the objectives of P.D. 1214 as enunciated in its WHEREAS clauses.

We agree with the trial court that with the waiver of their right to the issuance of a mining patent
upon their application for a mining lease, their status is reduced to a mere applicant, their only
advantage over the others is the fact that they have already conducted explorations at the site and
this exploration may he ongoing. But still, this credential, so to speak, is not intended to tie the hands
of the government so as to prevent it from awarding the mining patent to some other applicants,
which in its belief may he more qualified than them.

WHEREFORE, the petition in G.R. No. 73931 is hereby DISMISSED for lack of merit; the Order of
dismissal assailed in G.R. No. 70423 is AFFIRMED and this petition is hereby likewise DISMISSED;
the Order of dismissal assailed in G.R. Nos. 63786-87 is AFFIRMED and these petitions are hereby
DISMISSED. No pronouncements as to costs.

SO ORDERED.

6. Republic vs Register of Deeds of Quezon 224 SCRA 537

G.R. No. 73974 May 31, 1995

REPUBLIC OF THE PHILIPPINES (Represented by the Director of Lands), petitioner,


vs.
THE REGISTER OF DEEDS OF QUEZON, MANUEL G. ATIENZA, DEVELOPMENT BANK OF
THE PHILIPPINES (Lucena Branch) and INTERMEDIATE APPELLATE COURT, respondents.

ROMERO, J.:

This petition for review on certiorari seeks to nullify and set aside the decision 1 of the then
Intermediate Appellate Court reversing the decision of the former Court of First Instance of Quezon,
Branch II at Lucena City 2 which annulled Original Certificate of Title (OCT) No. P-13840 and Free

Full Text Cases in Natural Resources


26
Patent (FP) No. 324198 issued to Manuel Atienza for a 17-hectare piece of land which turned out
to be within the forest zone in Pagbilao, Quezon.

On April 18, 1967, Atienza was awarded FP No. 324198 over a parcel of land located in Ila,
Malicboy, Pagbilao, Quezon, with an area of 172,028 square meters. By virtue of such award, he
was issued on May 5, 1967, OCT No. P-13840.

Sometime in 1968, an investigation was conducted by the Bureau of Lands in connection with
alleged land grabbing activities in Pagbilao. It appeared that some of the free patents, including that
of Atienza's, were fraudulently acquired. Thus, on March 19, 1970, a criminal complaint for
falsification of public documents was filed in the then Court of First Instance of Quezon, Branch II,
against Atienza and four other persons for allegedly falsifying their applications for free patent, the
survey plans, and other documents pertinent to said applications.

In its decision dated October 4, 1972, the court acquitted the accused of the crime charged but,
finding that the land covered by the application for free patent of private respondent was within the
forest zone, declared as null and void OCT No. P-13840 in Atienza's name and ordered the Register
of Deeds of Quezon to cancel the same.

Meanwhile, before the promulgation of said decision, or on May 10, 1972, then Acting Solicitor
General Conrado T. Limcaoco filed for the petitioner a complaint against Atienza, the Register of
Deeds of Quezon, and the Rural Bank of Sariaya, which was later dropped as defendant and, in an
amended complaint, substituted by the Development Bank of the Philippines as actual mortgagee
of the subject parcel of land. Docketed as Civil Case No. 7555, the complaint prayed for the
declaration of nullity of FP No. 324198 and OCT No. P-13840.

In his answer, Atienza claimed that the land in question was no longer within the unclassified public
forest land because by the approval of his application for free patent by the Bureau of Lands, the
land "was already alienable and disposable public agricultural land." Since the subject land was a
very small portion of Lot 5139 of the Pagbilao Cadastre, an area which had been declared
disposable public land by the cadastral court on March 9, 1932 in Cadastral Case No. 76 entitled
"El Govierno Filipino de las Islas Filipinas contra Jose Abastillas, et al., G.L.R.O. cadastral Record
No. 1124," he also averred that the Director of Lands had given due course to free and homestead
patent applications of claimants to Lot 5139. He further alleged that through a certain Sergio Castillo,
he had been in possession of the land since the Japanese occupation, cultivating it and introducing
improvements thereon. The DBP, after due and proper investigation and inspection of his title, even
granted him a loan with the subject property as collateral. Finally, he stated that his acquittal in the
criminal case proved that he committed no fraud in his application for free patent.

On July 27, 1981, the lower court rendered a decision with the categorical finding based on "solid
evidence" that "the land in question was found definitely within the forest zone denominated as
Project 21-A."

The dispositive portion thereof reads as follows:

WHEREFORE, in view of the foregoing, (J)udgment is hereby rendered:

Full Text Cases in Natural Resources


27
(a) Declaring as null and void Original Certificate of Title No. P-13840 in the name of defendant
Manuel G. Atienza, as well as Free Patent No. V-324198;

(b) Ordering defendant Manuel G. Atienza to pay the Development Bank of the Philippines, Lucena
City Branch, the sum of P15,053.97, and all interests due thereon; and

(c) Ordering defendant Manuel G. Atienza to pay the costs of this suit.

SO ORDERED.

On appeal, Atienza maintained that the land in question was not within the unclassified public forest
land and therefore alienable land of the public domain. The then Intermediate Appellate Court relied
only on the arguments he raised since petitioner had not filed any brief, and arrived at the conclusion
that "(t)he litigated land is part of public land alienable and disposable for homestead and [F]ree
Patent." On December 27, 1985, the appellate court set aside the lower court's decision, declared
as valid and subsisting Atienza's OCT, and dismissed the cross-claim of the DBP.

After receiving a copy of said decision, Assistant Solicitor General Oswaldo D. Agcaoili informed
the Director of Lands of the adverse decision of the appellate court, which noted that no appellee's
brief had been filed in said court. Agcaoili also stated that the Office of the Solicitor General (OSG)
had not been furnished with the appellant's brief; that the Bureau of Lands received notice of hearing
of the record on appeal filed by the appellant but the OSG had not been informed of the "action
taken thereon;" that since the Bureau of Lands had been furnished directly with relevant pleadings
and orders, the same office should "take immediate appropriate action on the decision;" and that it
may file a motion for reconsideration within fifteen (15) days from January 6, 1986, the date of
receipt by the OSG of the copy of the decision sought to be appealed.

On January 28, 1986, petitioner filed a motion for extension of time to file motion for reconsideration
which was denied in a resolution dated February 12, 1986. Petitioner's motion for reconsideration
of said resolution was likewise denied.

The instant petition for review on certiorari raises the following arguments: (a) petitioner was denied
due process and fair play when Atienza did not furnish it with a copy of his appellant's brief before
the then Intermediate Appellate Court thereby depriving it of the opportunity to rebut his assertions
which later became the sole basis of the assailed decision of December 27, 1985; (b) the appellate
court erred in holding that the land in question is part of the alienable and disposable public land in
complete disregard of the trial court's finding that it forms part of the unclassified public forest zone;
and (c) the appellate court erred in declaring that the land in question could be alienated and
disposed of in favor of Atienza.

We find for the petitioner.

Appeal is an essential part of our judicial system. As such, courts should proceed with caution so
as not to deprive a party of the right to appeal, particularly if the appeal is meritorious. 3 Respect for
the appellant's right, however, carries with it the corresponding respect for the appellee's similar
rights to fair play and justice. Thus, appeal being a purely statutory right, an appealing party must
strictly comply with the requisites laid down in the Rules of Court. 4

Full Text Cases in Natural Resources


28
Of paramount importance is the duty of an appellant to serve a copy of his brief upon the appellee
with proof of service thereof. 5 This procedural requirement is consonant with Section 2 of Rule 13,
which mandates that all pleadings and papers "shall be filed with the court, and served upon the
parties affected thereby." The importance of serving copies of the brief upon the adverse party is
underscored in Mozar v. Court of Appeals, 6 where the Court held that the appellees "should have
been given an opportunity to file their appellee's brief in the Court of Appeals if only to emphasize
the necessity of due process."

In this case, however, the Court of Appeals, oblivious of the fact that this case involves public lands
requiring as it does the exercise of extraordinary caution lest said lands be dissipated and
erroneously alienated to undeserving or unqualified private individuals, decided the appeal without
hearing the government's side.

Atienza avers that he furnished Atty. Francisco Torres, a lawyer in the Bureau of Lands and
designated special attorney for the Office of the Solicitor General, with two copies of the appellant's
brief, thereby implying that it was not his fault that petitioner failed to file its appellee's brief.

Such an assertion betrays a lack of comprehension of the role of the Solicitor General as
government counsel or of the OSG as the government's "law office." 7 Only the Solicitor General,
as the lawyer of the government, can bring or defend actions on behalf of the Republic of the
Philippines and, therefore, actions filed in the name of the Republic, if not initiated by the Solicitor
General, will be summarily dismissed. 8 Specifically, he is empowered to represent the Government
in all land registration and related proceedings, 9 such as, an action for cancellation of title and for
reversion of a homestead to the government. 10 Hence, he is entitled to be furnished with copies of
all court orders, notices and decisions. Consequently, service of decisions on the Solicitor General
is the proper basis for computing the reglementary period for filing appeals and for finality of
decisions. His representative, who may be a lawyer from the Bureau of Lands, has no legal authority
to decide whether or not an appeal should be made. 11

Service of the appellant's brief on Atty. Torres was no service at all upon the Solicitor General. It
may be argued that Atty. Torres could have transmitted one of the two copies of appellant's brief
upon the Solicitor General, but such omission does not excuse Atienza's failure to serve a copy of
his brief directly on the Solicitor General.

On the part of the appellate court, its decision based solely on, and even quoting verbatim from, the
appellant's brief was certainly arrived at in grave abuse of discretion. It denied appellee (petitioner
herein) of the opportunity to be heard and to rebut Atienza's allegations, in rank disregard of its right
to due process. Such violation of due process could have been rectified with the granting of
petitioner's motion for reconsideration by the appellate court, 12 but even the door to this recourse
was slammed by the appellate court with the denial of petitioner's motion for extension of time to
file motion for reconsideration in a resolution dated February 12, 1986, which ruling erroneously
applied the Habaluyas doctrine. 13

Such denial notwithstanding, petitioner filed its motion for reconsideration. Considering the clear
allegations thereunder, the appellate court would have done well, in the interest of justice, not to
blindly adhere to technical rules of procedure by dismissing outright said motion. As we declared
in Villareal v. Court of Appeals: 14

Full Text Cases in Natural Resources


29
. . . The requirements of due process are satisfied when the parties are afforded a fair and
reasonable opportunity to explain and air their side. The essence of due process is simply the
opportunity to be heard or as applied to administrative proceedings, an opportunity to explain one's
side or an opportunity to seek a reconsideration of the action or ruling taken. (Emphasis supplied)

In view of the foregoing and the long-standing procedural rule that this Court may review the findings
of facts of the Court of Appeals in the event that they may be contrary to those of the trial court, 15 in
order to attain substantial justice, the Court now reviews the facts of the case.

Under the Regalian Doctrine, all lands not otherwise clearly appearing to be privately-owned are
presumed to belong to the State. Forest lands, like mineral or timber lands which are public lands,
are not subject to private ownership unless they under the Constitution, become private properties.
In the absence of such classification, the land remains unclassified public land until released
therefrom and rendered open to disposition. 16

In our jurisdiction, the task of administering and disposing lands of the public domain belongs to the
Director of Lands, and ultimately, the Secretary of Agriculture and Natural Resources 17 (now the
Secretary of Environment and Natural Resources). 18 Classification of public lands is, thus, an
exclusive prerogative of the Executive Department through the Office of the President. 19 Courts
have no authority to do so. 20

Thus, in controversies involving the disposition of public agricultural lands, the burden of overcoming
the presumption of state ownership of lands of the public domain lies upon the private
claimant 21 who, in this case, is Atienza. The records show, however, that he failed to present clear,
positive and absolute evidence 22 to overcome said presumption and to support his claim.

Atienza's claim is rooted in the March 9, 1932 decision of the then Court of First Instance of Tayabas
in Cadastral Case No. 76, which was not given much weight by the court a quo, and for good
reasons.

Apart from his assertions before this Court, Atienza failed to present proof that he or his
predecessor-in-interest was one of the claimants who answered the petition filed by the then
Attorney-General in the said cadastral proceedings. The document reflecting said cadastral
decision, a xerox copy, indicated the claimants simply as "Jose Abastillas et al." In support of that
decision, Atienza presented a certification purportedly issued by someone from the Technical
Reference Section of the Surveys Division, apparently of the Bureau of Lands, stating that "Lot 5886
is a portion of Lot 5139 Pagbilao Cadastre," which evidence is, however, directly controverted by
the sketch plan showing that the land in controversy is actually outside the alienable and disposable
public lands, although part of Lot 5139.

The fact that Atienza acquired a title to the land is of no moment, notwithstanding the indefeasibility
of titles issued under the Torrens system. In Bornales v. Intermediate Appellate Court, 23 we ruled
that the indefeasibility of a certificate of title cannot be invoked by one who procured the same by
means of fraud. The "fraud" contemplated by the law (Sec. 32, P.D. 1529) is actual and extrinsic,
that is, "an intentional omission of fact required by law," 24 which in the case at bench consisted in
the failure of Atienza to state that the land sought to be registered still formed part of the unclassified
forest lands.

Full Text Cases in Natural Resources


30
WHEREFORE, the decision appealed from is hereby REVERSED and SET ASIDE and the decision
of the court a quo dated July 27, 1981, is REINSTATED.

SO ORDERED.

7. Ituralde vs Falcasantos GR. No. 128017 01/20/1999


AMON ITURALDE, petitioner, vs. ALFREDO FALCASANTOS, respondent.

DECISION
PARDO, J.:

The case is an appeal via certiorari from a decision of the Court of Appeals reversing that of the
Regional Trial Court, Branch 2, Basilan province, and dismissing petitioner's complaint for recovery
of possession and ownership of a parcel of land with the improvements existing thereon, situated
at Barangay Upper Baas, municipality of Lantawan, province of Basilan, with an area of 7.1248
hectares.
The facts may be related as follows:
On October 17, 1986, petitioner acquired by purchase from the heirs of Pedro Mana-ay a parcel of
land located at Baas, Lantawan, Basilan Province, with an area of 6.0000 hectares, more or less,
more particularly described as follows:

"A parcel of land, situated at Baas, Lantawan Basilan. Bounded on the North by property of
Alejandro Marso; on the East by property of Ramon Bacor; on the South by property of Atty. Ricardo
G. Mon and on the West by property of Librada Guerrero. Containing an area of 6.0000 hectares,
more or less."

However, on November 3, 1986, respondent applied with the Bureau of Lands in Isabela, Basilan
province, for the award to him of the same parcel of land under free patent. On November 17, 1986,
petitioner filed a protest to such application.
On February 7, 1989, the Regional Director of Lands rendered a decision giving respondent a period
of one hundred twenty (120) days to exercise the right to repurchase the land by reimbursing
petitioner of all expenses he incurred in the purchase of the property in question, and held in
abeyance respondent's application for free patent.
On October 11, 1989, the Regional Director issued an order declaring that respondent had waived
his right of repurchase, and rejected his application for free patent for lack of interest, and allowed
petitioner to file a public land application for the subject land.
On May 8, 1990, the Regional Director ordered respondent to vacate the land in question, but
respondent refused.
On July 24, 1990, petitioner filed with the Regional Trial Court, Basilan province, a complaint for
recovery of ownership and possession with preliminary injunction of the subject parcel of land.
In answer to the complaint, respondent alleged that the land occupied by him belonged to the
Republic of the Philippines, and that he had introduced improvements thereon such as coconut and
other fruit trees.
Full Text Cases in Natural Resources
31
After trial on the merits, on March 20, 1993, the trial court rendered decision declaring petitioner the
owner and possessor of the subject parcel of land with all the improvements existing thereon,
situated at Barangay Upper Baas, municipality of Lantawan, province of Basilan, with an area of
3.1248 hectares, and ordering respondent to vacate the land in question, to pay petitioner the
amount of ten thousand pesos (P10,000.00) as attorneys fee, the amount of five thousand pesos
(P5,000.00) as litigation expenses, and three hundred pesos (P300.00) as judicial cost.
In due time, petitioner appealed the trial court's decision to the Court of Appeals.
On December 20, 1996, the Court of Appeals rendered decision reversing the appealed decision,
and entering a new judgment dismissing petitioner's complaint without prejudice to any action that
petitioner may take if the subject land was declassified from forest land to alienable and disposable
land of the public domain.
Hence, the present recourse.
Petitioner submits that the Court of Appeals erred in setting aside the trial court's decision in his
favor and dismissing the complaint because when the Director of Lands allowed petitioner to file a
public land application for said property, it was equivalent to a declaration that said land was no
longer part of the public domain.
We deny the petition. The Court of Appeals correctly held that "the evidence is unrebutted that the
subject land is within the Forest Reserve Area as per L.C. Map No. 1557 certified on August 13,
1951."[1] and, hence, not capable of private appropriation and occupation.[2]
In Republic vs. Register of Deeds of Quezon, we held that "Forest lands, like mineral or timber lands
which are public lands, are not subject to private ownership unless they under the Constitution,
become private properties. In the absence of such classification, the land remains unclassified
public land until released therefrom and rendered open to disposition. [3]
In Sunbeam Convenience Foods Inc. vs. Court of Appeals, we said: Thus, before any land may be
declassified from the forest group and converted into alienable or disposable land for agricultural or
other purposes, there must be a positive act from the government. Even rules on the confirmation
of imperfect titles do not apply unless and until the land classified as forest land is released in an
official proclamation to that effect so that it may form part of the disposable agricultural lands of the
public domain."[4]
Hence, a positive act of the government is needed to declassify a forest land into alienable or
disposable land for agricultural or other purposes.[5]
And the rule is Possession of forest lands, however long, cannot ripen into private ownership. [6]
What is more, there is yet no award or grant to petitioner of the land in question by free patent or
other ways of acquisition of public land. Consequently, he can not lawfully claim to be the owner of
the land in question.
WHEREFORE, the Court hereby AFFIRMS the appealed decision of the Court of Appeals in CA-G.
R. CV No. 42306, dismissing the complaint of petitioner before the Regional Trial Court, Basilan
province, in Civil Case No. 441-63.
No costs.
SO ORDERED.

Full Text Cases in Natural Resources


32
8.Republic vs CA and Dela Rosa GR. No. L-43938 04/15/1988

G.R. No. L-43938 April 15, 1988

REPUBLIC OF THE PHILIPPINES (DIRECTOR OF FOREST DEVELOPMENT), petitioner,


vs.
HON. COURT OF APPEALS (THIRD DIVISION) and JOSE Y. DE LA ROSA, respondents.

G.R. No. L-44081 April 15, 1988

BENGUET CONSOLIDATED, INC., petitioner,


vs.
HON. COURT OF APPEALS, JOSE Y. DE LA ROSA, VICTORIA, BENJAMIN and EDUARDO,
all surnamed DE LA ROSA, represented by their father JOSE Y. DE LA ROSA, respondents.

G.R. No. L-44092 April 15, 1988

ATOK-BIG WEDGE MINING COMPANY, petitioner,


vs.
HON. COURT OF APPEALS, JOSE Y. DE LA ROSA, VICTORlA, BENJAMIN and EDUARDO,
all surnamed DE LA ROSA, represented by their father, JOSE Y. DE LA ROSA, respondents.

CRUZ, J.:

The Regalian doctrine reserves to the State all natural wealth that may be found in the bowels of
the earth even if the land where the discovery is made be private. 1 In the cases at bar, which have
been consolidated because they pose a common issue, this doctrine was not correctly applied.

These cases arose from the application for registration of a parcel of land filed on February 11,
1965, by Jose de la Rosa on his own behalf and on behalf of his three children, Victoria, Benjamin
and Eduardo. The land, situated in Tuding, Itogon, Benguet Province, was divided into 9 lots and
covered by plan Psu-225009. According to the application, Lots 1-5 were sold to Jose de la Rosa
and Lots 6-9 to his children by Mamaya Balbalio and Jaime Alberto, respectively, in 1964. 2

The application was separately opposed by Benguet Consolidated, Inc. as to Lots 1-5, Atok Big
Wedge Corporation, as to Portions of Lots 1-5 and all of Lots 6-9, and by the Republic of the
Philippines, through the Bureau of Forestry Development, as to lots 1-9. 3

In support of the application, both Balbalio and Alberto testified that they had acquired the subject
land by virtue of prescription Balbalio claimed to have received Lots 1-5 from her father shortly after
the Liberation. She testified she was born in the land, which was possessed by her parents under
claim of ownership. 4 Alberto said he received Lots 6-9 in 1961 from his mother, Bella Alberto, who
declared that the land was planted by Jaime and his predecessors-in-interest to bananas, avocado,
nangka and camote, and was enclosed with a barbed-wire fence. She was corroborated by Felix
Marcos, 67 years old at the time, who recalled the earlier possession of the land by Alberto's

Full Text Cases in Natural Resources


33
5
father. Balbalio presented her tax declaration in 1956 and the realty tax receipts from that year to
1964, 6 Alberto his tax declaration in 1961 and the realty tax receipts from that year to 1964. 7

Benguet opposed on the ground that the June Bug mineral claim covering Lots 1-5 was sold to it on
September 22, 1934, by the successors-in-interest of James Kelly, who located the claim in
September 1909 and recorded it on October 14, 1909. From the date of its purchase, Benguet had
been in actual, continuous and exclusive possession of the land in concept of owner, as evidenced
by its construction of adits, its affidavits of annual assessment, its geological mappings, geological
samplings and trench side cuts, and its payment of taxes on the land. 8

For its part, Atok alleged that a portion of Lots 1-5 and all of Lots 6-9 were covered by the Emma
and Fredia mineral claims located by Harrison and Reynolds on December 25, 1930, and recorded
on January 2, 1931, in the office of the mining recorder of Baguio. These claims were purchased
from these locators on November 2, 1931, by Atok, which has since then been in open, continuous
and exclusive possession of the said lots as evidenced by its annual assessment work on the claims,
such as the boring of tunnels, and its payment of annual taxes thereon. 9

The location of the mineral claims was made in accordance with Section 21 of the Philippine Bill of
1902 which provided that:

SEC. 21. All valuable mineral deposits in public lands in the philippine Islands both surveyed and
unsurveyed are hereby declared to be free and open to exploration, occupation and purchase and
the land in which they are found to occupation and purchase by the citizens of the United States, or
of said islands.

The Bureau of Forestry Development also interposed its objection, arguing that the land sought to
be registered was covered by the Central Cordillera Forest Reserve under Proclamation No. 217
dated February 16, 1929. Moreover, by reason of its nature, it was not subject to alienation under
the Constitutions of 1935 and 1973. 10

The trial court * denied the application, holding that the applicants had failed to prove their claim of
possession and ownership of the land sought to be registered. 11 The applicants appealed to the
respondent court, * which reversed the trial court and recognized the claims of the applicant, but
subject to the rights of Benguet and Atok respecting their mining claims. 12 In other words, the Court
of Appeals affirmed the surface rights of the de la Rosas over the land while at the same time
reserving the sub-surface rights of Benguet and Atok by virtue of their mining claims.

Both Benguet and Atok have appealed to this Court, invoking their superior right of ownership. The
Republic has filed its own petition for review and reiterates its argument that neither the private
respondents nor the two mining companies have any valid claim to the land because it is not
alienable and registerable.

It is true that the subject property was considered forest land and included in the Central Cordillera
Forest Reserve, but this did not impair the rights already vested in Benguet and Atok at that time.
The Court of Appeals correctly declared that:

Full Text Cases in Natural Resources


34
There is no question that the 9 lots applied for are within the June Bug mineral claims of Benguet
and the "Fredia and Emma" mineral claims of Atok. The June Bug mineral claim of plaintiff Benguet
was one of the 16 mining claims of James E. Kelly, American and mining locator. He filed his
declaration of the location of the June Bug mineral and the same was recorded in the Mining
Recorder's Office on October 14, 1909. All of the Kelly claims ha subsequently been acquired by
Benguet Consolidated, Inc. Benguet's evidence is that it had made improvements on the June Bug
mineral claim consisting of mine tunnels prior to 1935. It had submitted the required affidavit of
annual assessment. After World War II, Benguet introduced improvements on mineral claim June
Bug, and also conducted geological mappings, geological sampling and trench side cuts. In 1948,
Benguet redeclared the "June Bug" for taxation and had religiously paid the taxes.

The Emma and Fredia claims were two of the several claims of Harrison registered in 1931, and
which Atok representatives acquired. Portions of Lots 1 to 5 and all of Lots 6 to 9 are within the
Emma and Fredia mineral claims of Atok Big Wedge Mining Company.

The June Bug mineral claim of Benguet and the Fredia and Emma mineral claims of Atok having
been perfected prior to the approval of the Constitution of the Philippines of 1935, they were
removed from the public domain and had become private properties of Benguet and Atok.

It is not disputed that the location of the mining claim under consideration was perfected prior to
November 15, 1935, when the Government of the Commonwealth was inaugurated; and according
to the laws existing at that time, as construed and applied by this court in McDaniel v. Apacible and
Cuisia (42 Phil. 749), a valid location of a mining claim segregated the area from the public domain.
Said the court in that case: The moment the locator discovered a valuable mineral deposit on the
lands located, and perfected his location in accordance with law, the power of the United States
Government to deprive him of the exclusive right to the possession and enjoyment of the located
claim was gone, the lands had become mineral lands and they were exempted from lands that could
be granted to any other person. The reservations of public lands cannot be made so as to include
prior mineral perfected locations; and, of course, if a valid mining location is made upon public lands
afterwards included in a reservation, such inclusion or reservation does not affect the validity of the
former location. By such location and perfection, the land located is segregated from the public
domain even as against the Government. (Union Oil Co. v. Smith, 249 U.S. 337; Van Mess v.
Roonet, 160 Cal. 131; 27 Cyc. 546).

"The legal effect of a valid location of a mining claim is not only to segregate the area from the public
domain, but to grant to the locator the beneficial ownership of the claim and the right to a patent
therefor upon compliance with the terms and conditions prescribed by law. Where there is a valid
location of a mining claim, the area becomes segregated from the public domain and the property
of the locator." (St. Louis Mining & Milling Co. v. Montana Mining Co., 171 U.S. 650; 655; 43 Law
ed., 320, 322.) "When a location of a mining claim is perfected it has the effect of a grant by the
United States of the right of present and exclusive possession, with the right to the exclusive
enjoyment of all the surface ground as well as of all the minerals within the lines of the claim, except
as limited by the extralateral right of adjoining locators; and this is the locator's right before as well
as after the issuance of the patent. While a lode locator acquires a vested property right by virtue of
his location made in compliance with the mining laws, the fee remains in the government until patent
issues."(18 R.C.L. 1152) (Gold Creek Mining Corporation v. Hon. Eulogio Rodriguez, Sec. of

Full Text Cases in Natural Resources


35
Agriculture and Commerce, and Quirico Abadilla, Director of the Bureau of Mines, 66 Phil. 259, 265-
266)

It is of no importance whether Benguet and Atok had secured a patent for as held in the Gold Creek
Mining Corp. Case, for all physical purposes of ownership, the owner is not required to secure a
patent as long as he complies with the provisions of the mining laws; his possessory right, for all
practical purposes of ownership, is as good as though secured by patent.

We agree likewise with the oppositors that having complied with all the requirements of the mining
laws, the claims were removed from the public domain, and not even the government of the
Philippines can take away this right from them. The reason is obvious. Having become the private
properties of the oppositors, they cannot be deprived thereof without due process of law. 13

Such rights were not affected either by the stricture in the Commonwealth Constitution against the
alienation of all lands of the public domain except those agricultural in nature for this was made
subject to existing rights. Thus, in its Article XIII, Section 1, it was categorically provided that:

SEC. 1. All agricultural, timber and mineral lands of the public domain, waters, minerals, coal,
petroleum and other mineral oils, all forces of potential energy and other natural resources of the
Philipppines belong to the State, and their disposition, exploitation, development, or utilization shall
be limited to citizens of the Philippines or to corporations or associations at least 60% of the capital
of which is owned by such citizens, subject to any existing right, grant, lease or concession at the
time of the inauguration of the government established under this Constitution. Natural resources
with the exception of public agricultural lands, shall not be alienated, and no license, concession, or
lease for the exploitation, development or utilization of any of the natural resources shall be granted
for a period exceeding 25 years, except as to water rights for irrigation, water supply, fisheries, or
industrial uses other than the development of water power, in which case beneficial use may be the
measure and the limit of the grant.

Implementing this provision, Act No. 4268, approved on November 8, 1935, declared:

Any provision of existing laws, executive order, proclamation to the contrary notwithstanding, all
locations of mining claim made prior to February 8, 1935 within lands set apart as forest reserve
under Sec. 1826 of the Revised Administrative Code which would be valid and subsisting location
except to the existence of said reserve are hereby declared to be valid and subsisting locations as
of the date of their respective locations.

The perfection of the mining claim converted the property to mineral land and under the laws then
in force removed it from the public domain. 14 By such act, the locators acquired exclusive rights
over the land, against even the government, without need of any further act such as the purchase
of the land or the obtention of a patent over it. 15As the land had become the private property of the
locators, they had the right to transfer the same, as they did, to Benguet and Atok.

It is true, as the Court of Appeals observed, that such private property was subject to the
"vicissitudes of ownership," or even to forfeiture by non-user or abandonment or, as the private
respondents aver, by acquisitive prescription. However, the method invoked by the de la Rosas is
not available in the case at bar, for two reasons.

Full Text Cases in Natural Resources


36
First, the trial court found that the evidence of open, continuous, adverse and exclusive possession
submitted by the applicants was insufficient to support their claim of ownership. They themselves
had acquired the land only in 1964 and applied for its registration in 1965, relying on the earlier
alleged possession of their predecessors-in-interest. 16The trial judge, who had the opportunity to
consider the evidence first-hand and observe the demeanor of the witnesses and test their credibility
was not convinced. We defer to his judgment in the absence of a showing that it was reached with
grave abuse of discretion or without sufficient basis. 17

Second, even if it be assumed that the predecessors-in-interest of the de la Rosas had really been
in possession of the subject property, their possession was not in the concept of owner of the mining
claim but of the property as agricultural land, which it was not. The property was mineral land, and
they were claiming it as agricultural land. They were not disputing the lights of the mining locators
nor were they seeking to oust them as such and to replace them in the mining of the land. In fact,
Balbalio testified that she was aware of the diggings being undertaken "down below" 18 but she did
not mind, much less protest, the same although she claimed to be the owner of the said land.

The Court of Appeals justified this by saying there is "no conflict of interest" between the owners of
the surface rights and the owners of the sub-surface rights. This is rather doctrine, for it is a well-
known principle that the owner of piece of land has rights not only to its surface but also to everything
underneath and the airspace above it up to a reasonable height. 19 Under the aforesaid ruling, the
land is classified as mineral underneath and agricultural on the surface, subject to separate claims
of title. This is also difficult to understand, especially in its practical application.

Under the theory of the respondent court, the surface owner will be planting on the land while the
mining locator will be boring tunnels underneath. The farmer cannot dig a well because he may
interfere with the operations below and the miner cannot blast a tunnel lest he destroy the crops
above. How deep can the farmer, and how high can the miner, go without encroaching on each
other's rights? Where is the dividing line between the surface and the sub-surface rights?

The Court feels that the rights over the land are indivisible and that the land itself cannot be half
agricultural and half mineral. The classification must be categorical; the land must be either
completely mineral or completely agricultural. In the instant case, as already observed, the land
which was originally classified as forest land ceased to be so and became mineral — and completely
mineral — once the mining claims were perfected. 20 As long as mining operations were being
undertaken thereon, or underneath, it did not cease to be so and become agricultural, even if only
partly so, because it was enclosed with a fence and was cultivated by those who were unlawfully
occupying the surface.

What must have misled the respondent court is Commonwealth Act No. 137, providing as follows:

Sec. 3. All mineral lands of the public domain and minerals belong to the State, and their disposition,
exploitation, development or utilization, shall be limited to citizens of the Philippines, or to
corporations, or associations, at least 60% of the capital of which is owned by such citizens, subject
to any existing right, grant, lease or concession at the time of the inauguration of government
established under the Constitution.

Full Text Cases in Natural Resources


37
SEC. 4. The ownership of, and the right to the use of land for agricultural, industrial, commercial,
residential, or for any purpose other than mining does not include the ownership of, nor the right to
extract or utilize, the minerals which may be found on or under the surface.

SEC. 5. The ownership of, and the right to extract and utilize, the minerals included within all areas
for which public agricultural land patents are granted are excluded and excepted from all such
patents.

SEC. 6. The ownership of, and the right to extract and utilize, the minerals included within all areas
for which Torrens titles are granted are excluded and excepted from all such titles.

This is an application of the Regalian doctrine which, as its name implies, is intended for the benefit
of the State, not of private persons. The rule simply reserves to the State all minerals that may be
found in public and even private land devoted to "agricultural, industrial, commercial, residential or
(for) any purpose other than mining." Thus, if a person is the owner of agricultural land in which
minerals are discovered, his ownership of such land does not give him the right to extract or utilize
the said minerals without the permission of the State to which such minerals belong.

The flaw in the reasoning of the respondent court is in supposing that the rights over the land could
be used for both mining and non-mining purposes simultaneously. The correct interpretation is that
once minerals are discovered in the land, whatever the use to which it is being devoted at the time,
such use may be discontinued by the State to enable it to extract the minerals therein in the exercise
of its sovereign prerogative. The land is thus converted to mineral land and may not be used by any
private party, including the registered owner thereof, for any other purpose that will impede the
mining operations to be undertaken therein, For the loss sustained by such owner, he is of course
entitled to just compensation under the Mining Laws or in appropriate expropriation proceedings. 21

Our holding is that Benguet and Atok have exclusive rights to the property in question by virtue of
their respective mining claims which they validly acquired before the Constitution of 1935 prohibited
the alienation of all lands of the public domain except agricultural lands, subject to vested rights
existing at the time of its adoption. The land was not and could not have been transferred to the
private respondents by virtue of acquisitive prescription, nor could its use be shared simultaneously
by them and the mining companies for agricultural and mineral purposes.

WHEREFORE, the decision of the respondent court dated April 30, 1976, is SET ASIDE and that
of the trial court dated March 11, 1969, is REINSTATED, without any pronouncement as to costs.

SO ORDERED.

9.Heirs of Gozo vs Philippine Union Mission Corporation of the Seventh Day Adventist Church GR.
No. 195990 08/05/2015

[ GR No. 195990, Aug 05, 2015 ]

HEIRS OF RAFAEL GOZO v. PHILIPPINE UNION MISSION CORPORATION OF SEVENTH DAY


ADVENTIST CHURCH +

Full Text Cases in Natural Resources


38
DECISION

PEREZ, J.:
This is a Petition for Review on Certiorari[1] filed by petitioners Heirs of Rafael Gozo seeking to reverse
and set aside the 10 November 2010 Decision [2] of the Court of Appeals and its 14 February 2011
Resolution[3] in CA-G.R. CV No. 00188. The assailed decision and resolution reversed the 30 June
2004 Decision of the Regional Trial Court (RTC) of Kapatagan, Lanao del Norte and held that the action
for nullification and recovery of possession filed by the petitioners is already, barred by laches. The
dispositive portion of the Court of Appeals Decision reads:

ACCORDINGLY, the Decision dated 30 June 2004 of the court a quo is REVERSED and SET ASIDE.
The South Philippine Union Mission of the Seventh Day Adventist Church remains the absolute owner
of the donated property.[4]
The Facts

Petitioners claim that they are the heirs of the Spouses Rafael and Concepcion Gozo (Spouses Gozo)
who, before their death, were the original owners of a parcel of land with an area 236,638 square meters
located in Sitio Simpak, Brgy. Lala, Municipality of Kolambugan, Lanao del Norte. The respondents
claim that they own a 5,000 square-meter portion of the property. The assertion is based on the 28
February 1937 Deed of Donation[5] in favor of respondent Philippine Union Mission Corporation of the
Seventh Day Adventist (PUMCO-SDA). Respondents took possession of the subject property by
introducing improvements thereon through the construction of a church building, and later on, an
elementary school. On the date the Deed of Donation is executed in 1937, the Spouses Gozo were not
the registered owners of the property yet although they were the lawful possessors thereof. It was only
on 5 October 1953 that the Original Certificate of Title (OCT) No. P-642 covering the entire property
was issued in the name of Rafael Gozo (Rafael) married to Concepcion Gozo (Concepcion) pursuant
to the Homestead Patent granted by the President of the Philippines on 22 August 1953. [6]

In view of Rafael's prior death, however, his heirs, Concepcion, and their six children, namely, Abnera,
Benia, Castillo, Dilbert, Filipinas and Grace caused the extrajudicial partition of the property.
Accordingly, the Register of Deeds of Lanao del Norte issued a new certificate of title under Transfer
Certificate of Title (TCT) No. (T-347)-292[7] under the names of the heirs on 13 January 1954.

On 30 July 1992, Concepcion caused the survey and the subdivision of the entire property including
the portion occupied by PUMCO-SDA.[8] It was at this point that respondents brought to the attention
of Concepcion that the 5,000 square-meter portion of the property is already owned by respondent
PUMCO-SDA in view of the Deed of Donation she executed together with her husband in their favor in
1937. When Concepcion, however, verified the matter with the Register Deeds, it appeared that the
donation was not annotated in the title. The absence of annotation of the so-called encumbrance in the
title prompted petitioners not to recognize the donation claimed by the respondents. The matter was
left unresolved until Concepcion died and the rest of the owners continued to pursue their claims to
recover the subject property from the respondents.

A compromise was initially reached by the parties wherein the petitioners were allowed by respondents
to harvest from the coconut trees planted on the subject property but a misunderstanding ensued
causing respondents to file a case for qualified theft against the petitioners.
Full Text Cases in Natural Resources
39

On 19 June 2000 or around six decades after the Deed of Donation was executed, petitioners filed an
action for Declaration of Nullity of Document, Recovery of Possession and Ownership with Damages
against PUMCO-SDA before the RTC of Kapatagan, Lanao del Norte.[9]In their-Complaint docketed as
Civil Case No. 21-201, petitioners claimed that the possession of PUMCO-SDA on the subject property
was merely tolerated by petitioners and therefore could not ripen into ownership.[10] In addition,
petitioners argued that the signatures of the Spouses Gozo were forged underscoring the stark contrast
between the genuine signatures of their parents from the ones appearing in the deed. [11] Finally,
petitioners averred that granting for the sake of argument that the said signatures were genuine, the
deed of donation will remain invalid for lack of acceptance which is an essential requisite for a valid
contract of donation.[12]

For their part, respondents insisted on the validity of the donation and on the genuineness of the
signatures of the donors who had voluntarily parted with their property as faithful devotees of the church
for the pursuit of social and religious ends.[13] They further contended that from the moment the
Spouses Gozo delivered the subject property to respondents in 1937, they were already in open, public,
continuous and adverse possession thereof in the concept of an owner.[14] A considerable improvement
was claimed to have been introduced into the property in the form of church and school
buildings.[15] The argument of the petitioners, therefore, that the donation was invalid for lack of
acceptance, a question which came 63 years after it was executed, is already barred by laches.

After the pre-trial conference, trial on the merits ensued. Both parties adduced documentary and
testimonial evidence to support their respective positions.

On 30 June 2004, the RTC rendered a Decision[16] in favor of the petitioners thereby declaring that they
are the rightful owners of the subject property since the contract of donation which purportedly
transferred the. ownership of the subject property to PUMCO-SDA is void for lack of acceptance. In
upholding the right of the petitioners to the land, the court a quo held that an action or defense for the
declaration of nullity of a contract does not prescribe. Anent the claim that petitioners slept on their
rights, the RTC adjudged that the equitable doctrine of laches is inapplicable in the case at bar because
the action of the registered owners to recover possession is based on Torrens title which cannot be
barred by laches. The RTC disposed in this wise:

WHEREFORE, in view of the foregoing consideration, judgment is hereby rendered in favor of the
[petitioners], to wit:

Declaring the 5,000 square meter portion of the land covered by TCT [No.] (T-347)-292 part of the
(1)
common property of the [petitioners]; and

(2) Declaring the Deed of Donation as void.

The [petitioners], however, are not entitled to damages, attorney's fees and cost of litigation prayed
for.[17]
On appeal, the Court of Appeals reversed the RTC Decision[18] and ordered the dismissal of petitioners'
complaint on the ground of laches. The appellate court opined that petitioners failed to assert their
rights over the land for more than 60 years, thus, laches had set in. Even if petitioners were the
registered owners of the land in dispute, laches would still bar them from recovering possession of the
same.
Full Text Cases in Natural Resources
40

The Motion for Reconsideration filed by the petitioners was likewise denied by the appellate court in a
Resolution[19] dated 14 February 2011.

Petitioners are now before this Court via this instant Petition for Review on Certiorari seeking the
reversal of the Court of Appeals Decision and Resolution on the sole ground that:

THE HONORABLE COURT OF APPEALS ERRED IN REVERSING AND SETTING ASIDE THE
DECISION OF THE COURT A QUOON THE GROUND OF LACHES.[20]
The Court's Ruling

While the opposing parties center their arguments and counterarguments on the timeliness of raising
the question of the validity of' donation, a careful scrutiny of the records, however, reveals a significant
fact that at the time the Deed of Donation was executed by the Spouses Gozo on 28 February 1937,
the subject property was part of the inalienable public domain. It was only almost after two decades
later or on 5 October 1953 that the State ceded its right over the land in favor of the Spouses Gozo by
granting their patent application and issuing an original certificate of title in their favor. Prior to such
conferment of title, the Spouses Gozo possessed no right to dispose of the land which, by all intents
and purposes, belongs to the State.

Under the Regalian doctrine, which is embodied in Article XII, Section 2 of our Constitution, all lands of
the public domain belong to the State, which is the source of any asserted right to any ownership of
land. All lands not appearing to be clearly within private ownership are presumed to belong to the State.
Accordingly, public lands not shown to have been reclassified or released as alienable agricultural land
or alienated to a private person by the State remain part of the inalienable public domain. [21]

The classification of public lands is an exclusive prerogative of the executive department of the
government and not the Courts. In the absence of such classification, the land remains as an
unclassified land until it is released therefrom and rendered open to disposition. This is in consonance
with the Regalian doctrine that all lands of the public domain belong to the State and that the State is
the source of any asserted right to ownership in land and charged with the conservation of such
patrimony.[22]

All lands not appearing to be clearly within private ownership are presumed to belong to the State.
Accordingly, all public lands not shown to have been reclassified or released as alienable agricultural
land or alienated to a private person by the State remain part of the alienable public domain. As already
well-settled in jurisprudence, no public land can be acquired by private persons without any grant,
express or implied, from the government; and it is indispensable that the person claiming title to public
land should show that his title was acquired from the State or any other mode of acquisition recognized
by law. To prove that the land subject of an application for registration is alienable, the applicant must
establish the existence of a positive act of the government such as a presidential proclamation or an
executive order, an administrative action, investigation reports of Bureau of Lands investigators, and a
legislative act or a statute. The applicant may also secure a certification from the Government that the
land applied for is alienable and disposable.[23]

Commonwealth Act No. 141, also known as the Public Land Act, as amended by Presidential Decree
No. 1073, remains to this day the existing general law governing the classification and disposition of
lands of the public domain, other than timber and mineral lands. The following provisions under Title I,
Full Text Cases in Natural Resources
41
Chapter II of the Public Land Act, as amended, is very specific on how lands of the public domain
become alienable or disposable:[24]

SEC. 6. The President, upon the recommendation of the Secretary of Agriculture and Natural
Resources, shall from time to time classify the lands of the public domain into:

(a) Alienable or disposable,


(b) Timber, and
(c) Mineral lands,
and may at any time and in a like manner transfer such lands from one class to another, for the purposes
of their administration and disposition.

SEC. 7. For the purposes of the administration and disposition of alienable or disposable public lands,
the Batasang Pambansa or the President, upon recommendation by the Secretary of Natural
Resources, may from time to time declare what public lands are open to disposition or concession
under this Act.

x x x x

SEC. 8. Only those lands shall be declared open to disposition or concession which have been officially
delimited and classified and, when practicable, surveyed, and which have not been reserved for public
or quasi-public uses, nor appropriated by the Government, nor in any manner become private property,
nor those on which a private right authorized and recognized by this Act or any other valid law may be
claimed, or which, having been reserved or appropriated, have ceased to be so. However, the President
may, for reasons of public interest, declare lands of the public domain open to disposition before the
same have had their boundaries established or been surveyed, or may, for the same reason, suspend
their concession or disposition until they are again declared open to concession or disposition by
proclamation duly published or by Act of the Congress.

SEC. 9. For the purpose of their administration and disposition, the lands of the public domain alienable
or open to disposition shall be classified, according to the use or purposes to which such lands are
destined, as follows:

(a) Agricultural;
(b) Residential, commercial, industrial, or for similar productive purposes;
(c) Educational, charitable, or other similar purposes; and
(d) Reservations for townsites and for public and quasi-public uses.
The President, upon recommendation by the Secretary of Agriculture and Natural Resources, shall
from time to time make the classifications provided for in this section, and may, at any time and in a
similar manner, transfer lands from one class to another.[25]
By virtue of Presidential Decree No. 705, otherwise known as the Revised Forestry Code, the President
delegated to the DENR Secretary the power to determine which of the unclassified lands of the public
domain are (1) needed for forest purposes and declare them as permanent forest to form part of the
forest reserves; and (2) not needed for forest purposes and declare them as alienable and disposable
lands.[26]

Per the Public Land Act, alienable and disposable public lands suitable for agricultural purposes can

Full Text Cases in Natural Resources


42
be disposed of only as follows:

1. For homestead settlement;


2. By sale;
3. By lease; and
4. By confirmation of imperfect or incomplete titles:

(a) By judicial legalization;


(b) By administrative legalization (free patent).[27]
Homestead over alienable and disposable public agricultural land is granted after compliance
by an applicant with the conditions and requirements laid down under Title II, Chapter IV of the
Public Land Act, the most basic of which are quoted below:

SEC. 12. Any citizen of the Philippines over the age of eighteen years, or the head of a family, who
does not own more than twenty-four hectares of land in the Philippines or has not had the benefit of
any gratuitous allotment of more than twenty-four hectares of land since the occupation of the
Philippines by the United States, may enter a homestead of not exceeding twenty-four hectares of
agricultural land of the public domain.

SEC. 13. Upon the filing of an application for a homestead, the Director of Lands, if he finds that the
application should be approved, shall do so and authorize the applicant to take possession of the land
upon the payment of five pesos, Philippine currency, as entry fee. Within six months from and after the
date of the approval of the application, the applicant shall begin to work the homestead, otherwise he
shall lose his prior right to the land.

SEC. 14. No certificate shall be given or patent issued for the land applied for until at least one-fifth of
the land has been improved and cultivated. The period within which the land shall be cultivated shall
not be less than one nor more than five years, from and after the date of the approval of the application.
The applicant shall, within the said period, notify the Director of Lands as soon as he is ready to acquire
the title. If at the date of such notice, the applicant shall prove to the satisfaction of the Director of
Lands, that he has resided continuously for at least one year in the municipality in which the land is
located, or in a municipality adjacent to the same and has cultivated at least one-fifth of the land
continuously since the approval of the application, and shall make affidavit that no part of said land has
been alienated or encumbered, and that he has complied with all the requirements of this Act, then,
upon the payment of five pesos, as final fee, he shall be entitled to a patent.[28]
It is clear under the law that before compliance with the foregoing conditions and requirements the
applicant has no right over the land subject of the patent and therefore cannot dispose the same even
if such disposal was made gratuitously. It is an established principle that no one can give what one
does not have, nemo dat quod non habet.[29] It is true that gratuitous disposal in donation may consist
of a thing or a right but the term right must be understood in a "proprietary" sense over which the
possessor has jus disponendi.[30] This is because in true donations there results a consequent
impoverishment of the donor or diminution of his assets.[31] In Republic v. Court of Appeals,[32] the Court
declared the contract of donation, executed by the donor who has no proprietary right over the object
of the contract, null and void, viz:

Even on the gratuitous assumption that a donation of the military "camp site" was executed between
Eugenio de Jesus and Serafin Marabut, such donation would anyway be void because Eugenio de
Jesus held no dominical rights over the site when it was allegedly donated by him in 1936. In
Full Text Cases in Natural Resources
43
that year, Proclamation No. 85. of President Quezon already withdrew the area from sale or settlement
and reserved it for military purposes, x x x Eugenio de Jesus cannot be said to be possessed of that
"proprietary" right over the whole 33 hectares in 1936 including the disputed 12.8081 hectares for at
the time this 12.8081-hectare lot had already been severed from the mass disposable public lands by
Proclamation No. 85 and excluded from the Sales Award. Impoverishment of Eugenio's asset as a
result of such donation is therefore farfetched. (Emphasis supplied)
It is beyond question that at the time the gratuitous transfer was effected by the Spouses Gozo on 28
February 1937, the subject property was part of the public domain and is outside the commerce of
man. It was only on 5 October 1953 that the ownership of the property was vested by the State to the
Spouses Gozo by virtue of its issuance of the OCT pursuant to the Homestead Patent granted by the
President of the Philippines on 22 August 1953. Hence, the donation of the subject property which
took place before 5 October 1953 is null and void from the very start.[33]

As a void contract, the Deed of Donation produces no legal effect whatsoever. Quod nullum est, nullum
producit effectum[34] That which is a nullity produces no effect.[35] Logically, it could not have transferred
title to the subject property from the Spouses Gozo to PUMCO-SDA and there can be no basis for the
church's demand for the issuance of title under its name. Neither does the church have the right to
subsequently dispose the property nor invoke acquisitive prescription to justify its occupation. A void
contract is not susceptible to ratification, and the action for the declaration of absolute nullity of such
contract is imprescriptible.[36]

The lack of respondents' right over the property was confirmed when the Spouses Gozo had the entire
property, including the portion occupied by the church, surveyed and patented, and covered by their
homestead patent. Further, after a certificate of title was issued under their names, the Spouses Gozo
did not effect the annotation thereon of the supposed donation. Registration is the operative act that
gives validity to the transfer or creates a lien upon the land.[37] Indeed it has been ruled that where there
was nothing in the certificate of title to indicate any cloud or vice in the ownership of the property, or
any encumbrance thereon, the purchaser is not required to explore farther than what the Torrens title
upon its face indicates in quest for any hidden defect or inchoate right that may subsequently defeat
his right thereto.[38] If the rule were otherwise, the efficacy and conclusiveness of the certificate of title
which the Torrens system seeks to insure would entirely be futile and nugatory. [39] The public shall then
be denied of its foremost motivation for respecting and observing the Land Registration Act. [40]

Just as significantly, the homestead application of the Spouses Gozo over the entire area of the
property including that occupied by respondents and the issuance in their favor of the corresponding
title without any complaint or objection from the respondents, remove the case of the petitioners from
the operation of the doctrine of laches.

And, further than the issuance of an original title, the entire property was made subject of an
extrajudicial partition of the property by the Gozo heirs resulting in the issuance of TCTs in their names
in 1954. Again, in no instance during the partition did the respondents make known their claim over the
property.

Clearly from the facts, the petitioners asserted their rights repeatedly; it was the respondents who kept
silent all throughout about the supposed donee's rights.

WHEREFORE, premises considered, the instant petition is hereby GRANTED. The assailed Decision
dated 10 November 2010 and Resolution dated 14 February 2011 of the Court of Appeals in CA-G.R.
Full Text Cases in Natural Resources
44
CV No. 00188 are hereby REVERSED and SET ASIDE.

SO ORDERED.

10.Sangguniang Panglalawigan of Bataan vs Garcia Jr. GR No. 174964 10/05/2016

REYES, J.:
Before this Court is a Petition for Review on Certiorart[1] of the Decision[2] dated February 7, 2006
of the Court of Appeals (CA) in CA-G.R. SP No. 85902 upholding the Decision dated November 29,
2002 of the Regional Trial Court (RTC) of Bataan which granted the petition for a writ of mandamus
in Special Civil Action No. 7043.

Antecedent Facts

Lot Nos. 2193 and 2194 of the Bataan Cadastre, containing 1,222 square meters and 10,598 sq m,
respectively, were registered in the name of the Province of Bataan. Both lots were embraced in
Original Certificate of Title (OCT) No. N-182, and occupied by the Bataan Community Colleges
(BCC) and the Medina Lacson de Leon School of Arts and Trades (MLLSAT), both State-run
schools.[3]

On February 26, 1998, the Congress of the Philippines passed Republic Act (R.A.) No. 8562,
authored by Congressman Enrique T. Garcia, Jr. (Cong. Garcia), converting the MLLSAT into a
polytechnic college, to be known as the Bataan Polytechnic State College (BPSC), and integrating
thereto the BCC.[4] Section 24 of R.A. No. 8562 provides that:

All parcels of land belonging to the government occupied by the Medina Lacson de Leon School of
Arts and Trades and the Bataan Community Colleges are hereby declared to be the property of the
Bataan Polytechnic State College and shall be titled under that name: Provided, That should the
State College cease to exist or be abolished or should such parcels of land aforementioned be no
longer needed by the State College, the same shall revert to the Province of Bataan.

On the basis of the above provision, Cong. Garcia wrote to then Governor of Bataan Leonardo
Roman, and the Sangguniang Panlalawigan of Bataan (petitioner), requesting them to cause the
transfer of the title of the aforesaid lots to BPSC. No transfer was effected. [5]

Thus, Cong. Garcia, along with the faculty members and some concerned students of BPSC
(collectively, the respondents) filed a Special Civil Action for Mandamus with the RTC of Balanga,
Bataan against the Governor and the petitioner. Initially, the Board of Trustees of the BPSC was
impleaded as an unwilling plaintiff but was eventually included as co-petitioner in the civil suit
Full Text Cases in Natural Resources
45
pursuant to Resolution No. 14, Series of 2000 of the BPSC. [6]

In their Comment, the Governor and the petitioner took issue with the standing of the respondents,
arguing that they were not the real parties in interest who would be benefited or injured by the
judgment, or the party entitled to the avails of the suit. They asserted that the subject properties
were owned by the Province of Bataan and not the State, for them to be simply transferred to the
BPSC by virtue of the law.[7]

In its Decision dated November 29, 2002, the RTC granted the writ of mandamus. The fallo of the
RTC decision reads:

WHEREFORE, a writ of mandamus is hereby issued, ordering respondents to forthwith:

1. Deliver the owner's duplicate copy of [OCX] No. N-182 to the Register of Deeds of Bataan, free
from any hen or encumbrance;

2. Execute the corresponding deed of conveyance of the parcels of land in issue in favor of the
[BPSC]; and

3. Cause the transfer and registration of the title to and in the name of the [BPSC].

SO ORDERED.[8]

The Governor and the petitioner appealed to the CA alleging that the subject lots were the
patrimonial properties of the Province of Bataan, and as such they cannot be taken by the National
Government without due process of law and without just compensation. They also pointed out that
certain loan obligations of the Province of Bataan to the Land Bank of the Philippines (LBP) were
secured with a mortgage on the lots; and since the mortgage lien was duly annotated on its title,
OCT No. N-182, the writ of mandamus violated the non-impairment clause of the Constitution. The
Governor and the petitioner reiterated that the respondents had no legal standing since they were
not the real parties in interest.[9]

In the Decision[10] dated February 7, 2006, the CA affirmed the RTC.

The CA rejected the claim that the subject lots were the patrimonial properties of the Province of
Bataan, declaring that the petitioner failed to provide proof that the Province of Bataan acquired
them with its own private or corporate funds, and for this reason the lots must be presumed to belong
to the State, citing Salas, etc., et al. v. Hon. Jarencio, etc., et al.[11] Concerning the mortgage to the
LBP, the appellate court agreed with the RTC that the consent of the LBP to the transfer of title to
BPSC must be obtained, and the mortgage lien must be carried over to the new title. The CA also
held that BPSC is a real party in interest on the basis of Section 24 of R.A. No. 8562, and was
correctly impleaded as a co-petitioner. The subsequent motion for reconsideration was denied in
the CA Resolution[12] dated September 20, 2006; hence, this petition.

Full Text Cases in Natural Resources


46

Issues

WHETHER OR NOT THE SUBJECT PARCELS OF LAND ARE PATRIMONIAL PROPERTIES OF


THE PROVINCE OF BATAAN WHICH CANNOT BE TAKEN WITHOUT DUE PROCESS OF LAW
AND WITHOUT JUST COMPENSATION.

II

WHETHER OR NOT A WRIT OF MANDAMUS MAY BE ISSUED AGAINST THE PETITIONER TO


COMPEL THE TRANSFER OF THE SUBJECT PROPERTIES WITHOUT DUE PROCESS OF
LAW AND WITHOUT JUST COMPENSATION.[13]

The petitioner insists that the subject lots are not communal lands, or legua comunal as they were
known under the laws of colonial Spain, but are the patrimonial properties of the Province of Bataan,
which were issued a Torrens title by the Cadastral Court on August 11, 1969 in Cadastral Case No.
5;[14] that while in Salas,[15] the title of the State over the disputed lot was expressly recognized by
the City of Manila, this is not so in the case at bar;[16] that in the exercise of its proprietary rights over
the subject lots, the Province of Bataan has used them as collateral for its loan obligations with the
LBP;[17] that in its Manifestation and Motion dated February 24, 2000, the Board of Trustees of BPSC
even acknowledged the titles of the Province of Bataan over the subject properties. [18]

In addition to the above contentions, the petitioner proffers an alleged novel argument that R.A. No.
8562 infringes on the State's underlying policy of local autonomy for its territorial and political
subdivisions, found in Article X of the 1987 Constitution (formerly Article XI, 1973 Constitution) and
now fleshed out in a landmark legislation, R.A. No. 7160, better known as the Local Government
Code of 1991 (LGC). Thus, for this Court to still sustain its ruling in Salas would render the State's
policy of local autonomy purely illusory.[19]

Ruling of the Court

The decision of the CA is affirmed.

A. Under the well-entrenched and


time-honored Regalian Doctrine, all
Full Text Cases in Natural Resources
47
lands of the public domain are
under the absolute control and
ownership of the State.

The State's ownership of and control over all lands and resources of the public domain are beyond
dispute. Reproducing almost verbatim from the 1973 Constitution,[20] Section 2, Article XII of the
1987 Constitution provides that "[a]ll lands of the public domain, waters, minerals, coal, petroleum
and other mineral oils, all forces of potential energy, fisheries, forests or timber, wildlife, flora and
fauna, and other natural resources are owned by the State, x x x." In Section 1, Article XIII of the
Amended 1935 Constitution, it was also provided that "[a]ll agricultural timber, and mineral lands
of the public domain, waters, minerals, coal, petroleum, and other mineral oils, all forces of
potential energy and other natural resources of the Philippines belong to the State x x x."

Thus, in Cariño v. Insular Government,[21] a case of Philippine origin, the Supreme Court of the
United States of America acknowledged that "Spain in its earlier decrees embodied the universal
feudal theory that all lands were held from the Crown x x x." In Hong Hok v.
David,[22] citing Cariño, the Court likewise said that the theory is a manifestation of the concept of
the Regalian Doctrine, or jura regalia,[23] which is enshrined in our 1935, 1973, and 1987
Constitutions. As adopted in our republican system, this medieval concept is stripped of royal
overtones; and ownership of all lands belonging to the public domain is vested in the
State.[24] Under this well-entrenched and time-honored Regalian Doctrine, all lands of the public
domain are under the absolute control and ownership of the State.

B. Local government property


devoted to governmental purposes,
such as local administration, public
education, and public health, as
may be provided under special
laws, is classified as public.

In The Province of Zamboanga del Norte v. City of Zamboanga, et al.[25] cited by the CA, the
Province of Zamboanga del Norte sought to declare unconstitutional R.A. No. 3039, which
ordered the transfer of properties belonging to the Province of Zamboanga located within the
territory of the City of Zamboanga to the said City, for depriving the province of property without
due process and just compensation. In said case, the Court classified properties of local
governments as either (a) properties for public use, or (b) patrimonial properties, and held that the
capacity in which the property is held by a local government is dependent on the use to which it is
intended and for which it is devoted. If the property is owned by the municipal corporation in its
public and governmental capacity, it is public and Congress has absolute control over it; but if the
property is owned in its private or proprietary capacity, then it is patrimonial and Congress has no
absolute control, in which case, the municipality cannot be deprived of it without due process and
payment of just compensation.[26] In upholding the validity of R.A. No. 3 039, the Court noted that
it affected "lots used as capitol site, school sites and its grounds, hospital and leprosarium sites
and the high school playground sites - a total of 24 lots - since these were held by the former
Zamboanga province in its governmental capacity and therefore are subject to the absolute
control of Congress." [27]

According to the Court, there are two established norms to determine the classification of the
Full Text Cases in Natural Resources
48
properties: that of the Civil Code, particularly Articles 423 and 424 thereof, and that obtaining
under the law of Municipal Corporations. Articles 423 and 424 of the Civil Code provide, as
follows:

Art. 423. The property of provinces, cities and municipalities is divided into property for public use
and patrimonial property.

Art. 424. Property for public use, in the provinces, cities, and municipalities, consists of the provincial
roads, city streets, municipal streets, the squares, fountains, public waters, promenades, and public
works for public service paid for by said provinces, cities, or municipalities.

All other property possessed by any of them is patrimonial and shall be governed by this Code,
without prejudice to the provisions of special laws.

In Province of Zamboanga del Norte,[28] properties for the free and indiscriminate use of everyone
are classified under the Civil Code norm as for public use, while all other properties are
patrimonial in nature. In contrast, under the Municipal Corporations Law norm, to be considered
public property, it is 'enough that a property is held and devoted to a governmental purpose, such
as local administration, public education, and public health. [29] Nonetheless, the Court clarified that
the classification of properties in the municipalities, other than those for public use, as patrimonial
under Article 424 of the Civil Code, is "without prejudice to the provisions of special
laws,"[30]holding that the principles obtaining under the Law of Municipal Corporations can be
considered as "special laws"[31]

Moreover, in the 2009 case of Heirs of Mario Malabanan v. Republic of the Philippines,[32] the
Court reiterated that Article 420(2) of the Civil Code makes clear that properties "which belong to
the State, without being for public use, and are intended for some public service or for the
development of the national wealth," are public dominion property. For as long as the property
belongs to the State, although already classified as alienable or disposable, it remains property of
the public dominion when it is "intended for some public service or for the development of the
national wealth."[33]

C. Property registered in the name


of the municipal corporation but
without proof that it was acquired
with its corporate funds is deemed
held by it in trust for the State.

The Court takes instructions from the case, of Salas as to properties belonging to the municipal
government. In Salas, at issue was the constitutionality of R.A. No. 4118 passed on June 20,
1964,[34] whereby Congress reserved a lot, long titled in the name of the City of Manila, as
communal property, and converted it into disposable land of the State for resale in small lots to its
bona fide occupants. On February 24, 1919, Lot No. 1, Block 557 of the Cadastre of the City of
Manila, containing 9,689.80 sq m, was declared by the Court of First Instance of Manila, Branch 4,

Full Text Cases in Natural Resources


49
acting as a land registration court in Case No. 18, G.L.R.O. Record No. 111, as owned by the City
of Manila in fee simple. On August 21, 1920, OCT No. 4329 was issued in the name of the City of
Manila over the said lot. On various dates in 1924, the City of Manila sold portions of Lot No. 1,
Block 557 to a certain Pura Villanueva (Villanueva). OCT No. 4329 was cancelled, and transfer
certificates of title (TCT) were issued to Villanueva for the portions sold to her, while TCT No.
22547 was issued to the City of Manila for the remainder of Lot No. 1 containing 7,490.10 sq m,
now designated, as Lot No. 1-B-2-B of Block 557.[35]

On September 21, 1960, the local board of the City of Manila wrote to the President of the
Philippines seeking assistance in declaring the aforesaid lot as patrimonial property of the city for
the purpose of reselling the same in small lots to the actual occupants thereof. R.A. No. 4118 was
passed by Congress on June 20, 1964 for this purpose.[36] On February 18, 1965, Manila Mayor
Antonio Villegas (Mayor Villegas) was furnished a copy of a subdivision plan for TCT No. 22547.
He interposed no objection to the implementation of R.A. No. 4118, and TCT No. 22547 was duly
surrendered to the Land Authority.[37]

Inexplicably, now claiming that R.A. No. 4118 was unconstitutional, Mayor Villegas brought on
December 20, 1966 an action for injunction and/or prohibition with preliminary injunction, to
restrain, prohibit and enjoin the Land Authority and the Register of Deeds of Manila from
implementing R.A. No. 4118. On September 23, 1968, the RTC declared the said law
unconstitutional for depriving the City of Manila of its property without due process and just
compensation.[38]

Acting on the petition for review, the Court declared that Lot 1-B-2-B of Block 557 was a
communal property held in trust by the City of Manila for the State, and therefore subject to the
paramount power of Congress to dispose of. Thus:

[T]he City of Manila, although declared by the Cadastral Court as owner in fee simple, has not
shown by any shred of evidence in what manner it acquired said land as its private or patrimonial
property. It is true that the City of Manila as well as its predecessor, the Ayuntamiento de Manila,
could validly acquire property in its corporate or private capacity, following the accepted doctrine on
the dual character - public and private - of a municipal corporation. And when it acquires property
in its private capacity, it acts like an ordinary person capable of entering into contracts or making
transactions for the transmission of title or other real rights. When it comes to acquisition of land, it
must have done so under any of the modes established by law for the acquisition of ownership and
other real rights. In the absence of a title deed to any land claimed by the City of Manila as its own,
showing that it was acquired with its private or corporate funds, the presumption is that such land
came from the State upon the creation of the municipality (Unson vs. Lacson, et al., 100 Phil. 695).
Originally the municipality owned no patrimonial property except those that were granted by the
State not for its public but for private use. Other properties it owns are acquired in the course of the
exercise of its corporate powers as a juridical entity to which category a municipal corporation
pertains.

Communal lands or "legua comunal" came into existence when a town or pueblo was established
in this country under the laws of Spain (Law VII, Title III, Book VI, Recopilacion de las Leyes de
Indios). The municipalities of the Philippines were not entitled, as a matter of right, to any part of the
public domain for use as communal lands. The Spanish law provided that the usufruct of a portion
Full Text Cases in Natural Resources
50
of the public domain adjoining municipal territory might be granted by the Government for communal
purposes, upon proper petition, but, until granted, no rights therein passed to the municipalities,
and, in any event, the ultimate title remained in the sovereign (City of Manila vs. Insular
Government, 10 Phil. 327).

x x x x

It may, therefore, be laid down as a general rule that regardless of the source or classification of
land in the possession of a municipality, excepting those acquired with its own funds in its private
or corporate capacity, such property is held in trust for the State for the benefit of its inhabitants,
whether it be for governmental or proprietary purposes. It holds such lands subject to the paramount
power of the legislature to dispose of the same, for after all it owes its creation to it as an agent for
the performance of a part of its public work, the municipality being but a subdivision or
instrumentality thereof for purposes of local administration. Accordingly, the legal situation is the
same as if the State itself holds the property and puts it to a different use (2 Mc Quilin, Municipal
Corporations, 3rd Ed. p. 197, citing Monagham vs. Armatage, 218 Minn. 27, 15 N. W. 2nd 241).

True it is that the legislative control over a municipal corporation is not absolute even when it comes
to its property devoted to public use, for such control must not be exercised to the extent of depriving
persons of their property or rights without due process of law, or in a manner impairing the
obligations of contracts. Nevertheless, when it comes to property of the municipality which it did not
acquire in its private or corporate capacity with its own funds, the legislature can transfer its
administration and disposition to an agency of the National Government to be exposed of according
to its discretion. Here it did so in obedience to the constitutional mandate of promoting social justice
to insure the well-being and economic security of the people.[39] (Underscoring ours)

D. R.A. No. 8562 was not intended


to expropriate the subject lots titled
in the name of the Province of
Bataan, but to confirm their
character as communal land of the
State and to make them available
for disposition by the National
Government.

The case of Rabuco v. Hon. Villegas,[40] decided in 1974, is a virtual reprise of the 1968 case
of Salas. In Rabuco, the constitutionality of R.A. No. 3120[41] was challenged, which provided for
the subdivision of Lot No. 21-B, Block 610 of the Cadastre of the City of Manila, containing about
10,198 sq m into residential lots, and the sale thereof to the tenants and bona fide occupants. The
law declared Lot No. 21-B "reserved as communal property" and then ordered it converted into
"disposable and alienable lands of the State."[42]

The Court ruled that, like R.A. No. 4118 in Salas, R.A. No. 3120 was intended to implement the
social justice policy of the Constitution and the government's program of land for the landless.
Thus, the sale of the subdivided lots to the bona fide occupants by authority of Congress was not

Full Text Cases in Natural Resources


51
an exercise of eminent domain or expropriation without just compensation, which would have
been in violation of Section 1(2),[43] Article III of the 1935 Constitution, but simply a manifestation
of its right and power to deal with State property.[44] "It is established doctrine that the act of
classifying State property calls for the exercise of wide discretionary legislative power which will
not be interfered with by the courts."[45] In Rabuco, the rule in Salas was reiterated that property of
the public domain, although titled to the local government, is held by it in trust for the State. It
stated:

The Court [in Salas] reaffirmed the established general rule that "regardless of the source or
classification of land in the possession of a municipality, excepting those acquired with its own funds
in its private or corporate capacity, such property is held in trust for the State for the benefit of its
inhabitants, whether it be for governmental or proprietary purposes. It holds such lands subject to
the paramount power of the legislature to dispose of the same, for after all it owes its creation to it
as an agent for the performance of a part of its public work, the municipality being but a subdivision
or instrumentality thereof for purposes of local administration. Accordingly, the legal situation is the
same as if the State itself holds the property and puts it to a different use" and stressed that "the
property, as has been previously shown, was not acquired by the City of Manila with its own funds
in its private or proprietary capacity. That it has in its name a registered title is not questioned, but
this title should be deemed to be held in trust for the State as the land covered thereby was part of
the territory of the City of Manila granted by the sovereign upon its creation."[46]

E. The State's policy to promote


local autonomy and to devolve the
powers of the National Government
to its political subdivisions has for
its purpose to improve the quality
of local governance.

Sections 2 and 3, Article X of the 1987 Constitution, relied upon by the petitioner, provide:

Sec. 2. The territorial and political subdivisions shall enjoy local autonomy.

Sec. 3. The Congress shall enact a local government code which shall provide for a more responsive
and accountable local government structure instituted through a system of decentralization with
effective mechanisms of recall, initiative, and referendum, allocate among the different local
government units their powers, responsibilities, and resources, and provide for the qualifications,
election, appointment and removal, term, salaries, powers and functions and duties of local officials,
and all other matters relating to the organization and operation of the local units.

Pursuant to its mandate, the Congress passed the LGC in 1991 to spell out the above-declared
policy of the State, which is now amplified in Section 2 of R.A. No. 7160. It states, as follows:

Full Text Cases in Natural Resources


52

Sec. 2. Declaration of Policy. - (a) It is hereby declared the policy of the State that the territorial and
political subdivisions of the State shall enjoy genuine and meaningful local autonomy to enable them
to attain their fullest development as self-reliant communities and make them more effective
partners in the attainment of national goals. Toward this end, the State shall provide for a more
responsive and accountable local government structure instituted through a system of
decentralization whereby local government units shall be given more powers, authority,
responsibilities, and resources. The process of decentralization shall proceed from the National
Government to the local government units.

xxxx

Also invoked by the petitioners are Sections 18 and 22 of the LGC, which state as follows:

Sec. 18. Power to Generate and Apply Resources. — Local government units shall have the power
and authority to establish an organization that shall be responsible for the efficient and effective
implementation of their development plans, program objectives and priorities; to create their own
sources of revenues and to levy taxes, fees, and charges which shall accrue exclusively for their
use and disposition and which shall be retained by them; to have a just share in national taxes which
shall be automatically and directly released to them without need of any further action; to have an
equitable share in the, proceeds from the utilization and development of the national wealth and
resources within their respective territorial jurisdictions including sharing the same with the
inhabitants by way of direct benefits; to acquire, develop, lease, encumber, alienate, or otherwise
dispose of real or personal property held by them in their proprietary capacity and to apply their
resources and assets for productive, developmental, or welfare purposes, in the exercise or
furtherance of their governmental or proprietary powers and functions and thereby ensure their
development into self-reliant communities and active participants in the attainment of national goals.

Sec. 22. Corporate Powers. - x x x

x x x x

(d) Local government units shall enjoy full autonomy in the exercise of their proprietary functions
and in the management of their economic enterprises, subject to the limitations provided in this
Code and other applicable laws.

In the instant petition, it is essentially the petitioner's assertion that the State's policy of local
autonomy and decentralization endows the Province of Bataan with patrimonial rights to use or
dispose of the subject lots according to its own development plans, program objectives and
priorities.

Full Text Cases in Natural Resources


53
The Court disagrees.

Local autonomy and decentralization of State powers to the local political subdivisions are the
results of putting restraints upon the exercise by the Presidents of executive powers over local
governments. Section 4, Article X of the 1987 Constitution reads in part: "The President of the
Philippines shall exercise general supervision over local governments." As with the counterpart
provisions of our earlier Constitutions, the aforesaid provision has been interpreted to exclude the
President's power of control over local governments.[47] The Constitutions of 1935, 1973 and 1987
have uniformly differentiated the President's power of supervision over local governments and his
power of control of the executive departments, bureaus and offices.[48] In Pimentel, Jr. v. Hon.
Aguirre,[49] it was held that Section 4 confines the President's power over local governments to one
of general supervision, thus:

Under our present system of government, executive power is vested in the President. The members
of the Cabinet and other executive officials are merely alter egos. As such, they are subject to the
power of control of the President, at whose will and behest they can be removed from office; or their
actions and decisions changed, suspended or reversed. In contrast, the heads of political
subdivisions are elected by the people. Their sovereign powers emanate from the electorate, to
whom they are directly accountable. By constitutional fiat, they are subject to the President's
supervision only, not control, so long as their acts are exercised within the sphere of their legitimate
powers. By the same token, the President may not withhold or alter any authority or power given
them by the Constitution and the law.[50]

On the other hand, local autonomy and decentralization of State powers to the local political
subdivisions have for their object to make governance directly responsive at the local levels by
giving them a free hand to chart their own destiny and shape their future with minimum intervention
from central authorities, thereby rendering them accountable to their local
[51]
constituencies. Thus, [h]and in hand with the constitutional restraint on the President's power
over local governments is the state policy of ensuring local autonomy"[52] As farther explained
in Pimentel, Jr.:

Under the Philippine concept of local autonomy, the national government has not completely
relinquished all its powers over local governments, including autonomous regions. Only
administrative powers over local affairs are delegated to political subdivisions. The purpose of the
delegation is to make governance more directly responsive and effective at the local levels. In turn,
economic, political and social development at the smaller political units are expected to propel social
and economic growth and development. But to enable the country to develop as a whole, the
programs and policies effected locally must be integrated and coordinated towards a common
national goal. Thus, policy-setting for the entire country still lies in the President and Congress. As
we stated in Magtajas v. Pryce Properties Corp., Inc., municipal governments are still agents of the
national government.[53] (Citation omitted)

Full Text Cases in Natural Resources


54

It is clear, then, that local autonomy and decentralization do not deal directly with Issues
concerning ownership, classification, use or control of properties of the public domain held by local
governments. The State retains power over property of the public domain, exercised through
Congress.

F. The grant of autonomy to local


governments, although a radical
policy change under the 1973 and
1987 Constitutions, does not affect
the settled rule that they possess
property of the public domain in
trust for the State.

The 1973 Constitution devoted an entire Article, Article XI, consisting of five sections, to laying
down its policy for the empowerment of the local governments. The 1987 Constitution, in turn, fully
devotes all 21 sections of its Article X for local government. It introduces significant new
provisions, such as the establishment of autonomous regions (Section 18) and the guarantee of
just share of the local governments in the national taxes and equitable share in the proceeds from
the utilization of the national wealth (Sections 6 and 7). It was unlike in the 1935 Constitution,
which simply provided in Section 10 of Article VII, dealing with the Executive Department, that
"[t]he President shall have control of all executive departments, bureaus or offices, exercise
general provision over all local governments as may be provided by law, and take care that the
laws be faithfully executed."

The erudite Justice Enrique Fernando (Justice Fernando), in his highly instructive separate
concurring opinion in Rabuco,[54] did at first admit to doubts as to the continuing authoritativeness
of Province of Zamboanga del Norte and Salas, both promulgated before the effectivity of the
1973 Constitution, in view of the significant innovations introduced therein pertaining to the
autonomy of local governments. He stated that the goal of the 1973 Constitution was "the fullest
autonomy to local government units consistent with the basic theory of a unitary, not a federal,
polity,"[55] hoping thereby to attain "their fullest development as self-reliant
communities."[56]According to him, under the 1973 Constitution, "[tjhings have changed
radically,"[57] noting that under the 1935 Constitution, "[i]t could hardly be assumed x x x that x x x
the [local governments] could justifiably lay claim to real autonomy." [58] He observed thus:

We start with the declared principle of the State guaranteeing and promoting the autonomy of local
government units. We have likewise noted the earnestness of the framers as to the attainment of
such declared objective as set forth in the specific article on the matter. It is made obligatory on the
National Assembly to enact a local government code. What is more, unlike the general run of
statutes, it cannot be amended except by a majority vote of all its members. It is made to include "a
more responsive and accountable local government structure with an effective system of recall,"
with an expressed reference to "qualifications, election and removal, term, salaries, powers,
functions, and duties of local officials, [as well as] all other matters relating to the organization and
operation of the local units." Mention is likewise made of the "powers, responsibilities, and
resources," items that are identified with local autonomy. As if that were not enough, the last
Full Text Cases in Natural Resources
55
sentence of this particular provision reads: "However, any change in the existing form of local
government shall not take effect until ratified by a majority of the votes cast in a plebiscite called for
the purpose." To the extent that the last section requires that the creation, division, merger, abolition
or alteration of a boundary of a province, city, municipality, or barrio, must be in accordance with
the criteria established in the local government code and subject to the approval by a majority of
the votes cast in a plebiscite in such unit or units, the adherence to the basic principle of local self
government is quite clear. Equally significant is the stress on the competence of a province, city,
municipality or barrio "to create its own sources of revenue and to levy taxes subject to such
limitations as may be provided by law." The care and circumspection with which the framers saw to
the enjoyment of real local self-government not only in terms of administration but also in terms of
resources is thus manifest. Their intent is unmistakable. Unlike the case under the 1935
Constitution, there is thus a clear manifestation of the presumption now in favor of a local
government unit. It is a well-nigh complete departure from what was. Nor should it be ignored that
a highly urbanized city "shall be independent" not only of the national government but also of a
province. Would it not follow then that under the present dispensation, the moment property is
transferred to it by the national government, its control over the same should be as extensive and
as broad as possible, x x x.[59] (Citations omitted)

Up to that point, it could almost be presumed that Justice Fernando would dissent from the
lucid ponencia of Justice Claudio Teehankee (Justice Teehankee), borne of logical doubts as to
whether Province of Zamboanga del Norte and Salas still retained their unimpaired doctrinal force
under the then new 1973 Constitution. But two considerations kept him reined in, so to speak.
One was Justice Teehankee's "reference to the ratio decidendi of [Salas] as to the trust character
impressed on communal property of a municipal corporation, even if already titled," [60] "regardless
of the source of classification of land in the possession of a municipality, excepting those acquired
with its own funds in its private or corporate capacity."[61] Justice Fernando acknowledged that the
local government "holds such [communal property] subject to the paramount power of the
legislature to dispose of the same, for after all it owes its creation to it as an agent for the
performance of a part of its public work, the municipality being but a subdivision or instrumentality
thereof for purposes of local administration."[62]

Rabuco stressed that the properties in controversy were not acquired by the City of Manila with its
own private funds. Thus, according to Justice Fernando, "That [the City of Manila] has in its name
a registered title is not questioned, but this title should be deemed to be held in trust for the State
as the land covered thereby was part of the territory of the City of Manila granted by the sovereign
upon its creation."[63] This doctrine, according to Justice Fernando, has its basis in the Regalian
Doctrine and is unaffected by the grant of extensive local autonomy under the 1973 Constitution.
"It is my view that under the [1973] Constitution, as was the case under the 1935 charter, the
holding of a municipal corporation as a unit of state does not impair the plenary power of the
national government exercising dominical rights to dispose of it in a manner it sees fit, subject to
applicable constitutional limitations as to the citizenship of the grantee."[64]

The other consideration noted by Justice Fernando in the ponencia of Justice Teehankee
in Rabuco he found further compelling was "the even more fundamental principle of social justice,
which was given further stress and a wider scope in the present Constitution." [65] He concluded

Full Text Cases in Natural Resources


56
that R.A. No. 3120, like R.A. No. 4118, was intended to implement the social justice policy of the
Constitution and the government program of land for the landless, and was not "intended to
expropriate the property involved but merely to confirm its character as communal land of the
State and to make it available for disposition by the National Government." [66]

G. The Province of Bataan has the


duty to provide an adequate
security for its loans with the LBP,
without defeating BPSC's right to
hold title to the contested lots.

The RTC ordered the Province of Bataan to deliver the owner's duplicate copy of OCT No. N-182
to the Register of Deeds of Bataan, free from any lien or encumbrance, to execute the
corresponding deed of conveyance in favor of BPSC, and to cause the transfer and registration of
the title to and in the name of the said college. The Province of Bataan erroneously believed that it
could mortgage the subject lots, notwithstanding that it held the same in trust for the State and
despite the fact that the said lots were actually being occupied by two government schools. As the
RTC urged, then, the Province of Bataan must address this issue of security for its loans with
LBP. It cannot complain that its compliance with the order of the RTC might violate the non-
impairment clause of the Constitution, since its duty to provide a replacement security for its loans
with LBP is clear.

H. BPSC is entitled to a writ of


mandamus.

Section 3, Rule 65 of the 1997 Rules of Civil Procedure provides that a writ of mandamus shall
issue where a tribunal, corporation, board, officer or person unlawfully neglects the performance
of an act which the law specifically enjoins as a duty, to command the respondent to do the act
required to be done to protect the rights of the petitioner. Herein petitioner has argued that the
mandamus applicants are not entitled thereto because they are not real parties in interest. It is a
rule re-echoed in a long line of cases that every action must be prosecuted or defended in the
name of the real party in interest, meaning "the party who stands to be benefited or injured by the
judgment in the suit, or the party entitled to the avails of the suit."[67]

At issue in this petition is Section 24 of R.A. No. 8562, which directs that "[a]ll parcels of land
belonging to the government occupied by the [MLLSAT] and the [BCC] are hereby declared to be
the property of the [BPSC] and shall be titled under that name." There is no dispute that the
Congress has expressly intended to entrust to BPSC the titles to the subject lots. Being the sole
beneficiary of Section 24 of R.A. No. 8562, BPSC is the real party in interest, and is entitled to
mandamus to enforce its right thereunder.[68]

WHEREFORE, in view of the foregoing, the petition for review on certiorari is DENIED. The
Decision of the Court of Appeals dated February 7, 2006 in CA-G.R. SP No. 85902 is AFFIRMED.

SO ORDERED.

11.Republic vs Espinosa GR No. 186603 04/05/2017

Full Text Cases in Natural Resources


57
April 5, 2017

G.R. No. 186603

REPUBLIC OF THE PHILIPPINES, represented by the REGIONAL EXECUTIVE DIRECTOR,


DENR, REGION VI, ILOILO CITY, Petitioners,
vs.
VALENTINA REGISTER OF PROVINCE OCCIDENTAL, CALISTON, DIOSCORO ESCARDA,
ESPINOSA, DEEDS OF THE OF NEG ROS LEONILA and & SPOUSES ESTRELLA, Respondents

DECISION

JARDELEZA, J.:

This is a petition for review on certiorari1 seeking to nullify the Court of Appeals' (CA) July 25, 2008
Decision2 and February 4, 2009 Resolution3 in CA-G.R. CV No. 00421. The CA modified the May 12,
2004 Decision4 of the Regional Trial Court (RTC), Branch 61 of Kabankalan City, Negros Occidental,
and dismissed the reversion case filed by the Republic of the Philippines (State) against respondents
Valentina Espinosa and her successor-in-interest, Leonila B. Caliston, to wit: WHEREFORE, the
appeal is GRANTED. The Decision dated May 12, 2004 and Order dated July 16, 2004 are
hereby modified upholding the validity of Original Certificate of Title No. 191-N and Transfer Certificate
of Title No. 91117, respectively, issued in the names of Valentina Espinosa and Leonila Caliston. The
award of damages, attorney's fees and expenses of litigation in favor of Leonila Caliston is affirmed.

SO ORDERED.5

On October 26, 1955, Cadastral Decree No. N-31626 was issued to Valentina Espinosa (Espinosa) in
Cadastral Case No. 39, L.R.C. Cadastral Record No. 980. It covered a 28,880-square meter lot located
at Lot No. 3599 of Cadastral Record No. 980, Poblacion, Sipalay City, Negros Occidental (property).
By virtue of the decree, Original Certificate of Title (OCT) No. 191-N was issued on October 15, 1962
in the name ofEspinosa.6 On June 17, 1976, Espinosa sold the property to Leonila B. Caliston
(Caliston), who was later issued Transfer Certificate of Title (TCT) No. T- 911177 on June 29, 1976.8

On January 13, 2003, the State, represented by the Regional Executive Director of the Department of
Environment and Natural Resources (DENR), Region VI, Iloilo City, through the Office of the Solicitor
General (OSG), filed a Complaint9 for annulment of title and/or reversion of land with the RTC, Branch
61 of Kabankalan City, Negros Occidental. The State claimed that the property is inalienable public
land because it fell within a timberland area indicated under Project No. 27-C, Block C per Land
Classification (LC) Map No. 2978, as certified by the Director of Forestry on January 17, 1986.10

The spouses Dioscoro and Estrella Escarda (spouses Escarda) intervened, 11 alleging that they have
been occupying the property since 1976 on the belief that it belongs to the State. 12 They prayed that
Caliston be ordered to cease and desist from ejecting them. 13

In answer, Caliston countered that the property is not timberland. Invoking laches and prescription, she
argued that her title was issued earlier in 1962, while the map shows that the property was classified
only in 1986.14 Caliston also claimed that the spouses Escarda lacked the capacity or personality to

Full Text Cases in Natural Resources


58
intervene because only the State may initiate an action for reversion. She also alleged that the spouses
Escarda cannot claim a better right as against her because she merely tolerated their occupancy of the
property until their refusal to vacate it. 15 As counterclaim, Caliston claimed for moral and exemplary
damages, attorney's fees and litigation expenses against the spouses Escarda for the baseless and
malicious complaint. 16

The RTC rendered a Decision17 dated May 12, 2004. Relying on LC Map No. 2978, the trial court ruled
in favor of the State and ordered the reversion of the property to the mass of the public domain, viz.:

WHEREFORE, premises considered, judgment 1s hereby rendered as follows:

1. Declaring Original Certificate of Title No.191-N in the name of Valentina Espinosa and all its
derivative titles, such as: TCT No. T-91117 in the name of Leonila Caliston, null and void ab
initio;

2. Ordering defendants to surrender the owner's duplicate copy of OCT No. 191-N and TCT N[o].
T-91117 to defendant Register of Deeds for the Province of Negros Occidental and the latter to
cancel said titles and all their derivative titles, if any;

3. Ordering the reversion of the land covered by the aforesaid patent and title to the mass of the
public domain under the administration and disposition of the Director of Forestry (now Regional
Executive Director, Region VI, Iloilo City);

4. Declaring that defendant Leonila Caliston has better right over the subject lot as against
intervenors Spouses Dioscoro and Estrella Escarda; and

5. Ordering the intervenors to pay defendant Leonila Caliston the following sums:

a) Not less than P20,000.00 for moral damages;

b) Not less than Pl0,000.00 for exemplary damages;

c) Not less than Pl0,000.00 for attorney's fees, plus so much appearance fees of
₱2,000.00 incurred and/or paid by answering defendant in connection with this case; and

d) Not less than ₱5,000.00 for expenses of litigation.

SO ORDERED. 18

Caliston' s motion for reconsideration 19 was denied in an Order20 dated July 16, 2004. On August 5,
2004, Caliston filed a Notice of Appeal21 with the RTC. On the other hand, the spouses Escarda did not
file a notice of appeal. Records were then forwarded to the CA, where proceedings ensued.

There, Caliston argued that the trial court improperly relied upon LC Map No. 2978, which was prepared
long after the property was alienated and awarded to Espinosa, her predecessor-in-interest. The map,
the admissibility and genuineness of which have yet to be proved, cannot be used to defeat the
cadastral proceedings presumed to have been regularly conducted. Even assuming the map can be

Full Text Cases in Natural Resources


59
considered, Caliston claims that her property is situated in an area indicated as alienable and
disposable. She also reiterated her defenses of laches and prescription. 22

For its part, the State argued that the lower court did not err in relying upon LC Map No. 2978 though
it was prepared only in 1986. According to the State, forest lands are incapable of private appropriation
and possession, however long; prescription does not run against the government. 23

The CA rendered a Decision24 dated July 25, 2008 modifying the RTC Decision. It upheld the validity
of OCT No. 191-N and TCT No. 91117 issued in the names of Espinosa and Caliston, respectively,
and affirmed the award of damages, attorney's fees, and expenses of litigation in favor of Caliston.

The CA found that the State failed to prove fraud or misrepresentation on the part of Espinosa when
she was issued OCT No. 191-N. It further ruled that the State failed to prove that the property is forest
land. The lone piece of evidence consisting of LC Map No. 2978, certified by the Director of Forestry
on January 17, 1986, was not authenticated pursuant to Section 24, 25 Rule 132 of the Rules of Court.
It noted that the parties stipulated only as to the existence of the map, but not as to its genuineness or
the truthfulness of its content. Assuming that the map is admitted in evidence, Espinosa's rights over
the property, which accrued in 1962; should not be prejudiced by a subsequent classification by the
State done in 1986, or after 24 years. 26 The CA cited27 the case of SAAD Agro-Industries, Inc. v.
Republic of the Philippines. 28

In a Resolution29 dated February 4, 2009, the CA denied the State's Motion for Reconsideration.

Hence, this petition.

The lone issue presented is whether the State has sufficiently proved that the property is part of
inalienable forest land at the time Espinosa was granted the cadastral decree and issued a title.

We deny the petition.

The State failed to prove that the property was classified as forest land at the time of the grant of the
cadastral decree and issuance of title to Espinosa.

In land registration proceedings, the applicant has the burden of overcoming the presumption of State
ownership. It must establish, through incontrovertible evidence, that the land sought to be registered is
alienable or disposable based on a positive act of the government. 30 Since cadastral proceedings are
governed by the usual rules of practice, procedure, and evidence, a cadastral decree and a certificate
of title are issued only after the applicant proves all the requisite jurisdictional facts-that they are entitled
to the claimed lot, that all parties are heard, and that evidence is considered. 31 As such, the cadastral
decree is a judgment which adjudicates ownership after proving these jurisdictional facts.32

Here, it is undisputed that Espinosa was granted a cadastral decree and was subsequently issued OCT
No. 191-N, the predecessor title of Caliston's TCT No. 91117. Having been granted a decree in a
cadastral proceeding, Espinosa can be presumed to have overcome the presumption that the land
sought to be registered forms part of the public domain. 33 This means that Espinosa, as the applicant,

Full Text Cases in Natural Resources


60
was able to prove by incontrovertible evidence that the property is alienable and disposable property in
the cadastral proceedings.

This is not to say, however, that the State has no remedy to recover the property if indeed it is part of
the inalienable lands of the public domain. The State may still do so through an action for reversion, as
in the present case.

Reversion is the remedy where the State, pursuant to the Regalian doctrine, seeks to revert land back
to the mass of the public domain. 34 It is proper when public land is fraudulently awarded and disposed
of to private individuals or corporations.35 There are also instances when we granted reversion on
grounds other than fraud, such as when a "person obtains a title under the Public Land Act which
includes, by oversight, lands which cannot be registered under the Torrens system, or when the Director
of Lands did not have jurisdiction over the same because it is of the public domain." 36

In this case, the State, through the Solicitor General, alleges neither fraud nor misrepresentation in the
cadastral proceedings and in the issuance of the title in Espinosa's favor. The argument for the State
is merely that the property was unlawfully included in the certificate of title because it is of the public
domain.

Since the case is one for reversion and not one for land registration, the burden is on the State to prove
that the property was classified as timberland or forest land at the time it was decreed to Espinosa.37 To
reiterate, there is no burden on Caliston to prove that the property in question is alienable and
disposable land. 38 At this stage, it is reasonable to presume that Espinosa, from whom Caliston derived
her title, had already established that the property is alienable and disposable land considering that she
succeeded in obtaining the OCT over it.39 In this reversion proceeding, the State must prove that there
was an oversight or mistake in the inclusion of the property in Espinosa' s title because it was of public
dominion. This is consistent with the rule that the burden of proof rests on the party who, as determined
by the pleadings or the nature of the case, asserts the affirmative of an issue.40

Here, the State hinges its whole claim on its lone piece of evidence, the land classification map
prepared in 1986.

The records show, however, that LC Map No. 2978 was not formally offered in evidence. The rules
require that documentary evidence must be formally offered in evidence after the presentation of
testimonial evidence, and it may be done orally, or if allowed by the court, in writing. 41 Due process
requires a formal offer of evidence for the benefit of the adverse party, the trial court, and the appellate
courts.42 This gives the adverse party the opportunity to examine and oppose the admissibility of the
evidence.43 When evidence has not been formally offered, it should not be considered by the court in
arriving at its decision.44 Not having been offered formally, it was error for the trial court to have
considered the survey map. Consequently, it also erred in ordering the reversion of the property to the
mass of the public domain on the basis of the same.

Moreover, even assuming that the survey can be admitted in evidence, this will not help to further the
State's cause.1âwphi1This is because the only fact proved by the map is one already admitted by the
State, that is, that the land was reclassified in 1986.45 This fact does not address the
presumption/conclusion that Espinosa has, at the time of the cadastral proceedings conducted in 1955,

Full Text Cases in Natural Resources


61
proved that the land is alienable and disposable, as evidenced by the decree issued in his favor in
1962.

II

The reclassification of the area where the property is located in 1986 should not prejudice Espinosa
and her successor-in-interest.46 Apropos is the case of Sta. Monica Industrial and Dev't Corp. v. Court
of Appeals. 47 In that case, the State offered in evidence a land classification map to prove that at the
time the land was decreed to the original owner, it had not yet been released and still fell within the
forest zone. However, the map did not conclusively state the actual classification of the land at the time
it was adjudicated to the original owner. We thus ruled that the State failed to prove that the titles should
be annulled- Finally, we find the need to emphasize that in an action to annul a judgment, the burden
of proving the judgment's nullity rests upon the petitioner. The petitioner must establish by clear and
convincing evidence that the judgment is fatally defective. When the proceedings were originally filed
by the Republic before the Court of Appeals, the petitioner contended that when the decree in favor of
De Perio was issued by Judge Ostrand in 1912 the parcels of land were still part of the inalienable
public forests. However, petitioner's case rested solely on land classification maps drawn several
years after the issuance of the decree in 1912. These maps fail to conclusively establish the actual
classification of the land in 1912 and the years prior to that. Before this Court, petitioner reiterates
said contention and refers, for the first time, to a 1908 proclamation reserving the land in Zambales as
a naval reservation and alleging that the subject parcels of land are parts thereof. These, for reasons
discussed earlier, are insufficient to overcome the legal presumption in favor of the decree's regularity,
more so when we consider that notice of the application for registration and the date of hearing thereof,
addressed to the Attorney General, the Director of Lands, the Director of Public Works and the Director
of Forestry, among others, was published in the Official Gazette and that Governor General Smith's
Proclamation of 1908 itself recognizes private rights.48

We stress that our ruling is not inconsistent with the doctrine that forest lands are outside the commerce
of man and unsusceptible of private appropriation.1âwphi1 Neither are we changing the rule on
imprescriptibility of actions for reversion. We are merely deciding on the facts as proved by the record.
To allow a reversion based on a classification made at the time when the property was already declared
private property by virtue of a decree would be akin to expropriation of land without due process of
law.49

At this juncture, we agree with the CA' s application of SAAD AgroIndustries, Inc., 50 which involved a
complaint for annulment of title and reversion of a lot covered by a free patent and original title. To
support its claim that the lot was part of the timberland and forest reserve, the State submitted as
evidence a photocopy of a land classification map. This map also became the basis of the testimonies
of City Environment and Natural Resources Office officers declaring that the lot falls within the
timberland or forest reserve. The State, however, failed to submit either a certified true copy or an
official publication of the map, prompting the trial court to deny its admission in evidence. After
proceedings, the trial court dismissed the complaint due to the State's failure to show that the subject
lot therein is part of the timberland or forest reserve or has been classified as such before the issuance
of the free patent and the original title. The CA, relying on the map, reversed the trial court.

When the case was brought before this court, we reinstated the trial court's decision. We held that the
photocopy of the land classification map cannot be considered in evidence because it is excluded under

Full Text Cases in Natural Resources


62
the best evidence rule. We emphasized that all parties, including the Government, are bound by the
rules of admissibility and must comply with it- The rules of admissibility must be applied uniformly. The
same rule holds true when the Government is one of the parties. The Government, when it comes to
court to litigate with one of its citizens, must submit to the rules of procedure and its rights and privileges
at every stage of the proceedings are substantially in every respect the same as those of its citizens; it
cannot have a superior advantage. This is so because when a [sovereign] submits itself to the
jurisdiction of the court and participates therein, its claims and rights are justiciable by every other
principle and rule applicable to the claims and rights of the private parties under similar circumstances.
Failure to abide by the rules on admissibility renders the L.C. Map submitted by respondent
inadmissible as proof to show that the subject lot is part of the forest reserve. 51

We went on to explain that even if the map was admitted in evidence to prove that the lot was classified
as part of the timberland or forest reserve, the classification was made long after private interests had
intervened. Not only was the lot already occupied and cultivated, a free patent and a certificate of title
were also awarded and issued years ahead of the classification- Even assuming that the L.C. Map
submitted by respondent is admissible in evidence, still the land in question can hardly be considered
part of the timberland or forest reserve. L.C. Map No. 2961, which purports to be the "correct map of
the areas demarcated as permanent forest pursuant of the provisions of P.D. No. 705 as amended"
was made only in 1980. Thus, the delineation of the areas was made nine (9) years after Orcullo was
awarded the free patent over the subject lot.

xxx

Obviously, private interests have intervened before classification was made pursuant to P.D. No. 705.
Not only has Orcullo by herself and through her predecessors-ininterest cultivated and possessed the
subject lot since 1930, a free patent was also awarded to her and a title issued in her name as early as
1971. In fact, it appears that the issuance of the free patent and certificate of title was regular and in
order. Orcullo complied with the requisites for the acquisition of free patent provided under
Commonwealth Act No. 141 (Public Land Act), as certified by the Director of Lands and approved by
the Secretary of Agriculture and Natural Resources.

xxx

The Regalian doctrine is well-enshrined not only in the present Constitution, but also in the 1935 and
1973 Constitutions. The Court has always recognized and upheld the Regalian doctrine as the basic
foundation of the State's property regime. Nevertheless, in applying this doctrine, we must not lose
sight of the fact that in every claim or right by the Government against one of its citizens, the paramount
considerations of fairness and due process must be observed. Respondent in this case failed to show
that the subject lot is part of timberland or forest reserve it adverted to. In the face of the uncontroverted
status of Free Patent No. 473408 and OCT No. 0-6667 as valid and regular issuances, respondent's
insistence on the classification of the lot as part of the forest reserve must be rejected. 52

These principles laid down in SAAD Agro-Industries, Inc. undoubtedly apply here. As part of fair play
and due process, the State is as bound by the rules on formal offer of evidence as much as every
private party is. More, the State's subsequent reclassification of the area where the property is situated
cannot be used to defeat the rights of a private citizen who acquired the land in a valid and regular
proceeding conducted 24 years earlier.

Full Text Cases in Natural Resources


63
The result would have been different had the State proved that the property was already classified as
part of forest land at the time of the cadastral proceedings and when title was decreed to Espinosa in
1962. However, it failed to discharge this burden; the grant of title which carries with it the presumption
that Espinosa had already proved the alienable character of the property in the cadastral proceedings
stands. To grant the reversion based on a subsequent reclassification, more so on lack of evidence,
would amount to taking of private property without just compensation and due process of law. 53 This,
however, is not what our Constitution envisions; fairness and due process are paramount
considerations that must still be observed.54

WHEREFORE, the petition for review on certiorari is DENIED. The Court of Appeals' July 25, 2008
Decision and February 4, 2009 Resolution are AFFIRMED. No costs.

SO ORDERED.

12. Lee Hong Kok vs David GR No. L-30389 12/27/1972

G.R. No. L-30389 December 27, 1972

PEDRO LEE HONG HOK, SIMEON LEE HONG HOK, ROSITA LEE HONG HOK and LEONCIO LEE
HONG HOK, petitioners,
vs.
ANIANO DAVID, THE HON. SECRETARY OF AGRICULTURE AND NATURAL RESOURCES, THE
DIRECTOR OF LANDS and COURT OF APPEALS, respondents.

Augusto A. Pardalis for petitioners.

Luis General, Jr. for respondent Aniano David.

Office of the Solicitor General for other respondents.

FERNANDO, J.:p

Petitioners 1 in this appeal by certiorari would have us reverse a decision of respondent Court of
Appeals affirming a lower court judgment dismissing their complaint to have the Torrens Title 2 of
respondent Aniano David declared null and void. What makes the task for petitioners quite difficult is
that their factual support for their pretension to ownership of such disputed lot through accretion was
rejected by respondent Court of Appeals. Without such underpinning, they must perforce rely on a legal
theory, which, to put it mildly, is distinguished by unorthodoxy and is therefore far from persuasive. A
grant by the government through the appropriate public officials 3 exercising the competence duly
vested in them by law is not to be set at naught on the premise, unexpressed but implied, that land not
otherwise passing into private ownership may not be disposed of by the state. Such an assumption is
at war with settled principles of constitutional law. It cannot receive our assent. We affirm.

The decision of respondent Court of Appeals following that of the lower court makes clear that there is
no legal justification for nullifying the right of respondent Aniano David to the disputed lot arising from

Full Text Cases in Natural Resources


64
the grant made in his favor by respondent officials. As noted in the decision under review, he "acquired
lawful title thereby pursuant to his miscellaneous sales application in accordance with which an order
of award and for issuance of a sales patent was made by the Director of Lands on June 18, 1958,
covering Lot 2892 containing an area of 226 square meters, which is a portion of Lot 2863 of the Naga
Cadastre. On the basis of the order of award of the Director of Lands the Undersecretary of Agriculture
and Natural Resources issued on August 26, 1959, Miscellaneous Sales Patent No. V-1209 pursuant
to which OCT No. 510 was issued by the Register of Deeds of Naga City to defendant-appellee Aniano
David on October 21, 1959. According to the Stipulation of Facts, since the filing of the sales application
of Aniano David and during all the proceedings in connection with said application, up to the actual
issuance of the sales patent in his favor, the plaintiffs-appellants did not put up any opposition or
adverse claim thereto. This is fatal to them because after the registration and issuance of the certificate
and duplicate certificate of title based on a public land patent, the land covered thereby automatically
comes under the operation of Republic Act 496 subject to all the safeguards provided therein.... Under
Section 38 of Act 496 any question concerning the validity of the certificate of title based on fraud should
be raised within one year from the date of the issuance of the patent. Thereafter the certificate of title
based thereon becomes indefeasible.... In this case the land in question is not a private property as the
Director of Lands and the Secretary of Agriculture and Natural Resources have always sustained the
public character thereof for having been formed by reclamation.... The only remedy therefore, available
to the appellants is an action for reconveyance on the ground of fraud. In this case we do not see any
fraud committed by defendant-appellant Aniano David in applying for the purchase of the land involved
through his Miscellaneous Sales Application No. MSA-V-26747, entered in the records of the Bureau
of Lands [Miscellaneous Sales] Entry No. V-9033, because everything was done in the open. The
notices regarding the auction sale of the land were published, the actual sale and award thereof to
Aniano David were not clandestine but open and public official acts of an officer of the Government.
The application was merely a renewal of his deceased wife's application, and the said deceased
occupied the land since 1938." 4

On such finding of facts, the attempt of petitioners to elicit a different conclusion is likely to be attended
with frustration. The first error assigned predicated an accretion having taken place, notwithstanding its
rejection by respondent Court of Appeals, would seek to disregard what was accepted by respondent
Court as to how the disputed lot came into being, namely by reclamation. It does not therefore call for
any further consideration. Neither of the other two errors imputed to respondent Court, as to its holding
that authoritative doctrines preclude a party other than the government to dispute the validity of a grant
and the recognition of the indefeasible character of a public land patent after one year, is possessed of
merit. Consequently, as set forth at the outset, there is no justification for reversal.

1. More specifically, the shaft of criticism was let loose by petitioner aimed at this legal proposition set
forth in the exhaustive opinion of then Justice Salvador Esguerra of the Court of Appeals, now a
member of this Court: "There is, furthermore, a fatal defect of parties to this action. Only the
Government, represented by the Director of Lands, or the Secretary of Agriculture and Natural
Resources, can bring an action to cancel a void certificate of title issued pursuant to a void patent
(Lucas vs. Durian, 102 Phil. 1157; Director of Lands vs. Heirs of Ciriaco Carlo, G.R. No. L-12485, July
31, 1959). This was not done by said officers but by private parties like the plaintiffs, who cannot claim
that the patent and title issued for the land involved are void since they are not the registered owners
thereof nor had they been declared as owners in the cadastral proceedings of Naga Cadastre after
claiming it as their private property. The cases cited by appellants are not in point as they refer to private
registered lands or public lands over which vested rights have been acquired but notwithstanding such

Full Text Cases in Natural Resources


65
fact the Land Department subsequently granted patents to public land applicants." 5
Petitioner ought to
have known better. The above excerpt is invulnerable to attack. It is a restatement of a principle that
dates back to Maninang v. Consolacion, 6 a 1908 decision. As was there categorically stated: "The fact
that the grant was made by the government is undisputed. Whether the grant was in conformity with
the law or not is a question which the government may raise, but until it is raised by the government
and set aside, the defendant can not question it. The legality of the grant is a question between the
grantee and the government."7 The above citation was repeated ipsissimis verbis in Salazar v. Court
of Appeals.8 Bereft as petitioners were of the right of ownership in accordance with the findings of the
Court of Appeals, they cannot, in the language of Reyes v. Rodriguez, 9 "question the [title] legally
issued." 10 The second assignment of error is thus disposed of.

2. As there are overtones indicative of skepticism, if not of outright rejection, of the well-known
distinction in public law between the government authority possessed by the state which is appropriately
embraced in the concept of sovereignty, and its capacity to own or acquire property, it is not
inappropriate to pursue the matter further. The former comes under the heading of imperium and the
latter of dominium. The use of this term is appropriate with reference to lands held by the state in its
proprietary character. In such capacity, it may provide for the exploitation and use of lands and other
natural resources, including their disposition, except as limited by the Constitution. Dean Pound did
speak of the confusion that existed during the medieval era between such two concepts, but did note
the existence of res publicae as a corollary to dominium." 11 As far as the Philippines was concerned,
there was a recognition by Justice Holmes in Cariño v. Insular Government, 12 a case of Philippine
origin, that "Spain in its earlier decrees embodied the universal feudal theory that all lands were held
from the Crown...." 13 That was a manifestation of the concept of jura regalia, 14 which was adopted by
the present Constitution, ownership however being vested in the state as such rather than the head
thereof. What was stated by Holmes served to confirm a much more extensive discussion of the matter
in the leading case of Valenton v. Murciano, 15 decided in 1904. One of the royal decrees cited was
incorporated in the Recopilacion de Leyes de las Indias 16 in these words: "We having acquired full
sovereignty over the Indies and all lands, territories, and possessions not heretofore ceded away by
our royal predecessors, or by us, or in our name, still pertaining to the royal crown and patrimony, it is
our will that all lands which are held without proper and true deeds of grant be restored to us according
as they belong to us, in order that after reserving before all what to us or to our viceroys audiences,
and governors may seem necessary for public squares, ways, pastures, and commons in those places
which are peopled, taking into consideration not only their present condition, but also their future and
their probable increase, and after distributing to the natives what may be necessary for tillage and
pasturage, confirming them in what they now have and giving them more if necessary, all the rest of
said lands may remain free and unencumbered for us to dispose of as we may wish." 17

It could therefore be affirmed in Montano v. Insular Government" 18 that "as to the unappropriated public
lands constituting the public domain the sole power of legislation is vested in Congress, ..." 19 They
continue to possess that character until severed therefrom by state grant. 20 Where, as in this case, it
was found by the Court of Appeals that the disputed lot was the result of reclamation, its being correctly
categorized as public land is undeniable. 21What was held in Heirs of Datu Pendatun v. Director of
Lands 22 finds application. Thus: "There being no evidence whatever that the property in question was
ever acquired by the applicants or their ancestors either by composition title from the Spanish
Government or by possessory information title or by any other means for the acquisition of public lands,
the property must be held to be public domain." 23 For it is well-settled "that no public land can be
acquired by private persons without any grant, express or implied, from the government." 24 It is

Full Text Cases in Natural Resources


66
indispensable then that there be a showing of a title from the state or any other mode of acquisition
recognized by law. 25 The most recent restatement of the doctrine, found in an opinion of Justice J.B.L.
Reyes, follows: 26 "The applicant, having failed to establish his right or title over the northern portion of
Lot No. 463 involved in the present controversy, and there being no showing that the same has been
acquired by any private person from the Government, either by purchase or by grant, the property is
and remains part of the public domain." 27 To repeat, the second assignment of error is devoid of merit.

3. The last error assigned would take issue with this portion of the opinion of Justice Esguerra:
"According to the Stipulation of Facts, since the filing of the sales application of Aniano David and during
all the proceedings in connection with said application, up to the actual issuance of the sales patent in
his favor, the
plaintiffs-appellants did not put up any opposition or adverse claim thereto. This is fatal to them because
after the registration and issuance of the certificate and duplicate certificate of title based on a public
land patent, the land covered thereby automatically comes under the operation of Republic Act 496
subject to all the safeguards provided therein ... Under Section 38 of Act 496 any question concerning
the validity of the certificate of title based on fraud should be raised within one year from the date of the
issuance of the patent. Thereafter the certificate of title based thereon becomes indefeasible
..." 28 Petitioners cannot reconcile themselves to the view that respondent David's title is impressed with
the quality of indefeasibility. In thus manifesting such an attitude, they railed to accord deference to
controlling precedents. As far back as 1919, in Aquino v. Director of
29
Lands, Justice Malcolm, speaking for the Court, stated: "The proceedings under the Land
Registration Law and under the provisions of Chapter VI of the Public Land Law are the same in that
both are against the whole world, both take the nature of judicial proceedings, and for both the decree
of registration issued is conclusive and final." 30Such a view has been followed since then. 31 The latest
case in point is Cabacug v. Lao. 32 There is this revealing excerpt appearing in that decision: "It is said,
and with reason, that a holder of a land acquired under a free patent is more favorably situated than
that of an owner of registered property. Not only does a free patent have a force and effect of a Torrens
Title, but in addition the person to whom it is granted has likewise in his favor the right to repurchase
within a period of five years." 33 It is quite apparent, therefore, that petitioners' stand is legally
indefensible.

WHEREFORE, the decision of respondent Court of Appeals of January 31, 1969 and its resolution of
March 14, 1969 are affirmed. With costs against petitioners-appellants.

13.Cruz vs Secretary of DENR GR NO. 135385 12/06/2000

[G.R. No. 135385. December 6, 2000]

ISAGANI CRUZ and CESAR EUROPA, petitioners, vs. SECRETARY OF ENVIRONMENT AND
NATURAL RESOURCES, SECRETARY OF BUDGET AND MANAGEMENT and CHAIRMAN
and COMMISSIONERS OF THE NATIONAL COMMISSION ON INDIGENOUS
PEOPLES, respondents.
HON. JUAN M .FLAVIER, HON. PONCIANO BENNAGEN, BAYANI ASCARRAGA, EDTAMI
MANSAYANGAN, BASILIO WANDAG, EVELYN DUNUAN, YAOM TUGAS, ALFREMO
Full Text Cases in Natural Resources
67
CARPIANO, LIBERATO A. GABIN, MATERNIDAD M. COLAS, NARCISA M. DALUPINES,
BAI KIRAM-CONNIE SATURNO, BAE MLOMO-BEATRIZ T. ABASALA, DATU
BALITUNGTUNG-ANTONIO D. LUMANDONG, DATU MANTUMUKAW TEOFISTO
SABASALES, DATU EDUAARDO BANDA, DATU JOEL UNAD, DATU RAMON BAYAAN,
TIMUAY JOSE ANOY, TIMUAY MACARIO D. SALACAO, TIMUAY EDWIN B. ENDING, DATU
SAHAMPONG MALANAW VI, DATU BEN PENDAO CABIGON, BAI NANAPNAY-LIZA
SAWAY, BAY INAY DAYA-MELINDA S. REYMUNDO, BAI TINANGHAGA HELINITA T.
PANGAN, DATU MAKAPUKAW ADOLINO L. SAWAY, DATU MAUDAYAW-CRISPEN
SAWAY, VICKY MAKAY, LOURDES D. AMOS, GILBERT P. HOGGANG, TERESA GASPAR,
MANUEL S. ONALAN, MIA GRACE L. GIRON, ROSEMARIE G. PE, BENITO CARINO,
JOSEPH JUDE CARANTES, LYNETTE CARANTES-VIVAL, LANGLEY SEGUNDO, SATUR
S. BUGNAY, CARLING DOMULOT, ANDRES MENDIOGRIN, LEOPOLDO ABUGAN,
VIRGILIO CAYETANO, CONCHITA G. DESCAGA, LEVY ESTEVES, ODETTE G. ESTEVEZ,
RODOLFO C. AGUILAR, MAURO VALONES, PEPE H. ATONG, OFELIA T. DAVI,
PERFECTO B. GUINOSAO, WALTER N. TIMOL, MANUEL T. SELEN, OSCAR DALUNHAY,
RICO O. SULATAN, RAFFY MALINDA, ALFREDO ABILLANOS, JESSIE ANDILAB,
MIRLANDO H. MANGKULINTAS, SAMIE SATURNO, ROMEO A. LINDAHAY, ROEL S.
MANSANG-CAGAN, PAQUITO S. LIESES, FILIPE G. SAWAY, HERMINIA S. SAWAY,
JULIUS S. SAWAY, LEONARDA SAWAY, JIMMY UGYUB, SALVADOR TIONGSON,
VENANCIO APANG, MADION MALID, SUKIM MALID, NENENG MALID, MANGKATADONG
AUGUSTO DIANO, JOSEPHINE M. ALBESO, MORENO MALID, MARIO MANGCAL, FELAY
DIAMILING, SALOME P. SARZA, FELIPE P. BAGON, SAMMY SALNUNGAN, ANTONIO D.
EMBA, NORMA MAPANSAGONOS, ROMEO SALIGA, SR., JERSON P. GERADA, RENATO
T. BAGON, JR., SARING MASALONG, SOLEDAD M. GERARDA, ELIZABETH L. MENDI,
MORANTE S. TIWAN, DANILO M. MALUDAO, MINORS MARICEL MALID, represented by
her father CORNELIO MALID, MARCELINO M. LADRA, represented by her father MONICO
D. LADRA, JENNYLYN MALID, represented by her father TONY MALID, ARIEL M.
EVANGELISTA, represented by her mother LINAY BALBUENA, EDWARD M. EMUY, SR.,
SUSAN BOLANIO, OND, PULA BATO BLAAN TRIBAL FARMERS ASSOCIATION, INTER-
PEOPLES EXCHANGE, INC. and GREEN FORUM-WESTERN VISAYAS, intervenors.
COMMISSION ON HUMAN RIGHTS, intervenor.
IKALAHAN INDIGENOUS PEOPLE and HARIBON FOUNDATION FOR THE CONSERVATION OF
NATURAL RESOURCES, INC., intervenor.

RESOLUTION
PER CURIAM:

Petitioners Isagani Cruz and Cesar Europa brought this suit for prohibition and mandamus as
citizens and taxpayers, assailing the constitutionality of certain provisions of Republic Act No. 8371
(R.A. 8371), otherwise known as the Indigenous Peoples Rights Act of 1997 (IPRA), and its
Implementing Rules and Regulations (Implementing Rules).
In its resolution of September 29, 1998, the Court required respondents to comment. [1] In
compliance, respondents Chairperson and Commissioners of the National Commission on Indigenous
Peoples (NCIP), the government agency created under the IPRA to implement its provisions, filed on

Full Text Cases in Natural Resources


68
October 13, 1998 their Comment to the Petition, in which they defend the constitutionality of the IPRA
and pray that the petition be dismissed for lack of merit.
On October 19, 1998, respondents Secretary of the Department of Environment and Natural
Resources (DENR) and Secretary of the Department of Budget and Management (DBM) filed through
the Solicitor General a consolidated Comment. The Solicitor General is of the view that the IPRA is
partly unconstitutional on the ground that it grants ownership over natural resources to indigenous
peoples and prays that the petition be granted in part.
On November 10, 1998, a group of intervenors, composed of Sen. Juan Flavier, one of the authors
of the IPRA, Mr. Ponciano Bennagen, a member of the 1986 Constitutional Commission, and the
leaders and members of 112 groups of indigenous peoples (Flavier, et. al), filed their Motion for Leave
to Intervene. They join the NCIP in defending the constitutionality of IPRA and praying for the dismissal
of the petition.
On March 22, 1999, the Commission on Human Rights (CHR) likewise filed a Motion to Intervene
and/or to Appear as Amicus Curiae. The CHR asserts that IPRA is an expression of the principle of
parens patriae and that the State has the responsibility to protect and guarantee the rights of those who
are at a serious disadvantage like indigenous peoples. For this reason it prays that the petition be
dismissed.
On March 23, 1999, another group, composed of the Ikalahan Indigenous People and the Haribon
Foundation for the Conservation of Natural Resources, Inc. (Haribon, et al.), filed a motion to Intervene
with attached Comment-in-Intervention. They agree with the NCIP and Flavier, et al. that IPRA is
consistent with the Constitution and pray that the petition for prohibition and mandamus be dismissed.
The motions for intervention of the aforesaid groups and organizations were granted.
Oral arguments were heard on April 13, 1999. Thereafter, the parties and intervenors filed their
respective memoranda in which they reiterate the arguments adduced in their earlier pleadings and
during the hearing.
Petitioners assail the constitutionality of the following provisions of the IPRA and its Implementing
Rules on the ground that they amount to an unlawful deprivation of the States ownership over lands of
the public domain as well as minerals and other natural resources therein, in violation of the regalian
doctrine embodied in Section 2, Article XII of the Constitution:
(1) Section 3(a) which defines the extent and coverage of ancestral domains, and Section 3(b)
which, in turn, defines ancestral lands;
(2) Section 5, in relation to section 3(a), which provides that ancestral domains including
inalienable public lands, bodies of water, mineral and other resources found within ancestral
domains are private but community property of the indigenous peoples;
(3) Section 6 in relation to section 3(a) and 3(b) which defines the composition of ancestral
domains and ancestral lands;
(4) Section 7 which recognizes and enumerates the rights of the indigenous peoples over the
ancestral domains;
(5) Section 8 which recognizes and enumerates the rights of the indigenous peoples over the
ancestral lands;

Full Text Cases in Natural Resources


69
(6) Section 57 which provides for priority rights of the indigenous peoples in the harvesting,
extraction, development or exploration of minerals and other natural resources within the
areas claimed to be their ancestral domains, and the right to enter into agreements with
nonindigenous peoples for the development and utilization of natural resources therein for
a period not exceeding 25 years, renewable for not more than 25 years; and
(7) Section 58 which gives the indigenous peoples the responsibility to maintain, develop,
protect and conserve the ancestral domains and portions thereof which are found to be
necessary for critical watersheds, mangroves, wildlife sanctuaries, wilderness, protected
areas, forest cover or reforestation.[2]
Petitioners also content that, by providing for an all-encompassing definition of ancestral domains
and ancestral lands which might even include private lands found within said areas, Sections 3(a) and
3(b) violate the rights of private landowners.[3]
In addition, petitioners question the provisions of the IPRA defining the powers and jurisdiction of
the NCIP and making customary law applicable to the settlement of disputes involving ancestral
domains and ancestral lands on the ground that these provisions violate the due process clause of the
Constitution.[4]
These provisions are:
(1) sections 51 to 53 and 59 which detail the process of delineation and recognition of ancestral
domains and which vest on the NCIP the sole authority to delineate ancestral domains and
ancestral lands;
(2) Section 52[i] which provides that upon certification by the NCIP that a particular area is an
ancestral domain and upon notification to the following officials, namely, the Secretary of
Environment and Natural Resources, Secretary of Interior and Local Governments,
Secretary of Justice and Commissioner of the National Development Corporation, the
jurisdiction of said officials over said area terminates;
(3) Section 63 which provides the customary law, traditions and practices of indigenous peoples
shall be applied first with respect to property rights, claims of ownership, hereditary
succession and settlement of land disputes, and that any doubt or ambiguity in the
interpretation thereof shall be resolved in favor of the indigenous peoples;
(4) Section 65 which states that customary laws and practices shall be used to resolve disputes
involving indigenous peoples; and
(5) Section 66 which vests on the NCIP the jurisdiction over all claims and disputes involving
rights of the indigenous peoples.[5]
Finally, petitioners assail the validity of Rule VII, Part II, Section 1 of the NCIP Administrative Order
No. 1, series of 1998, which provides that the administrative relationship of the NCIP to the Office of
the President is characterized as a lateral but autonomous relationship for purposes of policy and
program coordination. They contend that said Rule infringes upon the Presidents power of control over
executive departments under Section 17, Article VII of the Constitution.[6]
Petitioners pray for the following:
(1) A declaration that Sections 3, 5, 6, 7, 8, 52[I], 57, 58, 59, 63, 65 and 66 and other related
provisions of R.A. 8371 are unconstitutional and invalid;

Full Text Cases in Natural Resources


70
(2) The issuance of a writ of prohibition directing the Chairperson and Commissioners of the
NCIP to cease and desist from implementing the assailed provisions of R.A. 8371 and its
Implementing Rules;
(3) The issuance of a writ of prohibition directing the Secretary of the Department of
Environment and Natural Resources to cease and desist from implementing Department of
Environment and Natural Resources Circular No. 2, series of 1998;
(4) The issuance of a writ of prohibition directing the Secretary of Budget and Management to
cease and desist from disbursing public funds for the implementation of the assailed
provisions of R.A. 8371; and
(5) The issuance of a writ of mandamus commanding the Secretary of Environment and Natural
Resources to comply with his duty of carrying out the States constitutional mandate to
control and supervise the exploration, development, utilization and conservation of
Philippine natural resources.[7]
After due deliberation on the petition, the members of the Court voted as follows:
Seven (7) voted to dismiss the petition. Justice Kapunan filed an opinion, which the Chief Justice
and Justices Bellosillo, Quisumbing, and Santiago join, sustaining the validity of the challenged
provisions of R.A. 8371. Justice Puno also filed a separate opinion sustaining all challenged provisions
of the law with the exception of Section 1, Part II, Rule III of NCIP Administrative Order No. 1, series of
1998, the Rules and Regulations Implementing the IPRA, and Section 57 of the IPRA which he
contends should be interpreted as dealing with the large-scale exploitation of natural resources and
should be read in conjunction with Section 2, Article XII of the 1987 Constitution. On the other hand,
Justice Mendoza voted to dismiss the petition solely on the ground that it does not raise a justiciable
controversy and petitioners do not have standing to question the constitutionality of R.A. 8371.
Seven (7) other members of the Court voted to grant the petition. Justice Panganiban filed a
separate opinion expressing the view that Sections 3 (a)(b), 5, 6, 7 (a)(b), 8, and related provisions of
R.A. 8371 are unconstitutional. He reserves judgment on the constitutionality of Sections 58, 59, 65,
and 66 of the law, which he believes must await the filing of specific cases by those whose rights may
have been violated by the IPRA. Justice Vitug also filed a separate opinion expressing the view that
Sections 3(a), 7, and 57 of R.A. 8371 are unconstitutional. Justices Melo, Pardo, Buena, Gonzaga-
Reyes, and De Leon join in the separate opinions of Justices Panganiban and Vitug.
As the votes were equally divided (7 to 7) and the necessary majority was not obtained, the case
was redeliberated upon. However, after redeliberation, the voting remained the same. Accordingly,
pursuant to Rule 56, Section 7 of the Rules of Civil Procedure, the petition is DISMISSED.
Attached hereto and made integral parts thereof are the separate opinions of Justices Puno, Vitug,
Kapunan, Mendoza, and Panganiban.
SO ORDERED.

14. Carino vs Insular Government GR No. L – 2746 December 06, 1906

G.R. No. L-2746 December 6, 1906

Full Text Cases in Natural Resources


71
MATEO CARIÑO, petitioner-appellant,
vs.
THE INSULAR GOVERNMENT, respondent-appellee.

Coudert Brothers for appellant.

Attorney-General Wilfley for appellee.

WILLARD, J.:

The appellant, on the 23d of June, 1903, by his attorney in fact, Metcalf A. Clarke, filed a petition in the
Court of Land Registration asking that he be inscribed as the owner of a tract of land in the municipality
of Baguio, in the province of Benguet, containing 146 hectares. The Government of the Philippine
Islands, appeared in the Court of Land Registration and opposed the petition. The Government of the
United States that the land was part of the military reservation of Baguio. Judgment was entered in the
Court of Land Registration in favor of the petitioner, from which judgment the respondents appealed in
accordance with the law then in force to the Court of First Instance of the province of Benguet. The
case was therein tried de novo, and judgment was entered dismissing the petition. The petitioner has
brought the case here by bill of exceptions.

The petitioner presented no documentary evidence of title, except a possessory information obtained
in 1901. By the provisions of the Mortgage Law, under which this possessory information was obtained
(art. 394), it produced only those effects which the laws give to mere possession.

The petition not having shown any title from the Government, and the land being agricultural, the case
is governed by the decisions of this court in the cases of Valenton et al. vs. Murciano 1 (2 Off. Gaz.,
434); Cansino et al. vs. Valdez et al. 2 (4 Off. Gaz., 488); and Tiglao vs. The Insular
Government 3 (4 Off. Gaz., 747). In these cases it was held that the mere possession of land such
as that in controversy in this case would give the possessor and title thereto as against the
Government; in other words, that the statute of limitations did not run against the State in
reference to its agricultural lands.lawphil.net

The petitioner, however, insists that although the statute of limitations as such did not run against the
Government of Spain in the Philippine Islands, yet a grant is to be conclusively presumed from
immemorial use and occupation. To say that the presumption of a grant is presumption of law is, in our
opinion, simply to say that it amounts to a statute of limitations; and for a court to hold that the statute
of limitations does not run against the Government as to its public agricultural lands, and at the same
time to hold that if a person has been in possession of such lands for thirty years it is conclusively
presumed that the Government has given him a deed therefor, would be to make two rulings directly
inconsistent with each other.

Considered as a presumption of fact, the contention could not be sustained in this particular case. Here
the surrounding circumstances are incompatible with the existence of a grant, It is known that for nearly
three hundred years all attempts to convert the Igorots of the Province of Benguet to the Christian

Full Text Cases in Natural Resources


72
religion completely failed, and that during that time they remained practically in the same condition as
they were when the Islands were first occupied by the Spaniards. To presume as a matter of fact that
during that time, and down to at least 1880, the provisions of the laws relating to the grant, adjustment,
and sale of public were taken advantage of by these deeds from the Government for these lands would
be to presume something which did not exist. The appellant says in his brief (p.10):

The Igorot, no less than the American Indian, is an aborigine, and is equally ignorant of the forms
of law and procedure necessary to protect his interests.

There is, moreover, in the case evidence that in 1894 the petitioner sought to obtain title from the
Government in accordance with the laws then in force. In 1901 he made a contract with Metalcalf A.
Clarke, by the terms of which he agreed to sell the land to Clarke for 6,000 pesos when he obtained
title thereto from the Government, and this contract he does not say that he is the owner, but simply
that he is in possession thereof. The court below found that the land is now worth upwards of P50,000.

The possession of the land has not been of such a character as to require the presumption of a grant.
No one has lived upon it for many years. It was never used for anything but pasturage of animals,
except insignificant portions thereof, and since the insurrection against Spain it has apparently not been
used by the petitioner for any purpose.

The petitioner relies upon the case of the United States vs. Chaves (159 U.S., 452) and the case of
The United States vs. Chaves (175 U.S., 509). In the case of Hays vs. The United States (175 U.S.
248) the court said at page 261;

But this presumption is subject to the limitation that where title is claimed from a deed which is
shown to be void, it will not be presumed that there was an independent grant
(Smith vs. Highbee, 12 Vermont,. 113), or where surrounding circumstances are inconsistent
with the theory of a grant. (Townsend vs. Downer, 32 Vermont, 183).

The substance of this doctrine is that lapse of time any be treated as helping out the presumption
of a grant, but where a void grant is shown, it affords no presumption that another valid grant
was made. Nor does such presumption arise if the surrounding circumstances are incompatible
with the existence of a grant. In this case under consideration we can not find any evidence
which justifies us in believing that a legal grant can have been made, and under those
circumstances we can not consider possession since the date of the treaty as dispensing with
the requirement that the title, if not perfect at that time, was one which the claimant would have
a lawful right to make perfect had the territory not been acquired by the United States.

In the case of Chaves vs. The United States (175 U.S., 552) the court made the following statement at
page 562:

Finally, it distinctly appears that the possession of the parties is insufficient in length of time to
prove a valid title. In United States vs. Chaves (159 U.S., 452) the possession was under the
claim of a grant made by the governor of New Mexico to the alleged grantees. The grant had
been lost, but it had been seen and read by witnesses, and its existence had been proved by
evidence sufficient, as we stated in the opinion (p. 460), to warrant 'the finding of the court below
that the complainant's title was derived from the Republic of Mexico, and was complete and

Full Text Cases in Natural Resources


73
perfect at the date when the United States acquired sovereignty in the territory of New Mexico,
within which the land was situated. We do not question the correctness to the remarks made by
Mr. Justice Shiras in regard to evidence of possession and the presumptions which may under
certain circumstances drawn as to the existence of a grant.

We do not deny the right of the duty of a court to presume its existence in a proper case, in order
to quiet a title and to give to long continued possession the quality of a rightful possession under
a legal right. We recognized and enforced such a rule in the case of United States vs. Chaves
decided at this term. in which the question is involved. We simply say in this case that the
possession was not a duration long enough to justify any such inference.

There is no proof of any valid grant, but on the contrary the evidence offered by the plaintiff
himself and upon which the bases the title that he asks the court to confirm, shows the existence
of a grant from a body which had no legal power to make it, and which, therefore, conveyed no
title whatever to its grantee, and the evidence is, as given by the plaintiff himself, that it was
under this grant alone that possession of the lands was taken. We can not presume (within the
time involved in this case) that any other and valid grant was ever made. The possession of the
plaintiff and of his grantors up to the time of the treaty of Guadalupe Hidalgo, in 1848, had not
been long enough to presume a grant. (Crispin vs. United States, 168 U.S., 208;
Hayes vs.United States, 170 U.S., 637, 649, 653; Hays vs. The United States, ante 248.) The
possession subsequently existing, we can not notice. Same authorities.

As we understand it, it is well settled in the United States that prescription does not run against the
Government as to its public lands — in other words, that if a person desires to obtain title to the public
lands of the United States situated within the boundaries of the States, he must do so in the way pointed
out by the law. We do not understand that a person in possession of unsurveyed public lands in the
State of Minnesota, for example, whose ancestors had occupied that the land for fortyh years, could
maintain in court a claim that he was the legal owner of the lands by granted the land to his ancestors,
a presumption founded not upon any proceedings taken in the General Land Office to acquire a patent
thereto, but upon the mere possession for that length of time.

The same is true of the public lands of Spain in the Philippine Islands. In the case of Valenton et al. vs.
Marciano it was said:

While the State has always recognized the right of the occupant to a deed if he proves a
possession for a sufficient length of time, yet it has always insisted that he must make that proof
before the proper administrative officers, and obtain from them his deed, and until he did the
State remained the absolute owner.

But in any event, and whatever the law may be elsewhere, it seems clear that this doctrine of
presumptive grant can not apply to the Philippines in view of the Spanish legislation for the Indies. From
time to time there were promulgated laws which required the person in possession of public lands to
exhibit their titles or grants thereto. If these titles or grants were found to be good, they were confirmed,
but if they were not, or if the persons had no grants or titles at all, they were evicted from the land.

For example, in Law 14, title 12, 4, Recompilation of the Laws of the Indies, it is stated:

Full Text Cases in Natural Resources


74
We therefore order and command that all viceroys and presidents of pretrial courts designate,
at such times as shall to them most expedient, a suitable period within which all possessors of
tracts, farms, plantations, and estates shall exhibit to them and to the court officers appointed
by them for this purpose their title deeds thereto. And those who are in possession by virtue of
proper deeds and receipts or by virtue of just prescriptive rights shall be protected, and all the
rest shall be restored to us to be disposed of at our will.

In the Royal Cedula of October 15, 1754, it was provided —

that any and all persons who, since the year 1700, and up to the date of promulgation and
publication of said order, shall have occupied royal lands, whether or not the same shall be
cultivated or tenanted, may, either in person or through their attorneys or representatives, appear
and exhibit to said subdelegates the titles and patents by virtue of which said lands are occupied.
Said subdelegates will designate as the period within which documents must be presented a
term sufficient in length and proportionate to the distance the interested party may have to travel
for the purpose of making the presentation. Said subdelegates will at the same time warn the
parties interested that in term designated, without a just and valid reason therefor, they will be
deprived of and evicted from their lands, and they will be granted to others.

In the regulations of June 25, 1880, it was provided as follows:

ART. 8. If the interested parties shall not ask an adjustment of the lands whose possession they
are unlawfully enjoining within the time of one year, or, the adjustment having been granted by
the authorities, they shall fail to fulfill their obligation in connection with the compromise, by
paying the proper sum into the treasury, the latter will, by virtue of the authority vested in it,
reassert the ownership of the Stated over the lands, and will, after fixing the whole thereof,
proceed to sell at public auction that part of the same which, either because it may have been
reduced to cultivation or is not located within the forest zone, is not deemed advisable to
preserve as State forest reservations. 4

In the royal decree of the 13th of February, 1894, published in the Official Gazzette of Manila of the
17th of April, 01894, it is provided in article 4 as follows:

ART. 4. The title to all agricultural lands which were capable of adjustment (composicion) under
the royal decree of the 25th of June, 1880, but the adjustments of which decree in the Gaceta
de Manila, will revert to the State. Any claim to such lands by those who might have applied for
the adjustment of the same, but who have not done so as the above mentioned date, will not
avail them in any way or at any time.

In view of these provisions of the law, it seems to us impossible to say that as to the public agricultural
lands in the Philippines there existed a conclusive presumption after a lapse of thirty or any other
number of years that the Government of Spain had granted to the possessor thereof a legal title thereto.

The plaintiff is not entitled to the benefits of paragraph 6 of section 54 of Act No. 926, the Public Land
Act, for the reason that act is not applicable to the Province of Benguet. The judgment of the court
below is affirmed, with the costs of this instance against the appellant.

Full Text Cases in Natural Resources


75
After the expiration of twenty days let judgment be entered accordingly and ten days thereafter the case
be returned to the court below for execution. So ordered.

Cariño v. Insular Government, 212 U.S. 449 (1909)

Cariño v. Insular Government of the Philippine Islands

No. 72

Argued January 13, 1909

Decided February 23, 1909

212 U.S. 449

ERROR TO THE SUPREME COURT OF THE PHILIPPINE ISLANDS

Syllabus

Writ of error is the general, and appeal the exceptional, method of bringing Cases to this Court. The
latter method is in the main confined to equity cases, and the former is proper to bring up a judgment
of the Supreme Court of the Philippine Islands affirming a judgment of the Court of Land Registration
dismissing an application for registration of land.

Although a province may be excepted from the operation of Act No. 926 of 1903 of the Philippine
Commission which provides for the registration and perfecting of new titles, one who actually owns
property in such province is entitled to registration under Act No. 496 of 1902, which applies to the
whole archipelago.

While, in legal theory and as against foreign nations, sovereignty is absolute, practically it is a question
of strength and of varying degree, and it is for a new sovereign to decide how far it will insist upon
theoretical relations of the subject to the former sovereign and how far it will recognize actual facts.

Page 212 U. S. 450

The acquisition of the Philippines was not for the purpose of acquiring the lands occupied by the
inhabitants, and under the Organic Act of July 1, 1902, c. 1369, 32 Stat. 691, providing that property
rights are to be administered for the benefit of the inhabitants, one who actually owned land for many
years cannot be deprived of it for failure to comply with certain ceremonies prescribed either by the
acts of the Philippine Commission or by Spanish law.

The Organic Act of the Philippines made a bill of rights embodying safeguards of the Constitution, and,
like the Constitution, extends those safeguards to all.

Every presumption of ownership is in favor of one actually occupying land for many years, and against
the government which seeks to deprive him of it, for failure to comply with provisions of a subsequently
enacted registration act.

Full Text Cases in Natural Resources


76
Title by prescription against the crown existed under Spanish law in force in the Philippine Islands prior
to their acquisition by the United States, and one occupying land in the Province of Benguet for more
than fifty years before the Treaty of Paris is entitled to the continued possession thereof.

7 Phil. 132 reversed.

The facts are stated in the opinion.

Page 212 U. S. 455

MR. JUSTICE HOLMES delivered the opinion of the Court.

This was an application to the Philippine Court of Land Registration for the registration of certain land.
The application was granted by the court on March 4, 1904. An appeal was taken to the Court of First
Instance of the Province of Benguet on behalf of the government of the Philippines, and also on behalf
of the United States, those governments having taken possession of the property for public and military
purposes. The Court of First Instance found the facts and dismissed the application upon grounds of
law. This judgment was affirmed by the supreme court, 7 Phil. 132, and the case then was brought here
by writ of error.

The material facts found are very few. The applicant and plaintiff in error is an Igorot of the Province of
Benguet, where the land lies. For more than fifty years before the Treaty of

Page 212 U. S. 456

Paris, April 11, 1899, as far back as the findings go, the plaintiff and his ancestors had held the land as
owners. His grandfather had lived upon it, and had maintained fences sufficient for the holding of cattle,
according to the custom of the country, some of the fences, it seems, having been of much earlier date.
His father had cultivated parts and had used parts for pasturing cattle, and he had used it for pasture
in his turn. They all had been recognized as owners by the Igorots, and he had inherited or received
the land from his father in accordance with Igorot custom. No document of title, however, had issued
from the Spanish Crown, and although, in 1893-1894 and again in 1896-1897, he made application for
one under the royal decrees then in force, nothing seems to have come of it, unless, perhaps,
information that lands in Benguet could not be conceded until those to be occupied for a sanatorium,
etc., had been designated -- a purpose that has been carried out by the Philippine government and the
United States. In 1901, the plaintiff filed a petition, alleging ownership, under the mortgage law, and the
lands were registered to him, that process, however, establishing only a possessory title, it is said.

Before we deal with the merits, we must dispose of a technical point. The government has spent some
energy in maintaining that this case should have been brought up by appeal, and not by writ of error.
We are of opinion, however, that the mode adopted was right. The proceeding for registration is likened
to bills in equity to quiet title, but it is different in principle. It is a proceeding in rem under a statute of
the type of the Torrens Act, such as was discussed in Tyler v. Court of Registration, 175 Mass. 71. It is
nearer to law than to equity, and is an assertion of legal title; but we think it unnecessary to put it into
either pigeon hole. A writ of error is the general method of bringing cases to this Court, an appeal the
exception, confined to equity in the main. There is no reason for not applying the general rule to this

Full Text Cases in Natural Resources


77
case. Ormsby v. Webb, 134 U. S. 47, 134 U. S. 65; Campbell v. Porter, 162 U. S. 478; Metropolitan R.
Co. v. District of Columbia, 195 U. S. 322.

Page 212 U. S. 457

Another preliminary matter may as well be disposed of here. It is suggested that, even if the applicant
have title, he cannot have it registered, because the Philippine Commission's Act No. 926, of 1903,
excepts the Province of Benguet among others from its operation. But that act deals with the acquisition
of new titles by homestead entries, purchase, etc., and the perfecting of titles begun under the Spanish
law. The applicant's claim is that he now owns the land, and is entitled to registration under the
Philippine Commission's Act No. 496, of 1902, which established a court for that purpose with
jurisdiction "throughout the Philippine Archipelago," § 2, and authorized in general terms applications
to be made by persons claiming to own the legal estate in fee simple, as the applicant does. He is
entitled to registration if his claim of ownership can be maintained.

We come, then, to the question on which the case was decided below -- namely, whether the plaintiff
owns the land. The position of the government, shortly stated, is that Spain assumed, asserted, and
had title to all the land in the Philippines except so far as it saw fit to permit private titles to be acquired;
that there was no prescription against the Crown, and that, if there was, a decree of June 25, 1880,
required registration within a limited time to make the title good; that the plaintiff's land was not
registered, and therefore became, if it was not always, public land; that the United States succeeded
to the title of Spain, and so that the plaintiff has no rights that the Philippine government is bound to
respect.

If we suppose for the moment that the government's contention is so far correct that the Crown of Spain
in form asserted a title to this land at the date of the Treaty of Paris, to which the United States
succeeded, it is not to be assumed without argument that the plaintiff's case is at an end. It is true that
Spain, in its earlier decrees, embodied the universal feudal theory that all lands were held from the
Crown, and perhaps the general attitude of conquering nations toward people not recognized as entitled
to the treatment accorded to those

Page 212 U. S. 458

in the same zone of civilization with themselves. It is true also that, in legal theory, sovereignty is
absolute, and that, as against foreign nations, the United States may assert, as Spain asserted,
absolute power. But it does not follow that, as against the inhabitants of the Philippines, the United
States asserts that Spain had such power. When theory is left on one side, sovereignty is a question
of strength, and may vary in degree. How far a new sovereign shall insist upon the theoretical relation
of the subjects to the head in the past, and how far it shall recognize actual facts, are matters for it to
decide.

The Province of Benguet was inhabited by a tribe that the Solicitor General, in his argument,
characterized as a savage tribe that never was brought under the civil or military government of the
Spanish Crown. It seems probable, if not certain, that the Spanish officials would not have granted to
anyone in that province the registration to which formerly the plaintiff was entitled by the Spanish laws,
and which would have made his title beyond question good. Whatever may have been the technical
position of Spain, it does not follow that, in the view of the United States, he had lost all rights and was

Full Text Cases in Natural Resources


78
a mere trespasser when the present government seized his land. The argument to that effect seems to
amount to a denial of native titles throughout an important part of the island of Luzon, at least, for the
want of ceremonies which the Spaniards would not have permitted and had not the power to enforce.

The acquisition of the Philippines was not like the settlement of the white race in the United States.
Whatever consideration may have been shown to the North American Indians, the dominant purpose
of the whites in America was to occupy the land. It is obvious that, however stated, the reason for our
taking over the Philippines was different. No one, we suppose, would deny that, so far as consistent
with paramount necessities, our first object in the internal administration of the islands is to do justice
to the natives, not to exploit their country for private gain. By the Organic Act of July 1, 1902, c. 1369,
§ 12, 32 Stat. 691, all the property and rights acquired there by the

Page 212 U. S. 459

United States are to be administered "for the benefit of the inhabitants thereof." It is reasonable to
suppose that the attitude thus assumed by the United States with regard to what was unquestionably
its own is also its attitude in deciding what it will claim for its own. The same statute made a bill of rights,
embodying the safeguards of the Constitution, and, like the Constitution, extends those safeguards to
all. It provides that

"no law shall be enacted in said islands which shall deprive any person of life, liberty, or property without
due process of law, or deny to any person therein the equal protection of the laws."

§ 5. In the light of the declaration that we have quoted from § 12, it is hard to believe that the United
States was ready to declare in the next breath that "any person" did not embrace the inhabitants of
Benguet, or that it meant by "property" only that which had become such by ceremonies of which
presumably a large part of the inhabitants never had heard, and that it proposed to treat as public land
what they, by native custom and by long association -- one of the profoundest factors in human thought
-- regarded as their own.

It is true that, by § 14, the government of the Philippines is empowered to enact rules and prescribe
terms for perfecting titles to public lands where some, but not all, Spanish conditions had been fulfilled,
and to issue patents to natives for not more than sixteen hectares of public lands actually occupied by
the native or his ancestors before August 13, 1898. But this section perhaps might be satisfied if
confined to cases where the occupation was of land admitted to be public land, and had not continued
for such a length of time and under such circumstances as to give rise to the understanding that the
occupants were owners at that date. We hesitate to suppose that it was intended to declare every
native who had not a paper title a trespasser, and to set the claims of all the wilder tribes afloat. It is
true again that there is excepted from the provision that we have quoted as to the administration of the
property and rights acquired by the United States such land and property as shall be designated by the
President for military or other reservations,

Page 212 U. S. 460

as this land since has been. But there still remains the question what property and rights the United
States asserted itself to have acquired.

Full Text Cases in Natural Resources


79
Whatever the law upon these points may be, and we mean to go no further than the necessities of
decision demand, every presumption is and ought to be against the government in a case like the
present. It might, perhaps, be proper and sufficient to say that when, as far back as testimony or
memory goes, the land has been held by individuals under a claim of private ownership, it will be
presumed to have been held in the same way from before the Spanish conquest, and never to have
been public land. Certainly, in a case like this, if there is doubt or ambiguity in the Spanish law, we
ought to give the applicant the benefit of the doubt. Whether justice to the natives and the import of the
organic act ought not to carry us beyond a subtle examination of ancient texts, or perhaps even beyond
the attitude of Spanish law, humane though it was, it is unnecessary to decide. If, in a tacit way, it was
assumed that the wild tribes of the Philippines were to be dealt with as the power and inclination of the
conqueror might dictate, Congress has not yet sanctioned the same course as the proper one "for the
benefit of the inhabitants thereof."

If the applicant's case is to be tried by the law of Spain, we do not discover such clear proof that it was
bad by that law as to satisfy us that he does not own the land. To begin with, the older decrees and
laws cited by the counsel for the plaintiff in error seem to indicate pretty clearly that the natives were
recognized as owning some lands, irrespective of any royal grant. In other words, Spain did not assume
to convert all the native inhabitants of the Philippines into trespassers, or even into tenants at will. For
instance, Book 4, Title 12, Law 14 of the Recopilacion de Leyes de las Indias, cited for a contrary
conclusion in Valenton v. Murciano, 3 Phil. 537, while it commands viceroys and others, when it seems
proper, to call for the exhibition of grants, directs them to confirm those who hold by good grants or justa
prescripcion. It is true that it

Page 212 U. S. 461

begins by the characteristic assertion of feudal overlordship and the origin of all titles in the King or his
predecessors. That was theory and discourse. The fact was that titles were admitted to exist that owed
nothing to the powers of Spain beyond this recognition in their books.

Prescription is mentioned again in the royal cedula of October 15, 1754, cited in 3 Phil. 546:

"Where such possessors shall not be able to produce title deeds, it shall be sufficient if they shall show
that ancient possession, as a valid title by prescription."

It may be that this means possession from before 1700; but, at all events, the principle is admitted. As
prescription, even against Crown lands, was recognized by the laws of Spain, we see no sufficient
reason for hesitating to admit that it was recognized in the Philippines in regard to lands over which
Spain had only a paper sovereignty.

The question comes, however, on the decree of June 25, 1880, for the adjustment of royal lands
wrongfully occupied by private individuals in the Philippine Islands. This begins with the usual theoretic
assertion that, for private ownership, there must have been a grant by competent authority; but instantly
descends to fact by providing that, for all legal effects, those who have been in possession for certain
times shall be deemed owners. For cultivated land, twenty years, uninterrupted, is enough. For
uncultivated, thirty. Art. 5. So that, when this decree went into effect, the applicant's father was owner
of the land by the very terms of the decree. But, it is said, the object of this law was to require the
adjustment or registration proceedings that it described, and in that way to require everyone to get a

Full Text Cases in Natural Resources


80
document of title or lose his land. That purpose may have been entertained, but it does not appear
clearly to have been applicable to all. The regulations purport to have been made "for the adjustment
of royal lands wrongfully occupied by private individuals." (We follow the translation in the government's
brief.) It does not appear that this land ever was royal land or wrongfully occupied. In Article 6, it is
provided that

"interested parties not included within the two preceding

Page 212 U. S. 462

articles [the articles recognizing prescription of twenty and thirty years] may legalize their possession,
and thereby acquire the full ownership of the said lands, by means of adjustment proceedings, to be
conducted in the following manner."

This seems, by its very terms, not to apply to those declared already to be owners by lapse of time.
Article 8 provides for the case of parties not asking an adjustment of the lands of which they are
unlawfully enjoying the possession, within one year, and threatens that the treasury "will reassert the
ownership of the state over the lands," and will sell at auction such part as it does not reserve. The
applicant's possession was not unlawful, and no attempt at any such proceedings against him or his
father ever was made. Finally, it should be noted that the natural construction of the decree is confirmed
by the report of the council of state. That report puts forward as a reason for the regulations that, in
view of the condition of almost all property in the Philippines, it is important to fix its status by general
rules on the principle that the lapse of a fixed period legalizes completely all possession, recommends
in two articles twenty and thirty years, as adopted in the decree, and then suggests that interested
parties not included in those articles may legalize their possession and acquire ownership by
adjustment at a certain price.

It is true that the language of Articles 4 and 5 attributes title to those "who may prove" possession for
the necessary time, and we do not overlook the argument that this means may prove in registration
proceedings. It may be that an English conveyancer would have recommended an application under
the foregoing decree, but certainly it was not calculated to convey to the mind of an Igorot chief the
notion that ancient family possessions were in danger, if he had read every word of it. The words "may
prove" (acrediten), as well, or better, in view of the other provisions, might be taken to mean when
called upon to do so in any litigation. There are indications that registration was expected from all, but
none sufficient to show that, for want of it, ownership actually gained would be lost.

Page 212 U. S. 463

The effect of the proof, wherever made, was not to confer title, but simply to establish it, as already
conferred by the decree, if not by earlier law. The royal decree of February 13, 1894, declaring forfeited
titles that were capable of adjustment under the decree of 1880, for which adjustment had not been
sought, should not be construed as a confiscation, but as the withdrawal of a privilege. As a matter of
fact, the applicant never was disturbed. This same decree is quoted by the Court of Land Registration
for another recognition of the common law prescription of thirty years as still running against alienable
Crown land.

Full Text Cases in Natural Resources


81
It will be perceived that the rights of the applicant under the Spanish law present a problem not without
difficulties for courts of a different legal tradition. We have deemed it proper on that account to notice
the possible effect of the change of sovereignty and the act of Congress establishing the fundamental
principles now to be observed. Upon a consideration of the whole case, we are of opinion that law and
justice require that the applicant should be granted what he seeks, and should not be deprived of what,
by the practice and belief of those among whom he lived, was his property, through a refined
interpretation of an almost forgotten law of Spain.

Judgment reversed.

15. Acting Registrars of Land Titles and Deeds of Pasay City vs Regional Trial Court Branch 57
Makati (GR No. 81564 04/26/1990)

263 Phil. 568

SARMIENTO, J.:
The petitioners* charge His Honor, Judge Francisco Velez of the Regional Trial Court, Branch
57, Makati, Metro Manila, with grave, abuse of discretion in issuing an order authorizing the private
respondent, through Domingo Palomares, to perform acts of ownership over a 2,574-hectare parcel of
land known as Hacienda de Maricaban spread out in various parts
of Makati, Pasig, Taguig, Pasay City, andParañaque. There is no controversy as to the facts.
On November 5, 1985, the private respondent, Domingo Palomares, as administrator of the heirs
of Delfin Casal, commenced suit with the Regional Trial Court, Branch 132, Makati, Metro Manila for
declaratory relief, quieting of title, cancellation of Transfer Certificate of Title No. 192, and cancellation
of entries upon Original Certificate of Title No. 291.
Palomares had earlier come to this Court (February 27, 1985) on a similar petition, and in addition, to
direct the Register of Deeds to issue a duplicate owner's copy of Original Certificate of Title No. 291,
embracing allegedly Hacienda de Maricaban, in lieu of the (alleged) lost one. On September 9, 1985,
the Court denied the petition for lack of merit. (G.R. No. 69834).
On December 19, 1985, the petitioners filed their answer.
On June 2, 1986, the private respondent filed a motion to admit amended complaint impleading the
Republic of the Philippines and the registers of deeds of Pasig, Makati, and Pasay City as parties-
respondents, and alleging, among other things, that: (1) on October 1, 1906, the Court of Land
Registration (James Ostrand, Presiding Judge) confirmed the title of Dolores Pascual Casal y Ochoa,
a native of Madrid, Spain, over the 2,574-hectare parcel above-mentioned; (2) on October 17, 1906,
the Register of Deeds of Rizal issued OCT No. 291 in her name; (3) upon her death, and successive
deaths of her heirs, the property devolved on Gerardo, Reynaldo, Lolita and Erlinda, all
surnamed Casal, great granchildren of Dolores; (4) no conveyances or dispositions of any kind have
been allegedly made upon the parcel; (5) TCT No. 192, which covers the same landholding; is allegedly
spurious and inexistent; (6) the State itself, by placing 27,213,255 square meters thereof under a
military reservation (Fort McKinley, now Fort Bonifacio), by Proclamation No. 423, and fifty hectares
thereof pursuant to Proclamation No. 192, had been guilty of landgrabbing; (7) any and all holders of
any and all TCTs emanating therefrom or from TCT No. 192, are null, void, and of no force and effect;

Full Text Cases in Natural Resources


82
and (8) as a consequence thereof, the heirs of Dolores Casal suffered various damages and attorney's
fees.
On June 26, 1986, the petitioners filed an answer, stating, among other things, that: (1) the estate of
Dolores Casal (or Delfin Casal, her grandchild) is not a juridical person authorized by law to bring suit;
(2) the registers of deeds of Makati, Pasig, and Pasay City are not the real parties in interest, but rather,
the registered owners over which the court had not acquired jurisdiction; (3) the non-joinder of the real
parties in interest is fatal; (4) OCT No. 291 has long been cancelled; (5) Judge Gregorio Pineda of the
then Court of First Instance of Rizal, Branch XXI, Pasig, had earlier denied prayers for the issuance of
duplicate owner's copy of OCT No. 291 because the land embraced therein had been validly delivered
to the Government; (6) the Supreme Court itself had denied the Casals' appeal;** (7) as a
consequence, res judicata is a bar; (8) prescription has also set in; and (9) the Casal's claims can not
validly override the titles of innocent purchasers for value.
On August 29, 1986, the respondent judge issued a temporary restraining order, directing the
petitioners to cease and desist from performing the acts complained of.
In a subsequent memorandum, the petitioners alleged that Dolores Casal had conveyed the property
to the Government of the United States in 1906 and the Manila Railroad Company on which
Judge Ostrand, the Presiding Judge of the Court of Land Registration, later Justice of this Court had
stamped his imprimatur.
On October 12, 1987, the respondent court issued an order in the tenor, as follows:
No other opposition having been registered, this Court hereby resolves to grant the plaintiffs' prayer in
the OMNIBUS MOTION in order to safeguard the integrity of the land embraced in OCT 291, hereby
authorizing for this purpose the plaintiff Domingo C.Palomares:
1. To order such subdivision and/or individual survey or surveys within Parcel II, Parcel III and
Parcel IV under Survey Plan Psu-2031 by a licensed geodetic engineer or engineers at plaintiffs
expense in order to facilitate and simplify the efficient administration of the property described in OCT
291; and
2. To sell, exchange lease or otherwise dispose (of) any area or areas or portion or portions thereof,
subject to the approval of the Intestate Estate Court, to cover expenses for the payment of taxes to
which the property is subject, as well as expenses of administration and for the protection of the integrity
of the said lands.
SO ORDERED.[1]
Eleven days later, or on October 23, 1987 to be precise, it issued another order, as follows:
Acting on the plaintiff's MOTION dated October 15, 1987 praying for the issuance of a Writ of Execution
implementing the Order of this Court dated October 12, 1987 before the expiration of the time to appeal,
and after inquiring from the plaintiff's counsel for their reason in seeking the same, the Court hereby
issues this clarificatory order affirming the power of the plaintiff Domingo C. Palomaresto execute and
perform the acts authorized in the said Order of October 12, 1987 without the need of a Writ of
Execution, where no relief has been sought therefrom by any party, said Order being implementable at
the instance of the said plaintiff Domingo C. Palomares, anytime when the said Order becomes final
15 days after the said plaintiff received copy of the same (see Section 39, Chapter IV, B.P. Blg.
129). Plaintiff Domingo C. Palomares may therefore take whatever steps he considers appropriate for
the implementation of the said Order without need of further Orders or additional authority from this
Court.
SO ORDERED.[2]
Full Text Cases in Natural Resources
83
The petitioners filed a notice of appeal; the respondent court, however, denied it, [3] "it being directed
against ... an interlocutory order ..."[4]
Hence, this recourse.
The petitioners interpose the following questions:
A. Whether or not respondent Court can validly decide before trial in favor of private respondent the
ownership and possession of the 25,743,514 square meters (of) land known as "Hacienda
de Maricaban", which is the main issue in this case;
B. Whether or not respondent Court can validly allow private respondent to exercise and perform all
acts of ownership and possession over the said land before trial;
C. Whether or not respondent Court has acquired jurisdiction to hear and decide this action;
D. Whether or not respondent Court committed grave abuse of discretion amounting to lack of
jurisdiction in not dismissing this action or allowing petitioners to appeal from the orders in question. [5]
In their comment, the private respondent averred, among other things, that: (1) the respondent court,
contrary to the petitioners' claim, did not decide the case "before trial"; (2) OCT No. 291 had not been
validly cancelled and that the rubber stamp impression thereon, "CANCELLED" is a forgery; (3) the act
of Judge Pineda, in denying issuance of OCT No. 291, duplicate owner's copy can not be
considered res judicata because that case involved purportedly a mere petition for issuance of
duplicate owner's copy; (4) non-joinder of proper parties is not a jurisdictional defect; (5)
the TCTs issued thereafter are a nullity because OCT No. 291 had not been shown to have been duly
cancelled; (6) OCT No. 291 has become imprescriptible; and (7) the private respondent has a valid
right of dominion over the property.
In the meantime, the private respondent came to this Court on certiorari (G.R. No. 90176) alleging that
on December 15, 1987, in connection with Sp. Proc. No. Pq-2993 of the Regional Trial Court, Branch
118 Pasay City, entitled "In the Matter of the Intestate Estate of the Late Fortunato Santiago and
Mariano Pantanilla, Crisanta P. Santiago, et al., Petitioners," Judge Conrado Vasquez, Jr. issued an
order disposing of certain parcels which the private respondent claims as forming part and parcel of
Hacienda de Maricaban.
On June 20, 1988, the respondent judge in G.R. No. 81564 filed his own comment, asserting, among
other things, that: (1) what he had sought to bar, by virtue of injunction, was incursions and forcible
entries of trespassers and squatters; (2) the petitioners can not rightly claim that he had prematurely
adjudicated the case, because there was allegedly no decision to begin with; (3) that he issued the writ
of preliminary injunction in order only to maintain the status quo ante bellum, that is to re-place the
private respondent, which had been allegedly in prior possession, in possession; (4) he did not allegedly
authorize unbridled "acts of ownership" to be exercised on the property; (5) all rights of dominion given
thereon were subject to the approval of the intestate estate court; (6) he denied the notice of appeal
because the order dated October 12, 1987, was interlocutory in nature from which no appeal lies; (7)
as to jurisdiction, the various motions filed by petitioners, allegedly accepting the court's jurisdiction,
have clothed the court with jurisdiction, and that besides, the jurisdictional question was never raised
except now.
On July 7, 1988, the petitioners filed a reply traversing the respondent judge's allegations.
On August 26, 1988, the respondent judge filed a supplemental comment. He reiterated that the writ
of injunction was directed only on such spaces not occupied by the Government
(Fort Bonifacio, Libingan ng mga Bayani, Ninoy Aquino International Airport, NayongPilipino,
Population Commission, National Science and Development Board, and National Housing Authority).
Full Text Cases in Natural Resources
84
Meanwhile, Atty. Antonio J. Dalangpan, for and on behalf purportedly of the "Heirs of Delfin Casal" and
the private respondent, Domingo Palomares, filed "Comment/Opposition in Intervention", dated
December 23, 1988 asking for the outright dismissal of the petition.
On December 14, 1989, the private respondent filed a manifestation, stating, among other things, that
assuming OCT No. 291 had been cancelled, there was still basis for the respondent judge to
prevent landgrabbers from entering into vacant portions of the estate embraced thereby.
The Court finds the issues, quintessentially, to be:
(1) Is OCT No. 291 still valid and subsisting?
(2) Did the respondent judge, in issuing the orders, dated October 12 and October 23, 1987, commit
a grave abuse of discretion equivalent to lack or excess of jurisdiction?
I.
Is OCT No. 291 still
valid and subsisting?
The Court takes judicial notice of the fact that the hectarage embraced by TCT No. 192 (OCT No. 291)
consists of Government property. Three things persuade the Court: (1) the decrees of Proclamations
Nos. 192 and 435; (2) the incontrovertible fact that OCT No. 291 has been duly cancelled; and (3) the
decision of the Court of Appeals in AC-G.R. CV No. 00293, affirming the decision of Hon. Gregorio
Pineda, Judge of the then Court of First Instance of Rizal, Branch XXI, in LRC (GLRO) Rec. No. 2484,
Case No. R-1467 thereof, entitled "In Re: Issuance of Owner's Duplicate of Certificate of Title No. 291,"
as well as our own Resolution in G.R. No. 69834, entitled "Domingo Palomares, et al., v. Intermediate
Appellate Court".
(a)
Proclamation No. 192 ("RESERVING FOR THE VETERANS CENTER SITE PURPOSES CERTAIN
PARCEL OF LAND OF THE PUBLIC DOMAIN SITUATED IN THE PROVINCE OF RIZAL, ISLAND
OF LUZON") and Proclamation No. 423 ("RESERVING FOR MILITARY PURPOSES CERTAIN
PARCELS OF THE PUBLIC DOMAIN SITUATED IN THE MUNICIPALITY OF PASIG, TAGUIG, AND
PARAÑAQUE, PROVINCE OF RIZAL, AND PASAY CITY") have the character of official assertions of
ownership, and the presumption is that they have been issued by right of sovereignty and in the
exercise of the State's dominical authority. We take not only judicial notice thereof [6] but accept the
same as a valid asseveration of regalian right over property.
With respect to the premises occupied by the Libingan ng mga Bayani, Ninoy Aquino International
Airport, Nayong Pilipino, the Population Commission, National Science and Development Board, and
the National Housing Authority, we do not have the slightest doubt that they stand on Government
property by sheer presumption that, unless otherwise shown, what the Government occupies is what
the Government owns.
While there is no presumption that property is Government property until otherwise shown, because
the law recognizes private ownership, thus:
Art. 425. Property of private ownership, besides the patrimonial property of the State, provinces, cities,
and municipalities, consists of all property belonging to private persons, either individually or
collectively.[7]
Full Text Cases in Natural Resources
85
we find hard evidence on record that: (1) the property covered by OCT No. 291 had been conveyed to
the United States of America; (2) it had been later ceded to the Republic of the Philippines; and (3) as
a consequence, OCT No. 291 was cancelled upon final orders of Judge Ostrand.
Be that as it may, the private respondent in G.R. No 81564 is pressed hard to establish the fact that
portions of the property, especially the open spaces referred to in the lower court's writ of injunction
and the private respondent's manifestation of December 14, 1989, and which open spaces it claims to
be outside Maricaban are indeed outside Maricaban (or OCT 291). With respect, however, to parts
thereof on which Fort Bonifacio, Libingan ng mga Bayani, Ninoy Aquino International
Airport, Nayong Pilipino, Population Commission, National Science and Development Board, and
National Housing Authority sit, the hands of the private respondent are tied.
Claims that Judge Ostrand's decree was a counterfeit is not only self-serving, it finds no support from
the records. The presumption is "that official duty has been regularly performed," [8] and the burden is
on the private respondent to prove irregular performance. The barren insistence that
Judge Ostrand's order was a forgery is not sufficient to overthrow the presumption. To begin with, the
act of forgery has been seasonably disputed by the petitioners. Secondly, the Acting Registrar of
Deeds of Pasig, who supposedly certified to the fake character of Judge Ostrand's order has himself
joined the other petitioners in opposing the reconveyance sought.
(b)
The decision in AC-G.R. No. 00293, dismissing the private respondent's petition for the issuance of a
new owner's copy of OCT No. 291, a dismissal affirmed by this Court in G.R. No. 69834, also militates
against the return of the property to the heirs of Delfin Casal. The Appellate Court's judgment, a
judgment sustained by this Court, operates as, at the very least, the law of the case between the parties,
that OCT No. 291 has been cancelled and the land covered has been conveyed and ceded to the
National Government. The fact that AC-G.R. CV No. 00293 dealt with a petition for issuance of lost
owner's duplicate copy is no argument because be that as it may, the private respondent can not
rightfully say that the heirs of Delfin Casal still have title to the land. If it can not secure a new owner's
copy, it can mean that they have lost title thereto.
(c)
The principle of res judicata is also a bar to the instant proceedings. It should be noted that in G.R. No.
69834, Mr. Domingo Palomaresprayed:
WHEREFORE, premises considered it is most respectfully prayed to the most Honorable Supreme
Court, that in the name of law, justice and fair play, to prevent and frustrate "land-grabbing" by the
government, decision be rendered:
FIRST, That a thorough review of the aforementioned resolution of the Intermediate Appellate Court be
made;
SECOND, That after due consideration, the resolution subject of review be set aside based on
the aforestated assignment of error;
THIRD, That the Order of the Lower Court dated Jan. 19, 1977 be affirmed as the lawful and valid
order;
FOURTH, To erase all doubts by declaring OCT No. 291 as continuously and existing validly against
the whole world;
FIFTH, Clearing OCT No. 291 of all adverse claims, since the herein petitioners are the true and legally
declared heirs; and

Full Text Cases in Natural Resources


86
SIXTH, Ordering the Register of Deeds of Pasig Rizal to issue the Owner's Duplicate Copy of OCT No.
291.
Petitioner-Appellant further prays for other just and equitable reliefs.***
When we therefore denied that petition, we, in effect, held that reconstitution (of lost duplicate owner's
copy) was not possible because the mother title (OCT No. 291) had been duly cancelled. And when
we therefore declared OCT No. 291 to have been cancelled, we perished all doubts as to the invalidity
of Mr. Palomares' pretenses of title to Maricaban. Our judgment was conclusive not only as to
Mr. Palomares, but also as to the existing status of the property. As we have held:
The lower Court correctly ruled that the present action is barred by the final judgment rendered in the
previous case of Tuason & Co. vs. Aguila, Civil Case No. Q-4275, of the Court of First Instance
of Rizal. The reason is plain: if the herein appellants really had a preferential right to a conveyance of
the land from J.M. Tuason & Co., or if the certificate of (Torrens) title held by Tuason & Co. were truly
void and ineffective, then these facts should have been pleaded by these appellants in the previous
case (Q-4275), since such facts, if true, constituted a defense to the claim of Tuason & Co. for recovery
of possession. If appellants failed to plead such defenses in that previous case, they are barred from
litigating the same in any subsequent proceeding, for it is a well established rule that as between the
same parties and on the same subject and cause of action, a final judgment is conclusive not only on
matters directly adjudicated, but also as to any other matter that could have been raised in relation
thereto.[9]
II
Did the respondent judge,
in issuing the order, dated
October 12, 1987, commit a
grave abuse of discretion
equivalent to lack or excess of
jurisdiction?
(a)
The Court has no doubt that Judge Velez is here guilty of grave abuse of discretion tantamount to lack
or excess of jurisdiction to warrant certiorari. As above-stated, what he gave away, by virtue
of reconveyance, was property that inalienably belongs to the Government or its successors. Worse,
he gave away property without notice to the actual possessors, that is, the present registered owner. It
is beyond debate, as we have indicated, that the land had been, since the cancellation of OCT No.
291, parcelled out to a succession of buyers and owners. In the absence of notice, it acquired no
jurisdiction to decree redelivery or reconveyance. It is well-established that owners of property over
which reconveyance is asserted are indispensable parties, without whom no relief is available and
without whom the court can render no valid judgment.[10]
Furthermore, the present holders of the land in question are innocent purchasers for value, or presumed
to be so in the absence of contrary evidence, against whom reconveyance does not lie.[11]
(b)
The respondent judge can not conceal his faults behind arguments that he did not intend to convey the
premises, but rather, to secure, allegedly, vacant portions thereof from interlopers. First, this is not
Full Text Cases in Natural Resources
87
stated in his order. Second, that order is clear and unequivocal that Domingo Palomares has the right
"(t)o sell, exchange, lease or otherwise dispose of any area or areas or portion or portions thereof
..."[12]Third and last, the security of the property is the lookout of the claimants, and not the court's. In
case the premises the respondent judge's injunctive writ have been directed belong to others, let them
air their plaints.
(c)
The Court is also agreed that the challenged order was issued with no benefit of trial or hearing. The
private respondent can not validly rely on AC-G.R. No. 00293 as the "trial or hearing" to justify the
issuance of its said order, in the first place, because it is a different proceeding. But above all, the
private respondent itself says that AC-G.R. CV No. 00293 can not be made a basis for
denying reconveyancebecause "the ... petition was merely for the issuance of a new owner's duplicate
copy..."[13] Accordingly, it can not invoke that case and yet, repudiate its effects. It is the height of
contradiction.
(d)
It was also grave error for the lower court to deny the Solicitor General's notice of appeal. The
Government had all the right to appeal because: (1) the order of October 12, 1987 was in the nature
of a final judgment, as "final judgment" is known in law (however it is captioned), that is to say, one that
"finally disposes of the pending action so that nothing more can be done with it in the trial court;" [14](2)
it did not merely maintain the status quo, but allowed Mr. Domingo Palomares to transact on the
property by near-right of dominion over it.
Judge Velez had therefore no reason, indeed, excuse, to deny the Government's notice of
appeal. What is plain is the fact that Judge Velez was hell-bent, so to speak, in blocking the
Government's efforts to defend what rightfully belongs to it.
What has obviously been lost on the parties, Judge Velez in particular, is the established principle that
injunction does not lie "to take property out of the possession or control of one party and place it into
that of another."[15] In this wise it has also been held:
xxx xxx xxx
It is a well established doctrine in this jurisdiction that an injunction is not the proper remedy for the
recovery of possession of real estate and the improvements thereon, as well as for
the ejectments therefrom of the actual occupants who claim to have title to or material interest
therein. The use of said remedy in such cases has invariably been considered unjustified, in open
violation of the legal presumption that the bona fide possessor of a certain piece of land and
improvements thereon, holds the same under claim of ownership and with a just title, and as an
advanced concession of the remedy to which the claimant might be entitled. (Citations omitted)[16]
xxx xxx x x.x
Injunction, moreover, is an extraordinary remedy. It lies only in certain cases, to wit:
Sec. 3. Grounds for issuance of preliminary injunction.-- A preliminary injunction may be granted at
any time after the commencement of the action and before judgment when it is established:
(a) That the plaintiff is entitled to the relief demanded, and the whole or part of such relief consists in
restraining the commission or continuance of the acts complained of, or in the performance of an act
or acts, either for a limited period or perpetually;
(b) That the commission or continuance of some act complained of during the litigation or the non-
performance thereof would probably work injustice to the plaintiff; or
Full Text Cases in Natural Resources
88
(c) That the defendant is doing, threatens, or is about to do, or is procuring or suffering to be done,
some act probably in violation of the plaintiffs rights respecting the subject of the action, and tending to
render the judgment ineffectual.[17]
xxx xxx xxx
The conspicuous and unusual zeal with which Judge Francisco Velez now defends his acts[18] has not
escaped us. His honor should have borne in mind that in proceedings under Rule 65 of the Rules, such
as the present cases, the judge is included only as a nominal party. Unless otherwise ordained by this
Court, he is not called upon to answer or comment on the petition, but rather, the private respondent. It
is indeed distressing to note that it is the very judge who has taken the cudgels for the latter in defending
its interests, when he, the judge, should have remained a neutral magistrate. Res ipsa loquitor.[19] He
must get his just deserts.
III
The Court thus closes the long-drawn tale of Hacienda de Maricaban. In this connection, let trial judges
be cautioned on the indiscriminate disposition of our dwindling natural resources to private
persons. Accordingly, we grant G.R. No. 81564 and dismiss G.R. No. 90176, and so also, end what
has come down as nearly a century of uncertainty, doubt, and conflict Maricaban has left in its trail. The
Court has finally spoken. Let the matter rest.
WHEREFORE:
1. The petition in G.R. No. 81564 is GRANTED:
(a) The Writ of Preliminary Injunction issued by our Resolution, dated April 13, 1988, enjoining the
respondent judge from enforcing his: (i) order of October 12, 1987 and (ii) the follow-up order of
October 23, 1987, is made permanent; and
(b) Original Certificate of Title No. 291 is declared duly CANCELLED;
2. The petition in G.R. No. 90176 is DISMISSED; and
3. Judge Francisco Velez is ordered to SHOW CAUSE why he should not be administratively dealt
with for giving away, by virtue of reconveyance, property that inalienably belongs to the Government,
without notice to the registered owner, and without benefit of trial or hearing; for blocking Government
efforts to defend what rightfully belongs to it; and for filing his comment of June 17, 1988 and
supplemental comment of August 26, 1988 without express leave of court.
Costs against the private respondent.
SO ORDERED.

Full Text Cases in Natural Resources

S-ar putea să vă placă și